Решить матрицу онлайн методом крамера онлайн: Онлайн калькулятор. Решение систем линейных уравнений. Метод Крамера

Содержание

Решение систем линейных уравнений методом Крамера онлайн

Одним из способов решения системы линейных алгебраических уравнений (СЛАУ) является метод Крамера . Предположим, нам дана СЛАУ вида:

a11x1a12x2a13x3b1a21x1a22x2a23x3b2a31x1a32x2a33x3b3

Требуется её решить, т.е. найти такие значения переменных x1, x2, x3 чтобы при подстановке их в исходную СЛАУ, последняя обращалась в верное тождество. Чтобы проиллюстрировать метод Крамера, запишем исходную СЛАУ в матричной форме:

a11a12a13a21a22a23a31a32a33x1x2x3b1b2b3

Первым шагом метода Крамера, является нахождение определителя матрицы СЛАУ:

Δa11a12a13a21a22a23a31a32a33

Если полученный определитель отличен от нуля, тогда исходная СЛАУ имеет единственное решение, которое может быть найдено методом Крамера.

Если полученный определитель равен нулю, тогда исходная СЛАУ может не иметь решений или иметь бесконечное множество решений которые не могут быть найдены методом Крамера.

Предположим, что полученный определитель не равен нулю:

Δ0

тогда, по методу Крамера, решения находятся по формулам:

xΔxΔyΔyΔzΔzΔ

причем ∆

x, ∆y и ∆z — определители полученные из определителя ∆ путем замены соответствующего столбца на вектор свободных коэффициентов. Например, определитель ∆x получается из определителя ∆ путем замены 1-ого столбца на вектор свободных коэффициентов:

Δxb1a12a13b2a22a23b3a32a33

аналогичным образом нужно сформировать определители ∆y и ∆z. Стоит отметить, что метод Крамера применим к СЛАУ в которых число уравнений равно числу неизвестных.

Данный онлайн калькулятор решает СЛАУ методом Крамера с описанием пошагового хода решения на русском языке. Коэффициенты СЛАУ могут быть не только числами или дробями, но также и параметрами. Для работы калькулятора необходимо ввести уравнения и выбрать переменные СЛАУ, которые необходимо найти.

калькулятор крамер

Вы искали калькулятор крамер? На нашем сайте вы можете получить ответ на любой математический вопрос здесь. Подробное решение с описанием и пояснениями поможет вам разобраться даже с самой сложной задачей и калькулятор крамера, не исключение. Мы поможем вам подготовиться к домашним работам, контрольным, олимпиадам, а так же к поступлению в вуз. И какой бы пример, какой бы запрос по математике вы не ввели — у нас уже есть решение.

Например, «калькулятор крамер».

Применение различных математических задач, калькуляторов, уравнений и функций широко распространено в нашей жизни. Они используются во многих расчетах, строительстве сооружений и даже спорте. Математику человек использовал еще в древности и с тех пор их применение только возрастает. Однако сейчас наука не стоит на месте и мы можем наслаждаться плодами ее деятельности, такими, например, как онлайн-калькулятор, который может решить задачи, такие, как калькулятор крамер,калькулятор крамера,калькулятор крамера матрицы онлайн,калькулятор крамера онлайн,калькулятор крамера с решением,калькулятор матриц крамера,калькулятор матриц метод крамера,калькулятор матриц метод крамера онлайн,калькулятор матриц методом крамера,калькулятор матриц онлайн метод крамера,калькулятор матриц онлайн методом крамера,калькулятор матриц онлайн с решением метод крамера,калькулятор матриц онлайн с решением методом крамера,калькулятор матриц с решением методом крамера онлайн,калькулятор матрица метод крамера,калькулятор матрицы крамера онлайн,калькулятор матрицы метод крамера,калькулятор матрицы метод крамера онлайн,калькулятор матрицы методом крамера,калькулятор матрицы онлайн метод крамера,калькулятор матрицы онлайн методом крамера,калькулятор матрицы онлайн с решением метод крамера,калькулятор матричный метод крамера,калькулятор метод крамера,калькулятор метод крамера онлайн с решением,калькулятор метода крамера,калькулятор методом крамера,калькулятор методом крамера онлайн,калькулятор онлайн для метода крамера,калькулятор онлайн матриц метод крамера,калькулятор онлайн матрицы метод крамера,калькулятор онлайн матрицы методом крамера,калькулятор онлайн методом крамера,калькулятор онлайн по формуле крамера онлайн калькулятор,калькулятор онлайн правило крамера,калькулятор онлайн решение матриц методом крамера,калькулятор онлайн решение систем методом крамера,калькулятор по методу крамера,калькулятор по формуле крамера,калькулятор решение методом крамера,калькулятор решение систем линейных уравнений методом крамера,калькулятор решение систем методом крамера,калькулятор решение уравнений методом крамера,калькулятор решения методом крамера,калькулятор системы уравнений онлайн методом крамера онлайн,калькулятор слау методом крамера,калькулятор формула крамера,калькулятор формула крамера онлайн,крамер калькулятор,крамер калькулятор онлайн,крамер онлайн,крамер онлайн калькулятор,крамера онлайн,крамера онлайн калькулятор,крамера решение онлайн,линейное уравнение методом крамера онлайн,матрица калькулятор метод крамера,матрица калькулятор онлайн метод крамера,матрица крамера онлайн,матрица метод крамера калькулятор,матрица метод крамера онлайн,матрица метод крамера онлайн калькулятор,матрица методом крамера онлайн,матрица онлайн калькулятор метод крамера,матрица онлайн крамера,матрица онлайн метод крамера,матрица онлайн методом крамера,матрицы калькулятор метод крамера,матрицы калькулятор методом крамера,матрицы калькулятор онлайн крамера,матрицы калькулятор онлайн метод крамера,матрицы метод крамера калькулятор,матрицы метод крамера онлайн,матрицы метод крамера онлайн калькулятор,матрицы метод крамера онлайн калькулятор с подробным решением,матрицы методом крамера калькулятор,матрицы онлайн калькулятор крамера,матрицы онлайн калькулятор метод крамера,матрицы онлайн калькулятор методом крамера,матрицы онлайн калькулятор с подробным решением метод крамера,матрицы онлайн калькулятор с решением метод крамера,матрицы онлайн метод крамера,матрицы онлайн методом крамера онлайн,матрицы онлайн решение методом крамера,матрицы онлайн решение методом крамера онлайн с,матрицы решение методом крамера онлайн,матрицы решение методом крамера онлайн с решением,матрицы решение онлайн методом крамера,матрицы решение онлайн методом крамера онлайн с,матричный калькулятор метод крамера,метод крамера калькулятор,метод крамера калькулятор онлайн,метод крамера калькулятор с подробным решением,метод крамера матриц онлайн калькулятор,метод крамера матрица онлайн,метод крамера матрицы калькулятор,метод крамера матрицы онлайн,метод крамера матрицы онлайн калькулятор,метод крамера матричный калькулятор,метод крамера онлайн,метод крамера онлайн калькулятор,метод крамера онлайн калькулятор матрицы,метод крамера онлайн калькулятор с подробным решением,метод крамера онлайн калькулятор с подробным решением и с проверкой,метод крамера онлайн калькулятор с подробным решением матрицы,метод крамера онлайн калькулятор с решением,метод крамера онлайн решение,метод крамера онлайн решение матриц,метод крамера онлайн с решением,метод крамера примеры с решением калькулятор,метод крамера примеры с решением онлайн,метод крамера решение матриц онлайн,метод крамера решение онлайн,метод крамера решения систем линейных уравнений онлайн,метод крамера с решением онлайн,метод решение матриц крамера онлайн,метода крамера калькулятор,методом крамера калькулятор,методом крамера онлайн,методом крамера онлайн калькулятор,методом крамера решить онлайн,методом крамера решить уравнение онлайн,онлайн калькулятор для метода крамера,онлайн калькулятор крамер,онлайн калькулятор крамера,онлайн калькулятор линейных уравнений методом крамера онлайн,онлайн калькулятор матриц метод крамера,онлайн калькулятор матриц метод крамера с решением,онлайн калькулятор матриц методом крамера,онлайн калькулятор матриц с решением метод крамера,онлайн калькулятор матрицы крамера,онлайн калькулятор матрицы метод крамера,онлайн калькулятор матрицы методом крамера,онлайн калькулятор метод крамера,онлайн калькулятор метод крамера матрицы,онлайн калькулятор метод крамера с решением,онлайн калькулятор методом крамера,онлайн калькулятор правило крамера,онлайн калькулятор решение линейных уравнений методом крамера,онлайн калькулятор решение матриц методом крамера,онлайн калькулятор решение методом крамера,онлайн калькулятор решение систем линейных уравнений методом крамера,онлайн калькулятор решение систем методом крамера,онлайн калькулятор решение систем уравнений методом крамера,онлайн калькулятор решение системы уравнений методом крамера,онлайн калькулятор решить систему методом крамера,онлайн калькулятор решить систему по правилу крамера,онлайн калькулятор систем линейных уравнений методом крамера онлайн,онлайн калькулятор систем методом крамера онлайн,онлайн калькулятор систем уравнений методом крамера онлайн,онлайн калькулятор система уравнений методом крамера онлайн,онлайн калькулятор системы уравнений методом крамера онлайн,онлайн калькулятор формула крамера,онлайн калькулятор формулы крамера,онлайн крамера,онлайн матрица крамера,онлайн матрица метод крамера,онлайн матрица методом крамера,онлайн матрицы метод крамера,онлайн методом крамера,онлайн правило крамера,онлайн решение крамера,онлайн решение крамера метод,онлайн решение линейных уравнений методом крамера,онлайн решение матриц крамера,онлайн решение матриц метод крамера,онлайн решение матрицы методом крамера,онлайн решение метод крамера,онлайн решение методом крамера,онлайн решение по крамеру,онлайн решение по формулам крамера онлайн,онлайн решение по формуле крамера,онлайн решение систем линейных уравнений методом крамера,онлайн решение систем линейных уравнений методом крамера онлайн,онлайн решение систем методом крамера,онлайн решение систем методом крамера онлайн с подробным решением,онлайн решение систем уравнений методом крамера,онлайн решение системы линейных уравнений методом крамера,онлайн решение системы линейных уравнений методом крамера онлайн,онлайн решение системы методом крамера,онлайн решение системы методом крамера онлайн калькулятор,онлайн решение системы уравнений методом крамера,онлайн решение слау методом крамера,онлайн решение уравнений крамера,онлайн решение уравнений методом крамера,онлайн решение уравнения методом крамера,онлайн решить систему линейных уравнений методом крамера,онлайн система крамера,онлайн система линейных уравнений методом крамера онлайн,онлайн система уравнений методом крамера,онлайн формула крамера,определитель онлайн методом крамера онлайн,по крамеру решение онлайн,по формулам крамера решить систему онлайн,по формуле крамера калькулятор,по формуле крамера решить систему уравнений онлайн,правило крамера калькулятор онлайн,правило крамера онлайн,правило крамера онлайн калькулятор,решение крамера онлайн,решение линейных систем уравнений методом крамера онлайн,решение линейных уравнений методом крамера калькулятор онлайн,решение линейных уравнений методом крамера онлайн,решение линейных уравнений методом крамера онлайн калькулятор,решение линейных уравнений онлайн методом крамера,решение матриц крамера онлайн,решение матриц метод крамера онлайн,решение матриц методом крамера онлайн,решение матриц методом крамера онлайн калькулятор,решение матриц методом крамера онлайн с подробным решением,решение матриц онлайн калькулятор методом крамера,решение матриц онлайн крамера,решение матриц онлайн метод крамера,решение матриц онлайн метод крамера онлайн,решение матриц онлайн методом крамера,решение матриц онлайн методом крамера онлайн,решение матриц онлайн методом крамера с подробным решением,решение матриц онлайн с подробным решением методом крамера,решение матрицы крамера онлайн,решение матрицы методом крамера онлайн,решение матрицы методом крамера онлайн с подробным решением,решение матрицы методом крамера онлайн с решением,решение матрицы онлайн крамера,решение матрицы онлайн методом крамера,решение матрицы онлайн методом крамера онлайн,решение матрицы онлайн методом крамера онлайн с,решение матрицы онлайн с подробным решением методом крамера,решение матричных уравнений методом крамера онлайн,решение матричных уравнений онлайн методом крамера,решение метод крамера онлайн,решение метод крамера решение онлайн,решение методом крамера калькулятор,решение методом крамера калькулятор онлайн,решение методом крамера матрицы онлайн,решение методом крамера матрицы онлайн калькулятор,решение методом крамера онлайн,решение методом крамера онлайн калькулятор,решение методом крамера онлайн с подробным решением,решение онлайн крамера,решение онлайн линейных уравнений методом крамера,решение онлайн матриц метод крамера,решение онлайн матрицы методом крамера,решение онлайн метод крамера,решение онлайн метод крамера онлайн,решение онлайн методом крамера,решение онлайн по крамеру,решение онлайн по формуле крамера,решение онлайн систем линейных уравнений методом крамера,решение онлайн систем линейных уравнений методом крамера онлайн,решение онлайн систем методом крамера,решение онлайн систем методом крамера онлайн с подробным решением,решение онлайн системы методом крамера онлайн калькулятор,решение онлайн слау методом крамера,решение по крамеру онлайн,решение по методу крамера онлайн,решение по формуле крамера онлайн,решение систем линейных уравнений методом крамера калькулятор,решение систем линейных уравнений методом крамера калькулятор онлайн,решение систем линейных уравнений методом крамера онлайн,решение систем линейных уравнений методом крамера онлайн калькулятор,решение систем линейных уравнений методом крамера онлайн с решением,решение систем линейных уравнений онлайн методом крамера,решение систем линейных уравнений онлайн методом крамера онлайн,решение систем линейных уравнений онлайн методом крамера онлайн с,решение систем методом крамера калькулятор,решение систем методом крамера калькулятор онлайн,решение систем методом крамера онлайн,решение систем методом крамера онлайн калькулятор,решение систем онлайн методом крамера,решение систем онлайн методом крамера онлайн с подробным решением,решение систем уравнений метод крамера онлайн,решение систем уравнений методом крамера онлайн,решение систем уравнений методом крамера онлайн калькулятор,решение систем уравнений методом крамера онлайн с подробным решением,решение систем уравнений онлайн метод крамера,решение систем уравнений онлайн методом крамера,решение систем уравнений онлайн методом крамера онлайн,решение систем уравнений онлайн с подробным решением методом крамера,решение системы линейных уравнений методом крамера онлайн,решение системы линейных уравнений методом крамера онлайн с решением,решение системы линейных уравнений онлайн методом крамера,решение системы линейных уравнений онлайн методом крамера онлайн,решение системы линейных уравнений онлайн методом крамера с решением,решение системы методом крамера онлайн,решение системы онлайн методом крамера,решение системы уравнений методом крамера онлайн,решение системы уравнений методом крамера онлайн калькулятор,решение системы уравнений методом крамера онлайн с решением,решение системы уравнений онлайн методом крамера,решение слау методом крамера онлайн,решение слау методом крамера онлайн с решением,решение слау онлайн методом крамера,решение уравнений крамера онлайн,решение уравнений методом крамера калькулятор,решение уравнений методом крамера онлайн,решение уравнений онлайн крамера,решение уравнений онлайн методом крамера,решение уравнений онлайн по формуле крамера онлайн,решение уравнения методом крамера онлайн,решение уравнения онлайн методом крамера,решить линейное уравнение методом крамера онлайн,решить линейное уравнение онлайн методом крамера,решить матрицу методом крамера онлайн,решить матрицу методом крамера онлайн калькулятор,решить матрицу методом крамера онлайн с подробным решением,решить матрицу онлайн калькулятор методом крамера,решить матрицу онлайн методом крамера,решить матрицу онлайн методом крамера онлайн,решить методом крамера онлайн,решить методом крамера систему линейных уравнений онлайн,решить онлайн матрицу методом крамера,решить онлайн систему линейных уравнений методом крамера,решить онлайн слау методом крамера,решить онлайн уравнение методом крамера,решить онлайн уравнение методом крамера с подробным решением,решить по формуле крамера онлайн,решить систему линейных уравнений методом крамера онлайн,решить систему линейных уравнений методом крамера онлайн с решением,решить систему линейных уравнений методом крамера решить онлайн,решить систему линейных уравнений онлайн методом крамера,решить систему линейных уравнений онлайн по формулам крамера,решить систему линейных уравнений по формулам крамера онлайн,решить систему методом крамера онлайн,решить систему методом крамера онлайн калькулятор,решить систему методом крамера онлайн с подробным решением,решить систему онлайн методом крамера,решить систему по правилу крамера онлайн,решить систему по правилу крамера онлайн калькулятор,решить систему по формулам крамера онлайн,решить систему по формулам крамера онлайн калькулятор,решить систему уравнений методом крамера онлайн,решить систему уравнений методом крамера онлайн с подробным решением,решить систему уравнений онлайн методом крамера,решить систему уравнений онлайн по формуле крамера,решить систему уравнений по формулам крамера онлайн с решением,решить систему уравнений по формуле крамера онлайн,решить слау методом крамера онлайн,решить слау онлайн методом крамера,решить уравнение методом крамера онлайн,решить уравнение методом крамера онлайн с подробным решением,решить уравнение онлайн методом крамера,система линейных уравнений методом крамера онлайн,система линейных уравнений методом крамера онлайн калькулятор,система линейных уравнений онлайн калькулятор методом крамера,система линейных уравнений онлайн методом крамера,система линейных уравнений онлайн методом крамера онлайн,система методом крамера онлайн,система уравнений методом крамера онлайн,система уравнений онлайн методом крамера,систему линейных уравнений решить по формулам крамера онлайн,слау методом крамера калькулятор,слау методом крамера онлайн,слау методом крамера онлайн решение,слау онлайн методом крамера онлайн,слау решение методом крамера онлайн,спецкласс ру онлайн калькулятор,формула крамера для решения системы линейных уравнений онлайн,формула крамера калькулятор,формула крамера онлайн,формула крамера онлайн калькулятор,формулы крамера онлайн калькулятор.

На этой странице вы найдёте калькулятор, который поможет решить любой вопрос, в том числе и калькулятор крамер. Просто введите задачу в окошко и нажмите «решить» здесь (например, калькулятор крамера матрицы онлайн).

Где можно решить любую задачу по математике, а так же калькулятор крамер Онлайн?

Решить задачу калькулятор крамер вы можете на нашем сайте https://pocketteacher.ru. Бесплатный онлайн решатель позволит решить онлайн задачу любой сложности за считанные секунды. Все, что вам необходимо сделать — это просто ввести свои данные в решателе. Так же вы можете посмотреть видео инструкцию и узнать, как правильно ввести вашу задачу на нашем сайте. А если у вас остались вопросы, то вы можете задать их в чате снизу слева на странице калькулятора.

Системы линейных уравнений методом крамера онлайн. Линейные уравнения. Решение систем линейных уравнений.

Метод Крамера

Для того чтобы освоить данный параграф Вы должны уметь раскрывать определители «два на два» и «три на три». Если с определителями плохо, пожалуйста, изучите урок Как вычислить определитель?

Сначала мы подробно рассмотрим правило Крамера для системы двух линейных уравнений с двумя неизвестными. Зачем? – Ведь простейшую систему можно решить школьным методом, методом почленного сложения!

Дело в том, что пусть иногда, но встречается такое задание – решить систему двух линейных уравнений с двумя неизвестными по формулам Крамера. Во-вторых, более простой пример поможет понять, как использовать правило Крамера для более сложного случая – системы трех уравнений с тремя неизвестными.

Кроме того, существуют системы линейных уравнений с двумя переменными, которые целесообразно решать именно по правилу Крамера!

Рассмотрим систему уравнений

На первом шаге вычислим определитель , его называют главным определителем системы .

метод Гаусса .

Если , то система имеет единственное решение, и для нахождения корней мы должны вычислить еще два определителя:
и

На практике вышеуказанные определители также могут обозначаться латинской буквой .

Корни уравнения находим по формулам:
,

Пример 7

Решить систему линейных уравнений

Решение : Мы видим, что коэффициенты уравнения достаточно велики, в правой части присутствуют десятичные дроби с запятой. Запятая – довольно редкий гость в практических заданиях по математике, эту систему я взял из эконометрической задачи.

Как решить такую систему? Можно попытаться выразить одну переменную через другую, но в этом случае наверняка получатся страшные навороченные дроби, с которыми крайне неудобно работать, да и оформление решения будет выглядеть просто ужасно. Можно умножить второе уравнение на 6 и провести почленное вычитание, но и здесь возникнут те же самые дроби.

Что делать? В подобных случаях и приходят на помощь формулы Крамера.

;

;

Ответ : ,

Оба корня обладают бесконечными хвостами, и найдены приближенно, что вполне приемлемо (и даже обыденно) для задач эконометрики.

Комментарии здесь не нужны, поскольку задание решается по готовым формулам, однако, есть один нюанс. Когда используете данный метод, обязательным фрагментом оформления задания является следующий фрагмент: «, значит, система имеет единственное решение» . В противном случае рецензент может Вас наказать за неуважение к теореме Крамера.

Совсем не лишней будет проверка, которую удобно провести на калькуляторе: подставляем приближенные значения в левую часть каждого уравнения системы. В результате с небольшой погрешностью должны получиться числа, которые находятся в правых частях.

Пример 8

Ответ представить в обыкновенных неправильных дробях. Сделать проверку.

Это пример для самостоятельного решения (пример чистового оформления и ответ в конце урока).

Переходим к рассмотрению правила Крамера для системы трех уравнений с тремя неизвестными:

Находим главный определитель системы:

Если , то система имеет бесконечно много решений или несовместна (не имеет решений). В этом случае правило Крамера не поможет, нужно использовать метод Гаусса .

Если , то система имеет единственное решение и для нахождения корней мы должны вычислить еще три определителя:
, ,

И, наконец, ответ рассчитывается по формулам:

Как видите, случай «три на три» принципиально ничем не отличается от случая «два на два», столбец свободных членов последовательно «прогуливается» слева направо по столбцам главного определителя.

Пример 9

Решить систему по формулам Крамера.

Решение : Решим систему по формулам Крамера.

, значит, система имеет единственное решение.

Ответ : .

Собственно, здесь опять комментировать особо нечего, ввиду того, что решение проходит по готовым формулам. Но есть пара замечаний.

Бывает так, что в результате вычислений получаются «плохие» несократимые дроби, например: .
Я рекомендую следующий алгоритм «лечения». Если под рукой нет компьютера, поступаем так:

1) Возможно, допущена ошибка в вычислениях. Как только Вы столкнулись с «плохой» дробью, сразу необходимо проверить, правильно ли переписано условие . Если условие переписано без ошибок, то нужно пересчитать определители, используя разложение по другой строке (столбцу).

2) Если в результате проверки ошибок не выявлено, то вероятнее всего, допущена опечатка в условии задания. В этом случае спокойно и ВНИМАТЕЛЬНО прорешиваем задание до конца, а затем обязательно делаем проверку и оформляем ее на чистовике после решения. Конечно, проверка дробного ответа – занятие неприятное, но зато будет обезоруживающий аргумент для преподавателя, который ну очень любит ставить минус за всякую бяку вроде . Как управляться с дробями, подробно расписано в ответе для Примера 8.

Если под рукой есть компьютер, то для проверки используйте автоматизированную программу, которую можно бесплатно скачать в самом начале урока. Кстати, выгоднее всего сразу воспользоваться программой (еще до начала решения), Вы сразу будете видеть промежуточный шаг, на котором допустили ошибку! Этот же калькулятор автоматически рассчитывает решение системы матричным методом.

Замечание второе. Время от времени встречаются системы в уравнениях которых отсутствуют некоторые переменные, например:

Здесь в первом уравнении отсутствует переменная , во втором – переменная . В таких случаях очень важно правильно и ВНИМАТЕЛЬНО записать главный определитель:
– на месте отсутствующих переменных ставятся нули.
Кстати определители с нулями рационально раскрывать по той строке (столбцу), в которой находится ноль, так как вычислений получается заметно меньше.

Пример 10

Решить систему по формулам Крамера.

Это пример для самостоятельного решения (образец чистового оформления и ответ в конце урока).

Для случая системы 4 уравнений с 4 неизвестными формулы Крамера записываются по аналогичным принципам. Живой пример можно посмотреть на уроке Свойства определителя. Понижение порядка определителя – пять определителей 4-го порядка вполне решабельны. Хотя задача уже весьма напоминает ботинок профессора на груди у студента-счастливчика.


Решение системы с помощью обратной матрицы

Метод обратной матрицы – это, по существу, частный случай матричного уравнения (см. Пример №3 указанного урока).

Для изучения данного параграфа необходимо уметь раскрывать определители, находить обратную матрицу и выполнять матричное умножение. Соответствующие ссылки будут даны по ходу объяснений.

Пример 11

Решить систему с матричным методом

Решение : Запишем систему в матричной форме:
, где

Пожалуйста, посмотрите на систему уравнений и на матрицы. По какому принципу записываем элементы в матрицы, думаю, всем понятно. Единственный комментарий: если бы в уравнениях отсутствовали некоторые переменные, то на соответствующих местах в матрице нужно было бы поставить нули.

Обратную матрицу найдем по формуле:
, где – транспонированная матрица алгебраических дополнений соответствующих элементов матрицы .

Сначала разбираемся с определителем:

Здесь определитель раскрыт по первой строке.

Внимание! Если , то обратной матрицы не существует, и решить систему матричным методом невозможно. В этом случае система решается методом исключения неизвестных (методом Гаусса) .

Теперь нужно вычислить 9 миноров и записать их в матрицу миноров

Справка: Полезно знать смысл двойных подстрочных индексов в линейной алгебре. Первая цифра – это номер строки, в которой находится данный элемент. Вторая цифра – это номер столбца, в котором находится данный элемент:

То есть, двойной подстрочный индекс указывает, что элемент находится в первой строке, третьем столбце, а, например, элемент находится в 3 строке, 2 столбце

В ходе решения расчет миноров лучше расписать подробно, хотя, при определенном опыте их можно приноровиться считать с ошибками устно.


2. Решение систем уравнений матричным методом (при помощи обратной матрицы).
3. Метод Гаусса решения систем уравнений.

Метод Крамера.

Метод Крамера применяется для решения систем линейных алгебраических уравнений (СЛАУ ).

Формулы на примере системы из двух уравнений с двумя переменными.
Дано: Решить методом Крамера систему

Относительно переменных х и у .
Решение:
Найдем определитель матрицы, составленный из коэффициентов системы Вычисление определителей. :



Применим формулы Крамера и найдем значения переменных:
и .
Пример 1:
Решить систему уравнений:

относительно переменных х и у .
Решение:


Заменим в этом определителе первый столбец столбцом коэффициентов из правой части системы и найдем его значение:

Сделаем аналогичное действие, заменив в первом определителе второй столбец:

Применим формулы Крамера и найдем значения переменных:
и .
Ответ:
Замечание: Этим методом можно решать системы и большей размерности.

Замечание: Если получается, что , а делить на ноль нельзя, то говорят, что система не имеет единственного решения. В этом случае система имеет или бесконечно много решений или не имеет решений вообще.

Пример 2 (бесконечное количество решений):

Решить систему уравнений:

относительно переменных х и у .
Решение:
Найдем определитель матрицы, составленный из коэффициентов системы:

Решение систем методом подстановки.

Первое из уравнений системы — равенство, верное при любых значениях переменных (потому что 4 всегда равно 4). Значит, остается только одно уравнение. Это уравнение связи между переменными .
Получили, решением системы являются любые пары значений переменных, связанных между собой равенством .
Общее решение запишется так:
Частные решения можно определять выбирая произвольное значение у и вычисляя х по этому равенству связи.

и т.д.
Таких решений бесконечно много.
Ответ: общее решение
Частные решения:

Пример 3 (решений нет, система несовместна):

Решить систему уравнений:

Решение:
Найдем определитель матрицы, составленный из коэффициентов системы:

Применять формулы Крамера нельзя. Решим эту систему методом подстановки

Второе уравнение системы — равенство, неверное ни при каких значениях переменных (конечно же, так как -15 не равно 2). Если одно из уравнений системы не верно ни при каких значениях переменных, то и вся системы не имеет решений.
Ответ: решений нет

Метод Крамера основан на использовании определителей в решении систем линейных уравнений. Это значительно ускоряет процесс решения.

Метод Крамера может быть использован в решении системы стольких линейных уравнений, сколько в каждом уравнении неизвестных. Если определитель системы не равен нулю, то метод Крамера может быть использован в решении, если же равен нулю, то не может. Кроме того, метод Крамера может быть использован в решении систем линейных уравнений, имеющих единственное решение.

Определение . Определитель, составленный из коэффициентов при неизвестных, называется определителем системы и обозначается (дельта).

Определители

получаются путём замены коэффициентов при соответствующих неизвестных свободными членами:

;

.

Теорема Крамера . Если определитель системы отличен от нуля, то система линейных уравнений имеет одно единственное решение, причём неизвестное равно отношению определителей. В знаменателе – определитель системы, а в числителе – определитель, полученный из определителя системы путём замены коэффициентов при этом неизвестном свободными членами. Эта теорема имеет место для системы линейных уравнений любого порядка.

Пример 1. Решить систему линейных уравнений:

Согласно теореме Крамера имеем:

Итак, решение системы (2):

онлайн-калькулятором , решающим методом Крамера.

Три случая при решении систем линейных уравнений

Как явствует из теоремы Крамера , при решении системы линейных уравнений могут встретиться три случая:

Первый случай: система линейных уравнений имеет единственное решение

(система совместна и определённа)

Второй случай: система линейных уравнений имеет бесчисленное множество решений

(система совместна и неопределённа)

** ,

т. е. коэффициенты при неизвестных и свободные члены пропорциональны.

Третий случай: система линейных уравнений решений не имеет

(система несовместна)

Итак, система m линейных уравнений с n переменными называется несовместной , если у неё нет ни одного решения, и совместной , если она имеет хотя бы одно решение. Совместная система уравнений, имеющая только одно решение, называется определённой , а более одного – неопределённой .

Примеры решения систем линейных уравнений методом Крамера

Пусть дана система

.

На основании теоремы Крамера

………….
,

где

определитель системы. Остальные определители получим, заменяя столбец с коэффициентами соответствующей переменной (неизвестного) свободными членами:

Пример 2.

Следовательно, система является определённой. Для нахождения её решения вычисляем определители

По формулам Крамера находим:

Итак, (1; 0; -1) – единственное решение системы.

Для проверки решений систем уравнений 3 Х 3 и 4 Х 4 можно воспользоваться онлайн-калькулятором , решающим методом Крамера.

Если в системе линейных уравнений в одном или нескольких уравнениях отсутствуют какие-либо переменные, то в определителе соответствующие им элементы равны нулю! Таков следующий пример.

Пример 3. Решить систему линейных уравнений методом Крамера:

.

Решение. Находим определитель системы:

Посмотрите внимательно на систему уравнений и на определитель системы и повторите ответ на вопрос, в каких случаях один или несколько элементов определителя равны нулю. Итак, определитель не равен нулю, следовательно, система является определённой. Для нахождения её решения вычисляем определители при неизвестных

По формулам Крамера находим:

Итак, решение системы — (2; -1; 1).

Для проверки решений систем уравнений 3 Х 3 и 4 Х 4 можно воспользоваться онлайн-калькулятором , решающим методом Крамера.

К началу страницы

Продолжаем решать системы методом Крамера вместе

Как уже говорилось, если определитель системы равен нулю, а определители при неизвестных не равны нулю, система несовместна, то есть решений не имеет. Проиллюстрируем следующим примером.

Пример 6. Решить систему линейных уравнений методом Крамера:

Решение. Находим определитель системы:

Определитель системы равен нулю, следовательно, система линейных уравнений либо несовместна и определённа, либо несовместна, то есть не имеет решений. Для уточнения вычисляем определители при неизвестных

Определители при неизвестных не равны нулю, следовательно, система несовместна, то есть не имеет решений.

Для проверки решений систем уравнений 3 Х 3 и 4 Х 4 можно воспользоваться онлайн-калькулятором , решающим методом Крамера.

В задачах на системы линейных уравнений встречаются и такие, где кроме букв, обозначающих переменные, есть ещё и другие буквы. Эти буквы обозначают некоторое число, чаще всего действительное. На практике к таким уравнениям и системам уравнений приводят задачи на поиск общих свойств каких-либо явлений и предметов. То есть, изобрели вы какой-либо новый материал или устройство, а для описания его свойств, общих независимо от величины или количества экземпляра, нужно решить систему линейных уравнений, где вместо некоторых коэффициентов при переменных — буквы. За примерами далеко ходить не надо.

Следующий пример — на аналогичную задачу, только увеличивается количество уравнений, переменных, и букв, обозначающих некоторое действительное число.

Пример 8. Решить систему линейных уравнений методом Крамера:

Решение. Находим определитель системы:

Находим определители при неизвестных

Метод Крамера применяется для решения систем линейных алгебраических уравнений (СЛАУ), в которых число неизвестных переменных равно числу уравнений и определитель основной матрицы отличен от нуля. В этой статье мы разберем как по методу Крамера находятся неизвестные переменные и получим формулы. После этого перейдем к примерам и подробно опишем решение систем линейных алгебраических уравнений методом Крамера.

Навигация по странице.

Метод Крамера — вывод формул.

Пусть нам требуется решить систему линейных уравнений вида

Где x 1 , x 2 , …, x n – неизвестные переменные, a i j , i = 1, 2, …, n, j = 1, 2, …, n – числовые коэффициенты, b 1 , b 2 , …, b n — свободные члены. Решением СЛАУ называется такой набор значений x 1 , x 2 , …, x n при которых все уравнения системы обращаются в тождества.

В матричном виде эта система может быть записана как A ⋅ X = B , где — основная матрица системы, ее элементами являются коэффициенты при неизвестных переменных, — матрица – столбец свободных членов, а — матрица – столбец неизвестных переменных. После нахождения неизвестных переменных x 1 , x 2 , …, x n , матрица становится решением системы уравнений и равенство A ⋅ X = B обращается в тождество .

Будем считать, что матрица А – невырожденная, то есть, ее определитель отличен от нуля. В этом случае система линейных алгебраических уравнений имеет единственное решение, которое может быть найдено методом Крамера. (Методы решения систем при разобраны в разделе решение систем линейных алгебраических уравнений).

Метод Крамера основывается на двух свойствах определителя матрицы:

Итак, приступим к нахождению неизвестной переменной x 1 . Для этого умножим обе части первого уравнения системы на А 1 1 , обе части второго уравнения – на А 2 1 , и так далее, обе части n-ого уравнения – на А n 1 (то есть, уравнения системы умножаем на соответствующие алгебраические дополнения первого столбца матрицы А ):

Сложим все левые части уравнения системы, сгруппировав слагаемые при неизвестных переменных x 1 , x 2 , …, x n , и приравняем эту сумму к сумме всех правых частей уравнений:

Если обратиться к озвученным ранее свойствам определителя, то имеем

и предыдущее равенство примет вид

откуда

Аналогично находим x 2 . Для этого умножаем обе части уравнений системы на алгебраические дополнения второго столбца матрицы А :

Складываем все уравнения системы, группируем слагаемые при неизвестных переменных x 1 , x 2 , …, x n и применяем свойства определителя:

Откуда
.

Аналогично находятся оставшиеся неизвестные переменные.

Если обозначить

То получаем формулы для нахождения неизвестных переменных по методу Крамера .

Замечание.

Если система линейных алгебраических уравнений однородная, то есть , то она имеет лишь тривиальное решение (при ). Действительно, при нулевых свободных членах все определители будут равны нулю, так как будут содержать столбец нулевых элементов. Следовательно, формулы дадут .

Алгоритм решения систем линейных алгебраических уравнений методом Крамера.

Запишем алгоритм решения систем линейных алгебраических уравнений методом Крамера .

Примеры решения систем линейных алгебраических уравнений методом Крамера.

Разберем решения нескольких примеров.

Пример.

Найдите решение неоднородной системы линейных алгебраических уравнений методом Крамера .

Решение.

Основная матрица системы имеет вид . Вычислим ее определитель по формуле :

Так как определитель основной матрицы системы отличен от нуля, то СЛАУ имеет единственное решение, и оно может быть найдено методом Крамера. Запишем определители и . Заменяем первый столбец основной матрицы системы на столбец свободных членов, и получаем определитель . Аналогично заменяем второй столбец основной матрицы на столбец свободных членов, и получаем .

Вычисляем эти определители:

Находим неизвестные переменные x 1 и x 2 по формулам :

Выполним проверку. Подставим полученные значения x 1 и x 2 в исходную систему уравнений:

Оба уравнения системы обращаются в тождества, следовательно, решение найдено верно.

Ответ:

.

Некоторые элементы основной матрицы СЛАУ могут быть равны нулю. В этом случае в уравнениях системы будут отсутствовать соответствующие неизвестные переменные. Разберем пример.

Пример.

Найдите решение системы линейных уравнений методом Крамера .

Решение.

Перепишем систему в виде , чтобы стало видно основную матрицу системы . Найдем ее определитель по формуле

Имеем

Определитель основной матрицы отличен от нуля, следовательно, система линейных уравнений имеет единственное решение. Найдем его методом Крамера. Вычислим определители :

Таким образом,

Ответ:

Обозначения неизвестных переменных в уравнениях системы могут отличаться от x 1 , x 2 , …, x n . Это не влияет на процесс решения. А вот порядок следования неизвестных переменных в уравнениях системы очень важен при составлении основной матрицы и необходимых определителей метода Крамера. Поясним этот момент на примере.

Пример.

Используя метод Крамера, найдите решение системы трех линейных алгебраических уравнений с тремя неизвестными .

Решение.

В данном примере неизвестные переменные имеют другое обозначение (x , y и z вместо x 1 , x 2 и x 3 ). Это не влияет на ход решения, но будьте внимательны с обозначениями переменных. В качестве основной матрицы системы НЕЛЬЗЯ брать . Необходимо сначала упорядочить неизвестные переменные во всех уравнениях системы. Для этого перепишем систему уравнений как . Теперь основную матрицу системы хорошо видно . Вычислим ее определитель:

Определитель основной матрицы отличен от нуля, следовательно, система уравнений имеет единственное решение. Найдем его методом Крамера. Запишем определители (обратите внимание на обозначения) и вычислим их:

Осталось найти неизвестные переменные по формулам :

Выполним проверку. Для этого умножим основную матрицу на полученное решение (при необходимости смотрите раздел ):

В результате получили столбец свободных членов исходной системы уравнений, поэтому решение найдено верно.

Ответ:

x = 0, y = -2, z = 3 .

Пример.

Решите методом Крамера систему линейных уравнений , где a и b – некоторые действительные числа.

Решение.

Ответ:

Пример.

Найдите решение системы уравнений методом Крамера, — некоторое действительное число.

Решение.

Вычислим определитель основной матрицы системы: . выражения есть интервал , поэтому при любых действительных значениях . Следовательно, система уравнений имеет единственное решение, которое может быть найдено методом Крамера. Вычисляем и :

Пусть система линейных уравнений содержит столько уравнений, каково количество независимых переменных, т.е. имеет вид

Такие системы линейных уравнений называются квадратными. Определитель, составленный из коэффициентов при независимых переменных системы (1.5), называется главным определителем системы. Мы будем обозначать его греческой буквой D. Таким образом,

. (1.6)

Если в главном определителе произвольный (j -ый) столбец, заменить столбцом свободных членов системы (1. 5), то можно получить еще n вспомогательных определителей:

(j = 1, 2, …, n ). (1.7)

Правило Крамера решения квадратных систем линейных уравнений заключается в следующем. Если главный определитель D системы (1.5) отличен от нуля, то система имеет и притом единственное решение, которое можно найти по формулам:

(1.8)

Пример 1.5. Методом Крамера решить систему уравнений

.

Вычислим главный определитель системы:

Так как D¹0, то система имеет единственное решение, которое можно найти по формулам (1.8):

Таким образом,

Действия над матрицами

1. Умножение матрицы на число. Операция умножения матрицы на число определяется следующим образом.

2. Для того чтобы умножить матрицу на число, нужно все ее элементы умножить на это число. То есть

. (1.9)

Пример 1.6. .

Сложение матриц.

Данная операция вводится только для матриц одного и того же порядка.

Для того чтобы сложить две матрицы, необходимо к элементам одной матрицы прибавить соответствующие элементы другой матрицы:

(1.10)
Операция сложения матриц обладает свойствами ассоциативности и коммутативности.

Пример 1.7. .

Умножение матриц.

Если число столбцов матрицы А совпадает с числом строк матрицы В , то для таких матриц вводится операция умножения:

2

Таким образом, при умножении матрицы А размерности m ´n на матрицу В размерности n ´k мы получаем матрицу С размерности m ´k . При этом элементы матрицы С вычисляются по следующим формулам:

Задача 1.8. Найти, если это возможно, произведение матриц AB и BA :

Решение. 1) Для того чтобы найти произведение AB , необходимо строки матрицы A умножить на столбцы матрицы B :

2) Произведение BA не существует, т. к. количество столбцов матрицы B не совпадает с количеством строк матрицы A .

Обратная матрица. Решение систем линейных уравнений матричным способом

Матрица A — 1 называется обратной к квадратной матрице А , если выполнено равенство:

где через I обозначается единичная матрица того же порядка, что и матрица А :

.

Для того чтобы квадратная матрица имела обратную необходимо и достаточно, чтобы ее определитель был отличен от нуля. Обратную матрицу находят по формуле:

, (1.13)

где A ij — алгебраические дополнения к элементам a ij матрицы А (заметим, что алгебраические дополнения к строкам матрицы А располагаются в обратной матрице в виде соответствующих столбцов).

Пример 1.9. Найти обратную матрицу A — 1 к матрице

.

Обратную матрицу найдем по формуле (1.13), которая для случая n = 3 имеет вид:

.

Найдем det A = | A | = 1 × 3 × 8 + 2 × 5 × 3 + 2 × 4 × 3 — 3 × 3 × 3 — 1 × 5 × 4 — 2 × 2 × 8 = 24 + 30 + 24 — 27 — 20 — 32 = — 1. Так как определитель исходной матрицы отличен от нуля, то обратная матрица существует.

1) Найдем алгебраические дополнения A ij :

Для удобства нахождения обратной матрицы, алгебраические дополнения к строкам исходной матрицы мы расположили в соответствующие столбцы.

Из полученных алгебраических дополнений составим новую матрицу и разделим ее на определитель det A . Таким образом, мы получим обратную матрицу:

Квадратные системы линейных уравнений с отличным от нуля главным определителем можно решать с помощью обратной матрицы. Для этого систему (1.5) записывают в матричном виде:

где

Умножая обе части равенства (1.14) слева на A — 1 , мы получим решение системы:

, откуда

Таким образом, для того чтобы найти решение квадратной системы, нужно найти обратную матрицу к основной матрице системы и умножить ее справа на матрицу-столбец свободных членов.

Задача 1.10. Решить систему линейных уравнений

с помощью обратной матрицы.

Решение. Запишем систему в матричном виде: ,

где — основная матрица системы, — столбец неизвестных и — столбец свободных членов. Так как главный определитель системы , то основная матрица системы А имеет обратную матрицу А -1 . Для нахождения обратной матрицы А -1 , вычислим алгебраические дополнения ко всем элементам матрицы А :

Из полученных чисел составим матрицу (причем алгебраические дополнения к строкам матрицы А запишем в соответствующие столбцы) и разделим ее на определитель D. Таким образом, мы нашли обратную матрицу:

Решение системы находим по формуле (1.15):

Таким образом,

Решение систем линейных уравнений методом обыкновенных жордановых исключений

Пусть дана произвольная (не обязательно квадратная) система линейных уравнений:

(1.16)

Требуется найти решение системы, т.е. такой набор переменных , который удовлетворяет всем равенствам системы (1. 16). В общем случае система (1.16) может иметь не только одно решение, но и бесчисленное множество решений. Она может так же вообще не иметь решений.

При решении подобных задач используется хорошо известный из школьного курса метод исключения неизвестных, который еще называется методом обыкновенных жордановых исключений. Суть данного метода заключается в том, что в одном из уравнений системы (1.16) одна из переменных выражается через другие переменные. Затем эта переменная подставляется в другие уравнения системы. В результате получается система, содержащая на одно уравнение и на одну переменную меньше, чем исходная система. Уравнение, из которого выражалась переменная, запоминается.

Этот процесс повторяется до тех пор, пока в системе не останется одно последнее уравнение. В процессе исключения неизвестных некоторые уравнения могут превратиться в верные тождества, например . Такие уравнения из системы исключаются, так как они выполняются при любых значениях переменных и, следовательно, не оказывают влияния на решение системы. Если в процессе исключения неизвестных хотя бы одно уравнение становится равенством, которое не может выполняться ни при каких значениях переменных (например ), то мы делаем вывод, что система не имеет решения.

Если в ходе решения противоречивых уравнений не возникло, то из последнего уравнения находится одна из оставшихся в нем переменных. Если в последнем уравнении осталась только одна переменная, то она выражается числом. Если в последнем уравнении остаются еще и другие переменные, то они считаются параметрами, и выраженная через них переменная будет функцией этих параметров. Затем совершается так называемый «обратный ход». Найденную переменную подставляют в последнее запомненное уравнение и находят вторую переменную. Затем две найденные переменные подставляют в предпоследнее запомненное уравнение и находят третью переменную, и так далее, вплоть до первого запомненного уравнения.

В результате мы получаем решение системы. Данное решение будет являться единственным, если найденные переменные будут числами. Если же первая найденная переменная, а затем и все остальные будут зависеть от параметров, то система будет иметь бесчисленное множество решений (каждому набору параметров соответствует новое решение). Формулы, позволяющие найти решение системы в зависимости от того или иного набора параметров, называются общим решением системы.

Пример 1.11.

x

После запоминания первого уравнения и приведения подобных членов во втором и третьем уравнении мы приходим к системе:

Выразим y из второго уравнения и подставим его в первое уравнение:

Запомним второе уравнение, а из первого найдем z :

Совершая обратный ход, последовательно найдем y и z . Для этого сначала подставим в последнее запомненное уравнение , откуда найдем y :

.

Затем подставим и в первое запомненное уравнение , откуда найдем x :

Задача 1.12. Решить систему линейных уравнений методом исключения неизвестных:

. (1.17)

Решение. Выразим из первого уравнения переменную x и подставим ее во второе и третье уравнения:

.

Запомним первое уравнение

В данной системе первое и второе уравнения противоречат друг другу. Действительно, выражая y , получим, что 14 = 17. Данное равенство не выполняется, ни при каких значениях переменных x , y , и z . Следовательно, система (1.17) несовместна, т.е. не имеет решения.

Читателям предлагаем самостоятельно проверить, что главный определитель исходной системы (1.17) равен нулю.

Рассмотрим систему, отличающуюся от системы (1.17) всего лишь одним свободным членом.

Задача 1.13. Решить систему линейных уравнений методом исключения неизвестных:

. (1.18)

Решение. Как и прежде, выразим из первого уравнения переменную x и подставим ее во второе и третье уравнения:

.

Запомним первое уравнение и приведем подобные члены во втором и третьем уравнении. Мы приходим к системе:

Выражая y из первого уравнения и подставляя его во второе уравнение , мы получим тождество 14 = 14, которое не влияет на решение системы, и, следовательно, его можно из системы исключить.

В последнем запомненном равенстве переменную z будем считать параметром. Полагаем . Тогда

Подставим y и z в первое запомненное равенство и найдем x :

.

Таким образом, система (1.18) имеет бесчисленное множество решений, причем любое решение можно найти по формулам (1.19), выбирая произвольное значение параметра t :

(1.19)
Так решениями системы, например, являются следующие наборы переменных (1; 2; 0), (2; 26; 14) и т. д. Формулы (1.19) выражают общее (любое) решение системы (1.18).

В том случае, когда исходная система (1.16) имеет достаточно большое количество уравнений и неизвестных, указанный метод обыкновенных жордановых исключений представляется громоздким. Однако это не так. Достаточно вывести алгоритм пересчета коэффициентов системы при одном шаге в общем виде и оформить решение задачи в виде специальных жордановых таблиц.

Пусть дана система линейных форм (уравнений):

, (1.20)
где x j — независимые (искомые) переменные, a ij — постоянные коэффициенты
(i = 1, 2,…, m ; j = 1, 2,…, n ). Правые части системы y i (i = 1, 2,…, m ) могут быть как переменными (зависимыми), так и константами. Требуется найти решений данной системы методом исключения неизвестных.

Рассмотрим следующую операцию, называемую в дальнейшем «одним шагом обыкновенных жордановых исключений». Из произвольного (r -го) равенства выразим произвольную переменную (x s ) и подставим во все остальные равенства. Разумеется, это возможно только в том случае, когда a rs ¹ 0. Коэффициент a rs называется разрешающим (иногда направляющим или главным) элементом.

Мы получим следующую систему:

. (1.21)

Из s -го равенства системы (1.21) мы впоследствии найдем переменную x s (после того, как будут найдены остальные переменные). S -я строка запоминается и в дальнейшем из системы исключается. Оставшаяся система будет содержать на одно уравнение и на одну независимую переменную меньше, чем исходная система.

Вычислим коэффициенты полученной системы (1.21) через коэффициенты исходной системы (1.20). Начнем с r -го уравнения, которое после выражения переменной x s через остальные переменные будет выглядеть следующим образом:

Таким образом, новые коэффициенты r -го уравнения вычисляются по следующим формулам:

(1.23)
Вычислим теперь новые коэффициенты b ij (i ¹ r ) произвольного уравнения. Для этого подставим выраженную в (1.22) переменную x s в i -е уравнение системы (1.20):

После приведения подобных членов, получим:

(1.24)
Из равенства (1.24) получим формулы, по которым вычисляются остальные коэффициенты системы (1. 21) (за исключением r -го уравнения):

(1.25)
Преобразование систем линейных уравнений методом обыкновенных жордановых исключений оформляется в виде таблиц (матриц). Эти таблицы получили название «жордановых».

Так, задаче (1.20) ставится в соответствие следующая жорданова таблица:

Таблица 1.1

x 1x 2x j x s x n
y 1 =a 11a 12a 1j a 1s a 1n
…………………………………………………………………..
y i =a i 1a i 2a ij a is a in
…………………………………………………………………..
y r =a r 1a r 2a rj a rsa rn
………………………………………………………………….
y n =a m 1a m 2a mj a ms a mn

Жорданова таблица 1.1 содержит левый заглавный столбец, в который записывают правые части системы (1.20) и верхнюю заглавную строку, в которую записывают независимые переменные.

Остальные элементы таблицы образуют основную матрицу коэффициентов системы (1.20). Если умножить матрицу А на матрицу , состоящую из элементов верхней заглавной строки, то получится матрица , состоящая из элементов левого заглавного столбца. То есть, по существу, жорданова таблица это матричная форма записи системы линейных уравнений: . Системе (1.21) при этом соответствует следующая жорданова таблица:

Таблица 1.2

x 1x 2x j y r x n
y 1 =b 11b 12b 1 j b 1 s b 1 n
…………………………………………………………………. .
y i = b i 1b i 2b ij b is b in
…………………………………………………………………..
x s = b r 1b r 2b rj b rs b rn
………………………………………………………………….
y n = b m 1b m 2b mj b ms b mn

Разрешающий элемент a rs мы будем выделять жирным шрифтом. Напомним, что для осуществления одного шага жордановых исключений разрешающий элемент должен быть отличен от нуля. Строку таблицы, содержащую разрешающий элемент, называют разрешающей строкой. Столбец, содержащий разрешающий элемент, называют разрешающим столбцом. При переходе от данной таблицы к следующей таблице одна переменная (x s ) из верней заглавной строки таблицы перемещается в левый заглавный столбец и, наоборот, один из свободных членов системы (y r ) из левого заглавного столбца таблицы перемещается в верхнюю заглавную строку.

Опишем алгоритм пересчета коэффициентов при переходе от жордановой таблицы (1.1) к таблице (1.2), вытекающий из формул (1.23) и (1.25).

1. Разрешающий элемент заменяется обратным числом:

2. Остальные элементы разрешающей строки делятся на разрешающий элемент и изменяют знак на противоположный:

3. Остальные элементы разрешающего столбца делятся на разрешающий элемент:

4. Элементы, не попавшие в разрешающую строку и разрешающий столбец, пересчитываются по формулам:

Последняя формула легко запоминается, если заметить, что элементы, составляющие дробь , находятся на пересечении i -ой и r -ой строк и j -го и s -го столбцов (разрешающей строки, разрешающего столбца и той строки и столбца, на пересечении которых находится пересчитываемый элемент). Точнее, при запоминании формулы можно использовать следующую диаграмму:

-21-26-13-37

Совершая первый шаг жордановых исключений, в качестве разрешающего элемента можно выбрать любой элемент таблицы 1.3, расположенный в столбцах x 1 ,…, x 5 (все указанные элементы не равны нулю). Не следует только выбирать разрешающий элемент в последнем столбце, т.к. требуется находить независимые переменные x 1 ,…, x 5 . Выбираем, например, коэффициент 1 при переменной x 3 в третьей строке таблицы 1.3 (разрешающий элемент показан жирным шрифтом). При переходе к таблице 1.4 переменная x 3 из верхней заглавной строки меняется местами с константой 0 левого заглавного столбца (третья строка). При этом переменная x 3 выражается через остальные переменные.

Строку x 3 (табл.1.4) можно, предварительно запомнив, исключить из таблицы 1.4. Из таблицы 1.4 исключается так же третий столбец с нулем в верхней заглавной строке. Дело в том, что независимо от коэффициентов данного столбца b i 3 все соответствующие ему слагаемые каждого уравнения 0·b i 3 системы будут равны нулю. Поэтому указанные коэффициенты можно не вычислять. Исключив одну переменную x 3 и запомнив одно из уравнений, мы приходим к системе, соответствующей таблице 1.4 (с вычеркнутой строкой x 3). Выбирая в таблице 1.4 в качестве разрешающего элемента b 14 = -5, переходим к таблице 1.5. В таблице 1.5 запоминаем первую строку и исключаем ее из таблицы вместе с четвертым столбцом (с нулем наверху).

Таблица 1.5 Таблица 1.6

Из последней таблицы 1.7 находим: x 1 = — 3 + 2x 5 .

Последовательно подставляя уже найденные переменные в запомненные строки, находим остальные переменные:

Таким образом, система имеет бесчисленное множество решений. Переменной x 5 , можно придавать произвольные значения. Данная переменная выступает в роли параметра x 5 = t. Мы доказали совместность системы и нашли ее общее решение:

x 1 = — 3 + 2t

x 2 = — 1 — 3t

x 3 = — 2 + 4t . (1.27)
x 4 = 4 + 5t

x 5 = t

Придавая параметру t различные значения, мы получим бесчисленное множество решений исходной системы. Так, например, решением системы является следующий набор переменных (- 3; — 1; — 2; 4; 0).

Решение систем линейных уравнений методом Крамера

Краткая теория


Рассмотрим частный случай системы линейных уравнений, когда число уравнений системы совпадает с числом неизвестных, подлежащих нахождению, то есть когда .

Пусть дана система  линейных уравнений с  неизвестными:

Определитель,  составленный из коэффициентов  называется определителем системы:

Рассмотрим случай, когда  – в этом случае система является определенной, то есть имеет единственное решение.

Напишем определитель, который будет отличаться от определителя  только -м столбцом, который заменен столбцом свободных членов. Обозначив этот определитель через , будем иметь:

Систему уравнений можно решить по следующим формулам, придавая индексу  значения :

Рассмотренный метод решения системы уравнений называется методом Крамера, а формулы – формулами Крамера.

Другие методы решения системы линейных алгебраических уравнений (СЛАУ):

Пример решения задачи


Задача

Решить систему линейных алгебраических уравнений (СЛАУ) методом Крамера. 

Решение

Если вам сейчас не требуется платная помощь с решением задач, контрольных работ и типовых расчетов, но может потребоваться в дальнейшем, то, чтобы не потерять контакт
вступайте в группу ВК
сохраните контакт WhatsApp (+79688494598)
сохраните контакт Телеграм (@helptask) .

Решим систему уравнений. Для этого вычислим определители, составленные по правилам Крамера:

Ответ:

Если вам сейчас не требуется платная помощь с решением задач, контрольных работ и типовых расчетов, но может потребоваться в дальнейшем, то, чтобы не потерять контакт
вступайте в группу ВК
сохраните контакт WhatsApp (+79688494598)
сохраните контакт Телеграм (@helptask) .

На цену сильно влияет срочность решения (от суток до нескольких часов). Онлайн-помощь на экзамене/зачете (срок решения 1,5 часа и меньше) осуществляется по предварительной записи.

Заявку можно оставить прямо в чате ВКонтакте, WhatsApp или Telegram, предварительно сообщив необходимые вам сроки решения и скинув условие задач.

Метод Крамера — решение систем линейных уравнений, примеры — смотреть онлайн видео урок бесплатно! Автор: alWEBra — Линейная алгебра


В этом видео рассказывается о методе Крамера — решение систем линейных уравнений, примеры. Это один из методов решения систем алгебраических уравнений, его еще называют методом определителей. Метод Крамера достаточно прост в использовании и позволяет быстро найти искомое решение, хотя и имеет ряд недостатков. Стоит отметить, что система уравнений называется линейной в том случае, если неизвестные между собой не перемножаются и не возводятся в степень. Именно для решения таких систем можно использовать метод Крамера. Решить систему — это значит найти все такие значения неизвестных, которые обращают каждое уравнение системы в тождество. Для решения системы линейных уравнений методом Крамера, сначала вычисляют определитель матрицы, составленных из коэффициентов при неизвестных. Если этот определитель не равен нулю, то система имеет единственное решение и метод крамера можно использовать, а если он равен нулю, то для решения данной системы уравнений необходимо использовать другой метод, например метод Гаусса. В этом уроке будет изучены все формулы метода Крамера и рассмотрен пример решения системы уравнения. Видео урок «Метод Крамера — решение систем линейных уравнений, примеры» вы можете смотреть онлайн абсолютно бесплатно в любое удобное время. Успехов!


  • Автор: alWEBra
  • Длительность: 4:19
  • Дата: 20.11.2013
  • Смотрели: 511
  • Рейтинг: 5.0/1



Если у Вас есть качественные видео уроки, которых нет на нашем сайте, то Вы можете добавить их в нашу коллекцию. Для этого Вам необходимо загрузить их на видеохостинг (например, YouTube) и добавить код видео в форму добавления уроков. Возможность добавлять свои материалы доступна только для зарегистрированных пользователей.

Практическая работа Решение систем линейных уравнений методом Крамера

Практическая работа № 18

Тема: Решение систем линейных уравнений методом Крамера.

Цель работы: решить систему линейных уравнений методом Крамера.

Студен должен:

знать:

уметь:

Теоретическое обоснование.

Метод Крамера. Применение для систем линейных уравнений.

Задана система N линейных алгебраических уравнений (СЛАУ) с  неизвестными, коэффициентами при которых являются элементы матрицы , а свободными членами — числа

Первый индекс возле коэффициентов указывает в каком уравнении находится коэффициент, а второй — при котором из неизвестным он находится.

Если определитель матрицы не равен нулю

то система линейных алгебраических уравнений имеет единственное решение. Решением системы линейных алгебраических уравнений называется такая упорядоченная совокупность  чисел , которая при превращает каждое из уравнений системы в правильную равенство. Если правые части всех уравнений системы равны нулю, то систему уравнений называют однородной. В случае, когда некоторые из них отличны от нуля – неоднородной Если система линейных алгебраических уравнений имеет хоть одно решение, то она называется совместной, в противном случае — несовместимой. Если решение системы единственное, то система линейных уравнений называется определенной. В случае, когда решение совместной системы не единственное, систему уравнений называют неопределенной. Две системы линейных уравнений называются эквивалентными (или равносильными), если все решения одной системы является решениями второй, и наоборот. Эквивалентны (или равносильны) системы получаем с помощью эквивалентных преобразований.

Эквивалентные преобразования СЛАУ

1) перестановка местами уравнений;

2) умножение (или деление) уравнений на отличное от нуля число;

3) добавление к некоторого уравнения другого уравнения, умноженного на произвольное, отличное от нуля число.

Решение СЛАУ можно найти разными способами, например , по формулам Крамера (метод Крамера)

Теорема Крамера.  Если определитель системы  линейных алгебраических уравнений с  неизвестными отличен от нуля то эта система имеет единственное решение, которое находится по формулам Крамера: — определители, образованные с  заменой -го столбца, столбцом из свободных членов.

Если , а хотя бы один из  отличен от нуля, то СЛАУ решений не имеет. Если же , то СЛАУ имеет множество решений. Главный определительопределяется разностью перемножения коэффициентов относительно одной диагонали и другой диагонали.

вычитаем

Пример 1

Дана система трех линейных уравнений с тремя неизвестными. Решить систему методом Крамера

Решение.

Найдем определитель матрицы коэффициентов при неизвестных

Так как , то заданная система уравнений совместная и имеет единственное решение. Вычислим определители:

По формулам Крамера находим неизвестные

Итак единственное решение системы.

Ход работы:

  1. Изучить теоретическое обоснование.

  2. Представить результаты практических заданий преподавателю.

  3. Оформить отчет.

  4. Ответить на контрольные вопросы.

Содержание отчета:

  1. Название и цели работы.

  2. Решение системы линейных уравнений по варианту.

  3. Вывод.

Практические задания:

Задание 1. Решить методом Крамера.

2.

4.

6.

7.

8.

9.

10.

11.

12.

13.

14.

15.

16.

17.

18.

19.

20.

21.

22.

23.

24.

Задание 2. Проверить решение СЛАУ на компьютере в электронных таблицах Excel.

Размещаем исходные коэффициенты в диапазоне B2:E4. Для вычисления главного определителя выбираем коэффициенты при x1, x2, x3 и размещаем их в диапазоне B6:D8. Исходную формулу размещаем в ячейку E7. Она выглядит так: =B6*C7*D8+D6*B7*C8+B8*C6*D7-(D6*C7*B8+D8*C6*B7+B6*D7*C8). Для первого определителя берем свободные члены СЛУ и размещаем в первую колонку и вычисляем определитель по аналогии. В ячейке h22 будет вычисляться первый корень по формуле: =E12/E7 и т. д.

Контрольные вопросы.

  1. Как выглядит система линейных уравнений?

  2. Как вычисляется главный определитель системы?

  3. Как вычисляются дополнительные определители системы?

  4. В каком случае СЛАУ имеет единственное решение?

  5. По каким формулам вычисляются корни СЛАУ по методу Крамера?

Литература.

  1. Овечкин, Г. В Компьютерное моделирование [Текст]: учебник/ Г. В. Овечкин.- М — Академия, 2015. – 224 с.

  2. Колдаев, В. Д Численные методы и программирование [Электронный ресурс]: ИНФРА-М., 2016. – 336 с. (ЭБС Znanium.com). Режим доступа: http://znanium.com/bookread2.php?book=546692

  3. Колдаев, В. Д Основы алгоритмизации и программирования [Электронный ресурс]: ИНФРА-М., 2016. – 416 с. (ЭБС Znanium.com). Режим доступа: http://znanium.com/bookread2.php?book=537513

Решение уравнений в Excel методом итераций Крамера и Гаусса

В программе Excel имеется обширный инструментарий для решения различных видов уравнений разными методами.

Рассмотрим на примерах некоторые варианты решений.

Решение уравнений методом подбора параметров Excel

Инструмент «Подбор параметра» применяется в ситуации, когда известен результат, но неизвестны аргументы. Excel подбирает значения до тех пор, пока вычисление не даст нужный итог.

Путь к команде: «Данные» — «Работа с данными» — «Анализ «что-если»» — «Подбор параметра».

Рассмотрим на примере решение квадратного уравнения х2 + 3х + 2 = 0. Порядок нахождения корня средствами Excel:

  1. Введем в ячейку В2 формулу для нахождения значения функции. В качестве аргумента применим ссылку на ячейку В1.
  2. Открываем меню инструмента «Подбор параметра». В графе «Установить в ячейку» — ссылка на ячейку В2, где находится формула. В поле «Значение» вводим 0. Это то значение, которое нужно получить. В графе «Изменяя значение ячейки» — В1. Здесь должен отобразиться отобранный параметр.
  3. После нажатия ОК отобразится результат подбора. Если нужно его сохранить, вновь нажимаем ОК. В противном случае – «Отмена».

Для подбора параметра программа использует циклический процесс. Чтобы изменить число итераций и погрешность, нужно зайти в параметры Excel. На вкладке «Формулы» установить предельное количество итераций, относительную погрешность. Поставить галочку «включить итеративные вычисления».



Как решить систему уравнений матричным методом в Excel

Дана система уравнений:

  1. Значения элементов введем в ячейки Excel в виде таблицы.
  2. Найдем обратную матрицу. Выделим диапазон, куда впоследствии будут помещены элементы матрицы (ориентируемся на количество строк и столбцов в исходной матрице). Открываем список функций (fx). В категории «Математические» находим МОБР. Аргумент – массив ячеек с элементами исходной матрицы.
  3. Нажимаем ОК – в левом верхнем углу диапазона появляется значение. Последовательно жмем кнопку F2 и сочетание клавиш Ctrl + Shift + Enter.
  4. Умножим обратную матрицу Ах-1х на матрицу В (именно в таком порядке следования множителей!). Выделяем диапазон, где впоследствии появятся элементы результирующей матрицы (ориентируемся на число строк и столбцов матрицы В). Открываем диалоговое окно математической функции МУМНОЖ. Первый диапазон – обратная матрица. Второй – матрица В.
  5. Закрываем окно с аргументами функции нажатием кнопки ОК. Последовательно нажимаем кнопку F2 и комбинацию Ctrl + Shift + Enter.

Получены корни уравнений.

Решение системы уравнений методом Крамера в Excel

Возьмем систему уравнений из предыдущего примера:

Для их решения методом Крамера вычислим определители матриц, полученных заменой одного столбца в матрице А на столбец-матрицу В.

Для расчета определителей используем функцию МОПРЕД. Аргумент – диапазон с соответствующей матрицей.

Рассчитаем также определитель матрицы А (массив – диапазон матрицы А).

Определитель системы больше 0 – решение можно найти по формуле Крамера (Dx / |A|).

Для расчета Х1: =U2/$U$1, где U2 – D1. Для расчета Х2: =U3/$U$1. И т.д. Получим корни уравнений:

Решение систем уравнений методом Гаусса в Excel

Для примера возьмем простейшую систему уравнений:

3а + 2в – 5с = -1
2а – в – 3с = 13
а + 2в – с = 9

Коэффициенты запишем в матрицу А. Свободные члены – в матрицу В.

Для наглядности свободные члены выделим заливкой. Если в первой ячейке матрицы А оказался 0, нужно поменять местами строки, чтобы здесь оказалось отличное от 0 значение.

  1. Приведем все коэффициенты при а к 0. Кроме первого уравнения. Скопируем значения в первой строке двух матриц в ячейки В6:Е6. В ячейку В7 введем формулу: =B3:Е3-$B$2:$Е$2*(B3/$B$2). Выделим диапазон В7:Е7. Нажмем F2 и сочетание клавиш Ctrl + Shift + Enter. Мы отняли от второй строки первую, умноженную на отношение первых элементов второго и первого уравнения.
  2. Копируем введенную формулу на 8 и 9 строки. Так мы избавились от коэффициентов перед а. Сохранили только первое уравнение.
  3. Приведем к 0 коэффициенты перед в в третьем и четвертом уравнении. Копируем строки 6 и 7 (только значения). Переносим их ниже, в строки 10 и 11. Эти данные должны остаться неизменными. В ячейку В12 вводим формулу массива.
  4. Прямую прогонку по методу Гаусса сделали. В обратном порядке начнем прогонять с последней строки полученной матрицы. Все элементы данной строки нужно разделить на коэффициент при с. Введем в строку формулу массива: {=B12:E12/D12}.
  5. В строке 15: отнимем от второй строки третью, умноженную на коэффициент при с второй строки ({=(B11:E11-B16:E16*D11)/C11}). В строке 14: от первой строки отнимаем вторую и третью, умноженные на соответствующие коэффициенты ({=(B10:E10-B15:E15*C10-B16:E16*D10)/B10}). В последнем столбце новой матрицы получаем корни уравнения.

Примеры решения уравнений методом итераций в Excel

Вычисления в книге должны быть настроены следующим образом:

Делается это на вкладке «Формулы» в «Параметрах Excel». Найдем корень уравнения х – х3 + 1 = 0 (а = 1, b = 2) методом итерации с применением циклических ссылок. Формула:

Хn+1 = Xn– F (Xn) / M, n = 0, 1, 2, … .

M – максимальное значение производной по модулю. Чтобы найти М, произведем вычисления:

f’ (1) = -2 * f’ (2) = -11.

Полученное значение меньше 0. Поэтому функция будет с противоположным знаком: f (х) = -х + х3 – 1. М = 11.

В ячейку А3 введем значение: а = 1. Точность – три знака после запятой. Для расчета текущего значения х в соседнюю ячейку (В3) введем формулу: =ЕСЛИ(B3=0;A3;B3-(-B3+СТЕПЕНЬ(B3;3)-1/11)).

В ячейке С3 проконтролируем значение f (x): с помощью формулы =B3-СТЕПЕНЬ(B3;3)+1.

Корень уравнения – 1,179. Введем в ячейку А3 значение 2. Получим тот же результат:

Скачать решения уравнений в Excel

Корень на заданном промежутке один.

Калькулятор правила Крамерса

Как найти неизвестные переменные по правилу Крамерса?

Понятие матричного определителя появилось в Германии и Японии практически в одно и то же время. Секи впервые написал об этом в 1683 году в своем «Метод решения разрозненных задач ». Секи разработал шаблон для определителей для $ 2 \ times 2 $, $ 3 \ times 3 $, Матрицы $ 4 \ times 4 $ и $ 5 \ times 5 $ и использовали их для решения уравнений. В том же году Г. Лейбниц написал о методе решения система уравнений.Этот метод хорошо известен как Правило Крамера . Определитель квадратной матрицы $ A $ — это уникальное действительное число, которое является атрибутом матрицы $ A $. Определитель матрицы $ A $ обозначается $ det (A) $ или $ | A | $.

Правило Крамера — это формула для решения системы линейных уравнений. Он выводит решение в терминах определителей матрицы и матриц, полученных из нее, путем замены одного столбца вектором-столбцом правых частей уравнений. Он назван Габриэлем Крамером (17041752), и правило для произвольного числа неизвестных опубликовано в статье [Cramer, G.{th} $ столбец основной матрицы вектором правых частей уравнений и вычисляем его определитель, $ D_x $.

  • Чтобы найти решение $ x $ системы линейных уравнений по правилу Крамера, разделите определитель $ D_x $ на главный определитель $ D $;
  • Повторите предыдущий шаг для каждой переменной;
  • Если главный определитель равен нулю, система линейных уравнений либо несовместна, либо имеет бесконечно много решений.
    Правило Крамера в двух переменных : Рассмотрим систему уравнений:
    $$ \ begin {align} & a_1x + b_1y = \ color {синий} {c_1} \\ & a_2x + b_2y = \ color {синий} {c_2} \ end {align} $$ Главный определитель $$ D = \ left | \ begin {array} {cc} a_1 и b_1 \\ a_2 и b_2 \\ \ end {массив} \ right | $$ и два других детерминанта $$ D_x = \ left | \ begin {array} {cc} \ color {blue} {c_1} & b_1 \\ \ color {синий} {c_2} & b_2 \\ \ end {массив} \ right | \ quad \ mbox {и} \ quad D_y = \ left | \ begin {array} {cc} a_1 & \ color {синий} {c_1} \\ a_2 & \ color {синий} {c_2} \\ \ end {массив} \ right | $$ С помощью определителей можно найти $ x $ и $ y $ по правилу Крамера как
    . $$ x = \ frac {D_x} {D} = \ frac {\ left | \ begin {array} {cc} \ color {blue} {c_1} & b_1 \\ \ color {синий} {c_2} & b_2 \\ \ end {массив} \ right |} {\ left | \ begin {array} {cc} a_1 и b_1 \\ a_2 и b_2 \\ \ end {массив} \ right |} \ quad \ mbox {and} \ quad y = \ frac {D_y} {D} = \ frac {\ left | \ begin {array} {cc} a_1 & \ color {синий} {c_1} \\ a_2 & \ color {синий} {c_2} \\ \ end {массив} \ right |} {\ left | \ begin {array} {cc} a_1 и b_1 \\ a_2 и b_2 \\ \ end {массив} \ right |} $$ Если каждый определитель равен нулю, система согласована, а уравнения зависимы.У системы бесконечно много решений. Если $ D = 0 $ и $ D_x $ или $ D_y $ не равны нулю, система несовместима и не имеет решения.
    Правило Крамера в трех переменных : Рассмотрим систему уравнений: $$ \ begin {align} & a_1x + b_1y + c_1z = \ color {синий} {d_1} \\ & a_2x + b_2y + c_2z = \ цвет {синий} {d_2} \\ & a_3x + b_3y + c_3z = \ color {синий} {d_3} \\ \ end {align} $$ Главный определитель $$ D = \ left | \ begin {array} {ccc} a_1 и b_1 и c_1 \\ a_2 и b_2 и c_2 \\ a_3 & b_3 & c_3 \\ \ end {массив} \ right | $$ а остальные три детерминанта $$ D_x = \ left | \ begin {array} {ccc} \ color {blue} {d_1} & b_1 & c_1 \\ \ цвет {синий} {d_2} & b_2 & c_2 \\ \ color {blue} {d_3} & b_3 & c_3 \\ \ end {массив} \ right | \ quad D_y = \ left | \ begin {array} {ccc} a_1 & \ color {синий} {d_1} & c_1 \\ a_2 & \ цвет {синий} {d_2} & c_2 \\ a_3 & \ color {синий} {d_3} & c_3 \\ \ end {массив} \ right | \ quad \ mbox {and} \ quad D_z = \ left | \ begin {array} {ccc} a_1 & b_1 & \ color {синий} {d_1} \\ a_2 & b_2 & \ color {синий} {d_2} \\ a_3 & b_3 & \ color {синий} {d_3} \\ \ end {массив} \ right | $$ Решение системы трех уравнений есть $$ x = \ frac {D_x} {D}, \ quad y = \ frac {D_y} {D}, \ quad \ mbox {и} \ quad z = \ frac {D_z} {D} $$ Например, решим систему линейных уравнений: $$ \ begin {align} & 3x + 4y + 5z = 10 \\ & 5x + 6y + 7z = 12 \\ & 4x + 5y + 0z = 15 \\ \ end {align} $$ Сначала вычисляем главный определитель: $$ \ begin {align} D & = \ left | \ begin {array} {ccc} 3 и 4 и 5 \\ 5 и 6 и 7 \\ 4 и 5 и 0 \\ \ end {массив} \ right | \ & = \ left | \ begin {array} {ccc | cc} 3 и 4 и 5 и 3 и 4 \\ 5 и 6 и 7 и 5 и 6 \\ 4 и 5 и 0 и 4 и 5 \\ \ end {массив} \верно.= 3 \ cdot6 \ cdot0 + 4 \ cdot7 \ cdot4 + 5 \ cdot5 \ cdot 5-5 \ cdot6 \ cdot4-3 \ cdot7 \ cdot5-4 \ cdot6 \ cdot0 = 12 \ end {align} $$ По аналогии, $$ D_x = \ left | \ begin {array} {ccc} \ color {blue} {10} & 4 и 5 \\ \ color {blue} {12} & 6 и 7 \\ \ color {blue} {15} & 5 & 0 \\ \ end {массив} \ right | = -80, \ quad D_y = \ left | \ begin {array} {ccc} 3 & \ color {синий} {10} & 5 \\ 5 & ​​\ color {blue} {12} & 7 \\ 4 & \ color {blue} {15} & 0 \\ \ end {массив} \ right | = 100, \ quad D_z = \ left | \ begin {array} {ccc} 3 и 4 & \ color {синий} {10} \\ 5 и 6 & \ color {синий} {12} \\ 4 и 5 & \ color {синий} {15} \\ \ end {массив} \ right | = -8 $$

    3×3 Решатель Системы Уравнений

    О правиле Крамера

    Этот калькулятор использует правило Крамера для решения систем трех уравнений с тремя неизвестные.Правило Крамера можно сформулировать следующим образом:

    Учитывая систему:

    $$ \ begin {выровнено} a_1x + b_1y + c_1z = d_1 \\ а_2x + b_2y + c_2z = d_2 \\ a_3x + b_3y + c_3z = d_3 \ end {выровнен} $$

    с

    $$ D = \ left | \ begin {array} {ccc} a_1 и b_1 и c_1 \\ a_2 и b_2 и c_2 \\ a_3 & b_3 & c_3 \\ \ end {array} \ right | \ ne 0 $$ $$ D_x = \ left | \ begin {array} {ccc} d_1 & b_1 & c_1 \\ d_2 & b_2 & c_2 \\ d_3 & b_3 & c_3 \\ \ end {array} \ right | $$ $$ D_y = \ left | \ begin {array} {ccc} a_1 и d_1 и c_1 \\ a_2 & d_2 & c_2 \\ a_3 & d_3 & c_3 \\ \ end {array} \ right | $$ $$ D_z = \ left | \ begin {array} {ccc} a_1 и b_1 и d_1 \\ а_2 и b_2 и d_2 \\ a_3 & b_3 & d_3 \\ \ end {array} \ right | $$

    , то решение этой системы:

    $$ x = \ frac {D_x} {D} $$ $$ y = \ frac {D_y} {D} $$ $$ z = \ frac {D_z} {D} $$

    Пример: Решите систему уравнений, используя правило Крамера

    $$ \ begin {выровнено} 4x + 5y -2z = & -14 \\ 7x — ~ y + 2z = & 42 \\ 3x + ~ y + 4z = & 28 \\ \ end {выровнен} $$

    Решение: Сначала мы вычисляем $ D, ~ D_x, ~ D_y $ и $ D_z $.

    $$ \ begin {выровнено} & D ~~ = \ left | \ begin {массив} {ccc} {\ color {blue} {4}} & {\ color {red} {~ 5}} & {\ color {green} {- 2}} \\ {\ color {blue} {7}} & {\ color {red} {- 1}} & {\ color {green} {~ 2}} \\ {\ color {blue} {3}} & {\ color {red} {~ 1}} & {\ color {green} {~ 4}} \ end {array} \ right | = -16 + 30-14-6-8-140 = -154 \\ & D_x = \ left | \ begin {массив} {ccc} -14 & {\ color {red} {~ 5}} & {\ color {green} {- 2}} \\ ~ 42 & {\ color {red} {- 1}} & {\ color {green} {~ 2}} \\ ~ 28 & {\ color {red} {1}} & {\ color {green} {~ 4}} \ end {array} \ right | = 56 + 280 — 84 — 56 + 28 — 840 = -616 \\ & D_y = \ left | \ begin {массив} {ccc} {\ color {blue} {4}} & -14 & {\ color {green} {- 2}} \\ {\ color {blue} {7}} & ~ 42 & {\ color {green} {~ 2}} \\ {\ color {blue} {3}} & ~ 28 & {\ color {green} {~ 4}} \ end {array} \ right | = 672 — 84 — 392 + 252 — 224 + 392 = 616 \\ & D_Z = \ left | \ begin {array} {ccc} {\ color {blue} {4}} & {\ color {red} {~ 5}} & -14 \\ {\ color {blue} {7}} & {\ color {red} {- 1}} & ~ 42 \\ {\ color {blue} {3}} & {\ color {red} {~ 1}} & ~ 28 \ end {array} \ right | = -112 + 630 — 98 — 42 — 168 — 980 = -770 \\ \ end {выровнен} $$

    Следовательно,

    $$ \ begin {выровнено} & x = \ frac {D_x} {D} = \ frac {-616} {- 154} = 4 \\ & y = \ frac {D_y} {D} = \ frac {616} {- 154} = -4 \\ & z = \ frac {D_z} {D} = \ frac {-770} {- 154} = 5 \ end {выровнен} $$

    Примечание: Вы можете проверить решение с помощью вышеуказанного калькулятора

    Правило Крамера

    Крамера Правило


    Учитывая систему линейных уравнения, правило Крамера — удобный способ решить только одну из переменных без необходимости решать всю систему уравнений.Они обычно не обучать правилу Крамера таким образом, но это должно быть суть Правило: вместо решения всей системы уравнений можно использовать Крамеру нужно решить только одну-единственную переменную.

    Воспользуемся следующим система уравнений:

    У нас левая часть системы с переменными («матрица коэффициентов») а в правой части — значения ответов.Позволять D — определитель матрицы коэффициентов указанной выше системы, и пусть D x быть определителем, образованным заменой столбца x значения со значениями столбца ответа:

    система из
    уравнений

    коэффициент
    определитель матрицы

    ответ
    столбец

    D x : определитель коэффициента
    со столбцом ответов
    значений в
    x — столбец

    2 x + 1 и + 1 z = 3
    1 x 1 y 1 z = 0
    1 x + 2 и + 1 z = 0

    Аналогично D y и D z тогда будет: Авторское право Элизабет Стапель 2004-2011 Все права защищены

    Оценка каждого детерминанта (с использованием метода, описанного здесь), получаем:

    Правило Крамера гласит, что x = D x D , и = D y D , и z = D z D .То есть:

      x = 3 / 3 = 1, y = 6 / 3 = 2 , и z = 9 / 3 = 3

    Вот и все, что нужно для Cramer’s Правило.Чтобы найти любую желаемую переменную (назовите ее «» или «бета»), просто оцените определяющее частное D Д . (Пожалуйста не просите меня объяснять, почему это работает. Просто поверьте мне, что детерминанты может творить много видов магии.)

    • Учитывая следующее Система уравнений, найдите значение z .
    • Чтобы решить только для z , Сначала я нахожу определитель коэффициента.

      Затем я формирую D z заменив третий столбец значений столбцом ответов:


      Затем я составляю частное и упростить:

    Смысл правила Крамера в том, что вам не нужно решать всю систему, чтобы получить одно значение тебе нужно.Это сэкономило мне много времени на некоторых тестах по физике. я забыть, над чем мы работали (я думаю, что-то с проводами и токами), но правило Крамера было намного быстрее, чем любой другой метод решения (и Видит Бог, мне нужно было дополнительное время). Не позволяйте всем нижним индексам и прочему запутать вас; Правило действительно довольно простое. Вы просто выбираете переменную вы хотите найти, замените столбец значений этой переменной в определитель коэффициента со значениями столбца ответа, оцените, что определитель и разделите на определитель коэффициента.Это все там к нему.

    Почти.

    Что делать, если определитель коэффициента ноль? Нельзя делить на ноль, что это значит? Я не могу пойти в технические детали здесь, но « D = 0 «означает, что система уравнений не имеет единственного решения. Система может быть несовместимой (решение отсутствует вообще) или зависимое (бесконечное решение, которое может быть выражается как параметрическое решение, например «( a , a + 3, a 4) «).С точки зрения правила Крамера: « D = 0 «означает, что вам придется использовать другой метод (например, матрицу строковые операции) в решить систему. Если D = 0, вы не можете использовать Cramer’s Правило.

    Верх | Вернуться к индексу

    Цитируйте эту статью как:

    Стапель, Елизавета.«Правило Крамера». Purplemath . Доступна с
    https://www.purplemath.com/modules/cramers.htm . Доступ [Дата] [Месяц] 2016 г.

    Правило Крамера с вопросами и решениями

    .
    \ ( \оставил\{ \ begin {array} {lcl} a_1 b_ 2 x + b_1 b_2 y & = & c_1 b_2 \\ -a_2 b_1 x — b_2 b_1 y & = & — c_2 b_1 \ end {массив} \верно. \)

    Сложите левую и правую части приведенных выше уравнений и упростите, чтобы получить уравнение с одной переменной.
    \ ( a_1 b_2 x — a_2 b_1 x = c_1 b_2 — c_2 b_1 \)

    Выносим множитель x на левой стороне
    \ ( x (a_1 b_2 — a_2 b_1) = b_2 c_1 — b_1 c_2 \)

    Решите указанное выше уравнение относительно x
    \ ( x = \ dfrac {c_1 b_2 — c_2 b_1} {a_1 b_2 — a_2 b_1} \)

    Мы можем использовать аналогичные шаги, чтобы исключить x и решить относительно y, чтобы получить.
    \ ( y = \ dfrac {a_1 c_2 — a_2 c_1} {a_1 b_2 — a_2 b_1} \)

    Решение данной системы линейных уравнений 2 на 2 дается правилами Крамера следующим образом.
    \ [x = \ dfrac {D_x} {D}, y = \ dfrac {D_y} {D} \]
    Использование

    Общая система линейных уравнений 3 на 3 может быть записана следующим образом: \ [ \оставил\{ \ begin {array} {lcl} a_1 x + b_1 y + c_1 z = & \ color {красный} {d_1} & (1) \\ a_2 x + b_2 y + c_2 = & \ color {красный} {d_2} & (2) \\ a_3 x + b_3 y + c_3 = & \ color {красный} {d_3} & (2) \\ \ end {массив} \верно.\]

    Для системы линейных уравнений 3 на 3 правило Крамера дает следующее решение
    \ [x = \ dfrac {D_x} {D}, y = \ dfrac {D_y} {D}, z = \ dfrac {D_z} {D} \]
    , где \ (D, D_x, D_y \ text {и} D_z \) — определители матриц 3 на 3, определяемые

    \ (D = \ begin {vmatrix} a_1 & b_1 & c_1 \\ a_2 & b_2 & c_2 \\ a_3 & b_3 & c_3 \ end {vmatrix} \)

    \ (D_x = \ begin {vmatrix} \ color {red} {d_1} & b_1 & c_1 \\ \ color {red} {d_2} & b_2 & c_2 \\ \ color {red} {d_3} & b_3 & c_3 \ end {vmatrix} \), \ (D_y = \ begin {vmatrix} a_1 & \ color {red} {d_1} & c_1 \\ a_2 & \ color {red} {d_2} & c_2 \\ a_3 & \ color {red} {d_3} & c_3 \ end {vmatrix} \) , \ (D_z = \ begin {vmatrix} a_1 & b_1 & \ color {red} {d_1} \\ a_2 & b_2 & \ color {red} {d_2} \\ a_3 & b_3 & \ color {red} {d_3} \ end {vmatrix} \)

  • Часть 1
    а) Рассчитайте определители
    \ (D = \ begin {vmatrix} 5 & -2/3 \\ -1 & 1/2 \ end {vmatrix} = (5) (1/2) — (- 2/3) (- 1) = 11/6 \)

    \ (D_x = \ begin {vmatrix} 1/3 & -2/3 \\ -1/2 & 1/2 \ end {vmatrix} = (1/3) (1/2) — (- 2/3 ) (- 1/2) = -1/6 \)

    \ (D_y = \ begin {vmatrix} 5 & 1/3 \\ -1 & -1/2 \ end {vmatrix} = (5) (- 1/2) — (1/3) (- 1) = -13/6 \)

    Решение дается правилом Крамера следующим образом.
    \ (x = \ dfrac {D_x} {D} = — \ dfrac {1} {11} \)

    \ (y = \ dfrac {D_y} {D} = — \ dfrac {13} {11} \)

    б) Определители даются по формуле
    \ (D = \ begin {vmatrix} 0.1 & -0,3 \\ -0,2 & 1,3 \ end {vmatrix} = (0,1) (1,3) — (-0,3) (- 0,2) = 0,07 \)

    \ (D_x = \ begin {vmatrix} 1.1 & -0.3 \\ -1.5 & 1.3 \ end {vmatrix} = (1.1) (1.3) — (-0.3) (- 1.5) = 0.98 \)

    \ (D_y = \ begin {vmatrix} 0.1 & 1.1 \\ -0.2 & -1.5 \ end {vmatrix} = (0.1) (- 1.5) — (1.1) (- 0.2) = 0.07 \)

    Решение дается
    \ (x = \ dfrac {D_x} {D} = \ dfrac {0,98} {0,07} = 14 \)

    \ (y = \ dfrac {D_y} {D} = \ dfrac {0,07} {0,07} = 1 \)

    в) Определители 3 на 3 \ (D, D_x, D_y \ text {и} D_z \) вычисляются с использованием определителей 2 на 2 следующим образом
    \ (D = \ begin {vmatrix} -3 & 5 & -1 \\ 1/5 & -5 & — 3/5 \\ -4 & 4/5 & -1 \ end {vmatrix} = -3 \ cdot \ begin {vmatrix} -5 & — \ frac {3} {5} \\ \ frac {4} {5} & — 1 \ end {vmatrix} -5 \ cdot \ begin {vmatrix} \ frac {1} { 5} & — \ frac {3} {5} \\ -4 & -1 \ end {vmatrix} -1 \ cdot \ begin {vmatrix} \ frac {1} {5} & — 5 \\ -4 & \ frac { 4} {5} \ end {vmatrix} = 82/5 \)

    \ (D_x = \ begin {vmatrix} 1/2 & 5 & -1 \\ 0 & -5 & — 3/5 \\ -7 & 4/5 & -1 \ end {vmatrix} = \ frac { 1} {2} \ begin {vmatrix} -5 & — \ frac {3} {5} \\ \ frac {4} {5} & — 1 \ end {vmatrix} -5 \ cdot \ begin {vmatrix} 0 & — \ frac {3} {5} \\ -7 & -1 \ end {vmatrix} -1 \ cdot \ begin {vmatrix} 0 & -5 \\ -7 & \ frac {4} {5} \ end {vmatrix} = 2937 / 50 \)

    \ (D_y = \ begin {vmatrix} -3 & 1/2 & -1 \\ 1/5 & 0 & — 3/5 \\ -4 & -7 & -1 \ end {vmatrix} = -3 \ cdot \ begin {vmatrix} 0 & — \ frac {3} {5} \\ -7 & -1 \ end {vmatrix} — \ frac {1} {2} \ begin {vmatrix} \ frac {1} {5} & — \ frac {3} {5} \\ -4 & -1 \ end {vmatrix} -1 \ cdot \ begin {vmatrix} \ frac {1} {5} & 0 \\ -4 & -7 \ end {vmatrix} = 153/10 \)

    \ (D_z = \ begin {vmatrix} -3 & 5 & 1/2 \\ 1/5 & -5 & 0 \\ -4 & 4/5 & — 7 \ end {vmatrix} = -3 \ cdot \ begin {vmatrix} -5 & 0 \\ \ frac {4} {5} & — 7 \ end {vmatrix} -5 \ cdot \ begin {vmatrix} \ frac {1} {5} & 0 \\ -4 & -7 \ end {vmatrix} + \ frac {1} {2} \ begin {vmatrix} \ frac {1} {5} & — 5 \\ -4 & \ frac {4} {5} \ end {vmatrix} = -2698 / 25 \)

    Решение дается
    \ (x = \ dfrac {D_x} {D} = \ dfrac {2937/50} {82/5} = \ dfrac {2937} {820} \)

    \ (y = \ dfrac {D_y} {D} = \ dfrac {153/10} {82/5} = \ dfrac {153} {164} \)

    \ (z = \ dfrac {D_z} {D} = \ dfrac {-2698/25} {82/5} = — \ dfrac {1349} {205} \)

  • Часть 2
    a) Определители даются по формуле
    \ (D = \ begin {vmatrix} 5 & -k \\ -2 & 2k \ end {vmatrix} = (5) (2k) — (- k) (- 2) = 8 k \)

    \ (D_x = \ begin {vmatrix} 6 & -k \\ -3 & 2k \ end {vmatrix} = (6) (2k) — (- k) (- 3) = 9 k \)

    \ (D_y = \ begin {vmatrix} 5 & 6 \\ -2 & -3 \ end {vmatrix} = (5) (- 3) — (6) (- 2) = — 3 \)

    Решение системы
    \ (x = \ dfrac {D_x} {D} = \ dfrac {9k} {8k} = \ dfrac {9} {8} \) , \ (Y = \ dfrac {D_y} {D} = \ dfrac {-3} {8k} = — \ dfrac {3} {8k} \)

    б) Определители даются по формуле
    \ (D = \ begin {vmatrix} 2 & — 3 \\ 1 & 2 \ end {vmatrix} = (2) (2) — (-3) (1) = 7 \)

    \ (D_x = \ begin {vmatrix} k & — 3 \\ -2 k & 2 \ end {vmatrix} = (k) (2) — (-3) (- 2k) = -4k \)

    \ (D_y = \ begin {vmatrix} 2 & k \\ 1 & -2k \ end {vmatrix} = (2) (- 2k) — (k) (1) = -5k \)
    Решение системы
    \ (x = \ dfrac {D_x} {D} = \ dfrac {-4k} {7} = — \ dfrac {4} {7} k \) , \ (Y = \ dfrac {D_y} {D} = \ dfrac {-5k} {7} = — \ dfrac {5} {7} k \)

  • Часть 3
    a) Используя правило Крамера, решения даются по формуле
    \ (х = 6 q + 25 p \) \ (y = -2 q — 10 p \)

    б)
    1) Если мы заменим параметры p и q в системе уравнений в a) на 2 и 2 соответственно, мы получим систему уравнений
    \ ( \оставил\{ \ begin {array} {lcl} — х — 3 у & = & 10 \\ — 2 х — 5 лет & = & — 4 \ end {массив} \верно.\)
    , которую мы должны решить. Но система в а) решена для всех значений p и q. Следовательно, чтобы решить систему в b) 1), мы заменяем p и q их значениями (2 и 2) в решениях, полученных в a), что дает.
    \ (x = 6 q + 25 p = 6 (2) + 25 (2) = 62 \) \ (y = -2 q — 10 p = -2 (2) — 10 (2) = -24 \)

    2) Для этой системы p = 1/2 и q = 1; следовательно, решение
    \ (x = 6 q + 25 p = 6 (1) + 25 (1/2) = 37/2 \) \ (y = -2 q — 10 p = -2 (1) — 10 (1/2) ) = — 7 \)

    3) Для этой системы p = 10 и q = -3; следовательно, решение
    \ (x = 6 q + 25 p = 6 (-3) + 25 (10) = 232 \) \ (y = -2 q — 10 p = -2 (-3) — 10 (10) = — 94 \)

  • Часть 4
    Определители, используемые в правиле Крамера, даются

    \ (D = \ begin {vmatrix} 5 & -6 & 6 \\ \: \: 9 & -p & -1 \\ \: \: — 2 & -6 & 0 \ end {vmatrix} = 5 \ cdot \ begin {vmatrix} -p & -1 \\ -6 & 0 \ end {vmatrix} — \ left (-6 \ right) \ begin {vmatrix} 9 & -1 \\ -2 & 0 \ end {vmatrix} +6 \ cdot \ begin {vmatrix} 9 & -p \ \ -2 & -6 \ end {vmatrix} = -12p-366 \)

    \ (D_x = \ begin {vmatrix} -14 & -6 & 6 \\ \: \: 22 & -p & -1 \\ \: \: \: — 4 & -6 & 0 \ end {vmatrix} = -14 \ cdot \ begin {vmatrix} -p & -1 \\ -6 & 0 \ end {vmatrix} — \ left (-6 \ right) \ begin {vmatrix} 22 & -1 \\ -4 & 0 \ end {vmatrix} +6 \ cdot \ begin {vmatrix } 22 & -p \\ -4 & -6 \ end {vmatrix} = -24p-732 \)

    \ (D_y = \ begin {vmatrix} 5 & -14 & 6 \\ \: \: \: 9 & 22 & -1 \\ \: \: \: — 2 & -4 & 0 \ end {vmatrix} = = 5 \ cdot \ begin { vmatrix} 22 & -1 \\ -4 & 0 \ end {vmatrix} — \ left (-14 \ right) \ begin {vmatrix} 9 & -1 \\ -2 & 0 \ end {vmatrix} +6 \ cdot \ begin {vmatrix} 9 & 22 \\ -2 & -4 \ end {vmatrix} = 0 \)

    \ (D_z = \ begin {vmatrix} 5 & -6 & -14 \\ \: \: \: 9 & -p & 22 \\ \: \: \: — 2 & -6 & -4 \ end {vmatrix} = = 5 \ cdot \ begin {vmatrix} -p & 22 \\ -6 & -4 \ end {vmatrix} — \ left (-6 \ right) \ begin {vmatrix} 9 & 22 \\ -2 & -4 \ end {vmatrix} -14 \ cdot \ begin {vmatrix} 9 & -p \\ -2 & -6 \ end {vmatrix} = 48p + 1464 \)

    Решения даются правилом Крамера следующим образом.

    \ (x = \ dfrac {D_x} {D} = \ dfrac {-24p-732} {- 12p-366} = 2 \)

    \ (y = \ dfrac {D_y} {D} = \ dfrac {0} {- 12p-366} = 0 \)

    \ (z = \ dfrac {D_z} {D} = \ dfrac {48p + 1464} {- 12p-366} = -4 \)

  • Калькулятор определителя

    symbolab — Mjuo su

    Матричное сложение, умножение, инверсия, вычисление определителя и ранга, транспонирование, приведение к диагонали, треугольной форме, возведение в степень, решение систем линейных уравнений с шагом решения С помощью этого калькулятора вы можете: найти определитель матрицы

    Этот матричный калькулятор вычисляет определитель, обратные преобразования, ранг, характеристический многочлен, собственные значения и собственные векторы.Он разлагает матрицу с помощью LU и разложения Холецкого. Добро пожаловать в MathPortal. Владелец этого веб-сайта математик Милош

    Правило Крамера может использоваться для систем линейных уравнений, в которых количество уравнений равно количеству неизвестных переменных, а определитель матрицы коэффициентов не равен нулю (в противном случае система уравнений не имеет единственного решения — либо у нее нет

    Math 21b: Информационный бюллетень о детерминантах Определитель любой квадратной матрицы A является скаляром, обозначаемым det (A).[Неквадратные матрицы не имеют определителей.] Определитель квадратной матрицы A определяет, является ли A обратимым: если det (A) = 0, то A не обратима

    Калькулятор правила

    Крамера решает систему трех линейных уравнений с действительными коэффициентами. Это онлайн-алгебраический инструмент, запрограммированный для определения упорядоченной тройки как решения системы трех линейных уравнений. Используя этот калькулятор, мы сможем понять

    Детерминантная обратная матрица Матрица мощности Матрица транспонирования Матрица умножения Матрица Сложение / вычитание Матрица Калькулятор мощности Здесь вы можете возвести матрицу в степень с комплексными числами онлайн бесплатно.Вы можете изучить умножение отдельно от того, что было.

    Этот калькулятор обратной матрицы может помочь вам найти обратную квадратную матрицу независимо от ее типа (2 × 2, 3 × 3 или 4 × 4). Вы можете узнать больше сразу после инструмента. Как работает этот калькулятор обратной матрицы? Этот калькулятор обратной матрицы может помочь вам при попытке найти

    Введите квадратную матрицу, и этот калькулятор покажет вам шаг за шагом, как разложить матрицу на множители в форме: A = LU, где L — нижний треугольник, а U — верхний треугольник.Шаг 1. Для начала выберите количество строк и столбцов в матрице и нажмите кнопку «Создать матрицу».

    matrix-attribute-polynomial-Calculator es image / svg + xml Связанные сообщения блога Symbolab Матрица версий Matrix Symbolab с числами, упорядоченными по строкам и столбцам, чрезвычайно полезна в большинстве научных областей. Есть

    Symbolab: búsqueda de ecuaciones y solucionador matemático — resuelve problemas de álgebra, trigonometría y cálculo paso a paso Этот веб-сайт использует файлы cookie, чтобы обеспечить вам лучший опыт.Используя этот сайт, вы соглашаетесь с нашей Политикой в ​​отношении файлов cookie.

    matrix-gauss-jordan-Calculator es image / svg + xml Связанные сообщения блога Symbolab Матрица версий Matrix Symbolab, матрица с числами, упорядоченными по строкам и столбцам, чрезвычайно полезна в большинстве научных областей. Есть

    Этот онлайн-калькулятор поможет вам решить систему линейных уравнений с помощью правила Крамера. Воспользовавшись этим онлайн-калькулятором, вы получите подробное пошаговое решение вашей задачи, которое поможет понять алгоритм решения системы линейных

    .

    Матричный калькулятор Решение систем линейных уравнений Детерминантный калькулятор Калькулятор собственных значений Примеры решений Википедия: Матрицы Скрыть объявления Показать объявления Нахождение собственных значений и собственных векторов Этот калькулятор позволяет находить собственные значения и собственные векторы.

    Вот простой онлайн-линейно независимый или зависимый калькулятор для поиска линейной зависимости и взаимозависимости между векторами. Сначала введите размер столбца и размер строки, а затем введите значения, чтобы знать шаги исключения матрицы.

    Связанные сообщения блога Symbolab Расширенные математические решения — векторный калькулятор, расширенные векторы В прошлом блоге мы рассмотрели некоторые из более простых векторных тем. На этой неделе мы рассмотрим более тяжелые модели

    .

    Обратите внимание, что страницы WIMS создаются в интерактивном режиме; это не обычные файлы HTML.Они должны использоваться в интерактивном режиме ОНЛАЙН. Вам бесполезно собирать их через программу-робот. Описание: вычисляет определитель, обратный, собственные векторы,

    Определители матриц 3 × 3 называются определителями третьего порядка. Один из методов оценки детерминант третьего порядка называется разложением по минорам. Младший элемент — это определитель, формируемый при удалении строки и столбца, содержащих этот элемент.

    Чтобы улучшить 『Матричный калькулятор продуктов』, заполните, пожалуйста, анкету.Мужской или женский ? Мужской Женский Возраст Моложе 20 лет 20 лет Уровень 30 лет Уровень 40 лет Уровень 50 лет Уровень 60 лет и старше Род занятий Начальная школа

    Ввод данных калькулятора матриц

    Введите свою матрицу в ячейки ниже A 」или「 B 」. Или вы можете ввести в большую область вывода и нажать 「на A」 или 「на B」 (калькулятор постарается изо всех сил интерпретировать ваши данные). СОХРАНЕНИЕ Чтобы сохранить матрицу, нажмите 「из A」 или 「из B」, а затем скопируйте

    Определитель комплексных чисел Определитель комплексных чисел Определитель = декартово: Полярное: ПРИМЕЧАНИЕ: введите число в одной из следующих форм: 2 + i5, 2 + 5i, 2 + j5, 2 + 5j

    Найдите определитель заданной матрицы 3 × 3.Если вы видите это сообщение, это означает, что у нас возникли проблемы с загрузкой внешних ресурсов на нашем веб-сайте. Если вы находитесь за веб-фильтром, убедитесь, что домены * .kastatic.org и * .kasandbox.org разблокированы.

    Категория: определитель Вычислить определитель для матрицы 2 × 2, 3 × 3, 4 × 4, 5 × 5 с помощью кофакторов — шаг за шагом — с использованием упрощенной линейной алгебры TiNspire Существует несколько способов вычисления определителей квадратных матриц в зависимости от их размеры.

    Калькулятор для вычисления матрицы сопряжения с этапами вычисления.Онлайн-калькулятор для вычисления сопряженной матрицы NxN Калькулятор вычисляет сопряженную матрицу заданной матрицы NxN и использует результат для вычисления также обратной матрицы.

    Калькулятор обратного преобразования матрицы 2 × 2: Калькулятор, приведенный в этом разделе, может использоваться для поиска обратного значения матрицы 2 × 2. Он не только дает обратную матрицу 2 × 2, но также дает определитель и сопряженную матрицу 2 × 2, которую вы вводите. МАТРИЦА 2 × 2 ОБРАТНАЯ

    Определитель матрицы — это скалярное свойство этой матрицы.Определитель — это специальное число, которое определено только для квадратных матриц (множественное число для матрицы). Квадратная матрица имеет одинаковое количество строк и столбцов. Определитель используется, чтобы узнать, может ли матрица быть

    онлайн-калькулятор разложения LU матрицы, найдите верхнюю и нижнюю треугольную матрицу путем факторизации. Калькулятор определителя матрицы. Калькулятор умножения матрицы. Калькулятор сложения / вычитания матрицы. Калькулятор транспонирования матрицы. Генератор случайных матриц

    .

    Калькулятор дивергенции и изгиба

    Матричные решатели (калькуляторы) с шагами. Вы можете использовать дроби, например 1/3.

    Подождите, пока в поле 1,1 электронной таблицы не появится надпись «Готово!».

    Предположим, что строки 「i」 и 「j」 идентичны. Затем, если мы поменяем эти строки, мы получим ту же матрицу и, следовательно, тот же определитель. Однако обмен строками меняет знак определителя. Это требует того, что может быть правдой, только если. Пример # 5

    Линейная независимость и зависимость v. 1.25 ШАБЛОН ЗАДАЧИ Учитывая набор S = {v 1, v 2,, v n} векторов в векторном пространстве V, определите, является ли S линейно независимым или линейно зависимым.УКАЗАТЬ КОЛИЧЕСТВО ВЕКТОРОВ И ОТПРАВИТЬ.

    Дискриминантная функция Мэддри предполагает, что пациенты с алкогольным гепатитом могут иметь плохой прогноз и получать пользу от приема стероидов. Расчеты необходимо перепроверить, и их нельзя использовать отдельно для руководства по уходу за пациентом, и они не должны заменять

    .

    Калькулятор решает систему четырех уравнений с четырьмя неизвестными. Добро пожаловать в MathPortal. Владелец этого веб-сайта — математик Милош Петрович. Я разработал этот сайт и написал все уроки, формулы и калькуляторы.

    · PDF 檔案

    Свойства детерминантов Детерминанты Теперь на полпути мы оставляем прямоугольные матрицы и сосредотачиваемся на квадратных. Наши следующие большие темы — детерминанты и собственные значения. Определитель — это число, связанное с любой квадратной матрицей; мы

    EigenSpace 3 × 3 Matrix Calculator Простой онлайн-калькулятор EigenSpace для поиска пространства, порожденного собственными векторами квадратной матрицы. Это объединение нулевого вектора и набора всех собственных векторов, соответствующих собственному значению.Собственное значение — это множитель, который

    Воспользуйтесь этим точным и бесплатным калькулятором Якоби, чтобы вычислить любые проблемы и найти любую информацию, которая может вам понадобиться. Этот супер-полезный калькулятор — продукт компании Wolfram Alpha, одной из ведущих на сегодняшний день прорывных технологий и баз знаний. Вольфрам альфа проложенный

    24/9/2018 · Это приложение работает с функциями одной и двух переменных. Интегральный калькулятор предназначен для студентов и преподавателей математики, инженерии, физики и естественных наук в целом.Приложение может выполнять следующие математические операции: — Символьные примитивные, производные и интегральные вычисления. — Решатель системных уравнений и матричные операции (жорданова форма, собственные значения, определитель и т. Д.). — График 2Д и

    Чтобы улучшить этот 『Калькулятор разложения по сингулярным значениям, пожалуйста, заполните анкету. Мужской или женский ? Мужчина Женский Возраст Моложе 20 лет 20 лет Уровень 30 лет Уровень 40 лет Уровень 50 лет Уровень 60 лет и старше Род занятий

    Онлайн калькулятор.Этот онлайн-калькулятор решает систему линейных уравнений с использованием правила Крамера и показывает подробные этапы решения. Правило Крамера. Этот онлайн-калькулятор решает систему линейных уравнений с использованием правила Крамера и показывает подробные этапы решения

    Введите матрицу, и этот калькулятор покажет вам шаг за шагом, как вычислить основу для пространства столбцов этой матрицы. Шаг 1. Для начала выберите количество строк и столбцов в матрице и нажмите кнопку «Создать матрицу». Количество рядов:

    Линейные системы двух переменных и правило Крамера Напомним, что матрица — это прямоугольный массив чисел, состоящий из строк и столбцов.Мы классифицируем матрицы по количеству строк n и количеству столбцов m. Например, матрица 3 × 4, читаем «Матрица 3 на 4

    .

    Матричный калькулятор Матричный калькулятор вычисляет все важные аспекты матрицы: определитель, инверсию, след, норму. Просто войдите в матрицу, выберите то, что вы хотите вычислить, нажмите кнопку, и пусть калькулятор матриц сделает всю работу за вас!

    , как использовать определитель, чтобы найти площадь параллелограмма, вычислить определитель площади параллелограмма, образованного двумя двумерными векторами, 9 класс Связанные темы: Дополнительные уроки для 9 класса Задания по математике Примеры, решения, видео, рабочие листы

    Онлайн-калькулятор Численные методы Домашняя страница Онлайн-калькулятор Домашняя страница Метод деления пополам Онлайн-калькулятор Метод ложного положения Онлайн-калькулятор Метод Ньютона-Рафсона Онлайн-калькулятор Метод секанса Онлайн-калькулятор Итерационный (фиксированная точка

    ) Калькулятор радиуса сходимости

    — это бесплатный онлайн-инструмент, который отображает точку схождения для заданного ряда.Онлайн-калькулятор радиуса сходимости BYJU ускоряет вычисления и отображает точку схождения за доли секунды. Как к

    $ \ begingroup $ @JeanMarie Как мы можем показать, что объем задается абсолютным значением определителя? Тот же ответ здесь: мы должны проверить аксиомы объема, а именно умножение на скаляры, суммы векторов и нормализацию.

    1/1/2017 · Как использовать калькулятор (991ES Plus) — Поиск сопряженного и обратного 3 × 3. Автор сэр Ахсан Абдулла Патель. По вопросам обращайтесь: 0336-8151-868 Рекомендуемое качество 720p или 1080p.

    作者: сэр Ахсан Абдулла Патель

    Цель использования Калькулятор объема тетраэдра Помогает вычислить объем объекта, поверхность которого представляет собой замкнутую треугольную сетку. В настоящее время я использую другую формулу, чем вы, и мне интересно узнать, будет ли она быстрее по мере увеличения количества треугольников. Ошибка

    9.8: Решение систем по правилу Крамера

    Мы узнали, как решать системы уравнений с двумя переменными и тремя переменными, а также с помощью нескольких методов: подстановки, сложения, исключения Гаусса, использования обратной матрицы и построения графиков.Некоторые из этих методов применять проще, чем другие, и они более подходят в определенных ситуациях. В этом разделе мы изучим еще две стратегии решения систем уравнений.

    Вычисление определителя матрицы 2 × 2

    Определитель — это действительное число, которое может быть очень полезно в математике, потому что у него есть несколько приложений, таких как вычисление площади, объема и других величин. Здесь мы будем использовать определители, чтобы определить, является ли матрица обратимой, используя элементы квадратной матрицы, чтобы определить, существует ли решение системы уравнений.Однако, возможно, одним из наиболее интересных приложений является их использование в криптографии. Защищенные сигналы или сообщения иногда отправляются в виде матрицы. Расшифровать данные можно только с помощью обратимой матрицы и определителя. В наших целях мы ориентируемся на определитель как на показатель обратимости матрицы. Вычисление определителя матрицы требует следования определенным шаблонам, описанным в этом разделе.

    НАЙТИ ОПРЕДЕЛЕНИЕ МАТРИЦЫ 2 × 2

    Определитель матрицы 2 × 2, учитывая

    \ (A = \ begin {bmatrix} a & b \\ c & d \ end {bmatrix} \)

    определяется как

    Обратите внимание на изменение обозначений.Есть несколько способов указать определитель, включая \ (\ det (A) \) и замену скобок в матрице прямыми линиями, \ (| A | \).

    Пример \ (\ PageIndex {1} \): поиск определителя матрицы \ (2 × 2 \)

    Найдите определитель заданной матрицы.

    \ (A = \ begin {bmatrix} 5 & 2 \\ — 6 & 3 \ end {bmatrix} \)

    Решение

    \ [\ begin {align *} \ det (A) & = \ begin {vmatrix} 5 & 2 \\ — 6 & 3 \ end {vmatrix} \\ & = 5 (3) — (- 6) (2) \\ & = 27 \ end {align *} \]

    Использование правила Крамера для решения системы двух уравнений с двумя переменными

    Теперь мы представим последний метод решения систем уравнений, использующий определители.Этот метод, известный как правило Крамера , восходит к середине 18 века и назван в честь своего новатора, швейцарского математика Габриэля Крамера (1704-1752), который представил его в 1750 году в . Courbes algébriques . Правило Крамера — это жизнеспособный и эффективный метод поиска решений систем с произвольным числом неизвестных при условии, что у нас есть такое же количество уравнений, что и неизвестных.

    Правило Крамера даст нам единственное решение системы уравнений, если оно существует.Однако, если система не имеет решения или бесконечное количество решений, это будет обозначено нулевым определителем. Чтобы выяснить, является ли система непоследовательной или зависимой, необходимо использовать другой метод, например исключение.

    Чтобы понять правило Крамера, давайте внимательно рассмотрим, как мы решаем системы линейных уравнений с использованием основных операций со строками. Рассмотрим систему двух уравнений с двумя переменными.

    \ [\ begin {align} a_1x + b_1y & = c_1 (1) \ label {eq1} \\ a_2x + b_2y & = c_2 (2) \ label {eq2} \\ \ end {align} \]

    Мы исключаем одну переменную, используя операции со строками, и решаем для другой.Скажите, что мы хотим найти \ (x \). Если уравнение \ ref {eq2} умножается на коэффициент, противоположный коэффициенту \ (y \) в уравнении \ ref {eq1}, уравнение \ ref {eq1} умножается на коэффициент при \ (y \) в уравнении \ ref {eq2}, и мы добавляем два уравнения, переменная \ (y \) будет удалена.

    \ [\ begin {align *} & b_2a_1x + b_2b_1y = b_2c_1 & \ text {Multiply} R_1 \ text {by} b_2 \\ — & \ underline {b_1a_2x − b_1b_2y = −b_1c_2} & \ text {Multiply} R_2 \ text {by} −b_1 \\ & b_2a_1x − b_1a_2x = b_2c_1 − b_1c_2 \ end {align *} \]

    Теперь решите относительно \ (x \).

    \ [\ begin {align *} b_2a_1x − b_1a_2x & = b_2c_1 − b_1c_2 \\ x (b_2a_1 − b_1a_2) & = b_2c_1 − b_1c_2 \\ x & = \ dfrac {b_2c_1 − b_1c_2} {b_2a_1 − b_1a_2} = \ dfrac {\ begin {bmatrix} c_1 & b_1 \\ c_2 & b_2 \ end {bmatrix}} {\ begin {bmatrix} a_1 & b_1 \\ a_2 & b_2 \ end {bmatrix}} \ end {align *} \]

    Аналогично, чтобы найти \ (y \), мы исключим \ (x \).

    \ [\ begin {align *} & a_2a_1x + a_2b_1y = a_2c_1 & \ text {Multiply} R_1 \ text {by} a_2 \\ — & \ underline {a_1a_2x − a_1b_2y = −a_1c_2} & \ text {Multiply} R_2 \ текст {by} −a_1 \\ & a_2b_1y − a_1b_2y = a_2c_1 − a_1c_2 \ end {align *} \]

    Решение относительно \ (y \) дает

    \ [\ begin {align *} a_2b_1y − a_1b_2y & = a_2c_1 − a_1c_2 \\ y (a_2b_1 − a_1b_2) & = a_2c_1 − a_1c_2 \\ y & = \ dfrac {a_2c_1 − a_1c_2} {a_2b_1 − a_1b_2} = \ dfrac {a_1c_2 − a_2c_1} {a_1b_2 − a_2b_1} = \ dfrac {\ begin {bmatrix} a_1 & c_1 \\ a_2 & c_2 \ end {bmatrix}} {\ begin {bmatrix} a_1 & b_1 \\ a_2 & b_2 \ end {bmatrix}} \ end {align * } \]

    Обратите внимание, что знаменатель для \ (x \) и \ (y \) является определителем матрицы коэффициентов.

    Мы можем использовать эти формулы для решения относительно \ (x \) и \ (y \), но правило Крамера также вводит новые обозначения:

    • \ (D \): определитель матрицы коэффициентов
    • \ (D_x \): определитель числителя в решении \ (x \)

      \ [x = \ dfrac {D_x} {D} \]

    • \ (D_y \): определитель числителя в решении \ (y \)

      \ [y = \ dfrac {D_y} {D} \]

    Ключ к правилу Крамера заключается в замене интересующего столбца переменных столбцом констант и вычислении детерминантов.Тогда мы можем выразить \ (x \) и \ (y \) как частное двух определителей.

    ПРАВИЛО КРЕМЕРА ДЛЯ СИСТЕМ \ (2 × 2 \)

    Правило Крамера — это метод, который использует детерминанты для решения систем уравнений, которые имеют то же количество уравнений, что и переменные.

    Рассмотрим систему двух линейных уравнений с двумя переменными.

    \ [\ begin {align *} a_1x + b_1y & = c_1 \\ a_2x + b_2y & = c_2 \ end {align *} \]

    Решение, использующее правило Крамера, дается как

    \ [\ begin {align} x & = \ dfrac {D_x} {D} = \ dfrac {\ begin {bmatrix} c_1 & b_1 \\ c_2 & b_2 \ end {bmatrix}} {\ begin {bmatrix} a_1 & b_1 \\ a_2 & b_2 \ end { bmatrix}} \; , D \ neq 0 \\ y & = \ dfrac {D_y} {D} = \ dfrac {\ begin {bmatrix} a_1 & c_1 \\ a_2 & c_2 \ end {bmatrix}} {\ begin {bmatrix} a_1 & b_1 \\ a_2 & b_2 \ end {bmatrix }} \; , D \ neq 0 \ end {align} \]

    Если мы решаем для \ (x \), столбец \ (x \) заменяется постоянным столбцом.Если мы решаем для \ (y \), столбец \ (y \) заменяется постоянным столбцом.

    Пример \ (\ PageIndex {2} \): использование правила Крамера для решения системы \ (2 × 2 \)

    Решите следующую систему \ (2 × 2 \), используя правило Крамера.

    \ [\ begin {align *} 12x + 3y & = 15 \\ 2x-3y & = 13 \ end {align *} \]

    Решение

    Решите относительно \ (x \).

    \ [\ begin {align *} x & = \ dfrac {D_x} {D} \\ & = \ dfrac {\ begin {bmatrix} 15 & 3 \\ 13 & -3 \ end {bmatrix}} {\ begin {bmatrix} 12 & 3 \\ 2 & -3 \ end {bmatrix}} \\ & = \ dfrac {-45-39} {- 36-6} \\ & = \ dfrac {-84} {- 42} \\ & = 2 \ end {align *} \]

    Решите относительно \ (y \).

    \ [\ begin {align *} y & = \ dfrac {D_y} {D} \\ & = \ dfrac {\ begin {bmatrix} 12 & 15 \\ 2 & 13 \ end {bmatrix}} {\ begin {bmatrix} 12 & 3 \\ 2 & -3 \ end {bmatrix}} \\ & = \ dfrac {156-30} {- 36-6} \\ & = — \ dfrac {126} {42} \\ & = -3 \ end {align * } \]

    Решение: \ ((2, −3) \).

    Упражнение \ (\ PageIndex {1} \)

    Используйте правило Крамера для решения системы уравнений \ (2 × 2 \).

    \ [\ begin {align *} x + 2y & = -11 \\ -2x + y & = -13 \ end {align *} \]

    Ответ

    \ ((3, −7) \)

    Вычисление определителя матрицы 3 × 3

    Найти определитель матрицы 2 × 2 несложно, но найти определитель матрицы 3 × 3 сложнее.Один из способов — увеличить матрицу 3 × 3 повторением первых двух столбцов, получив матрицу 3 × 5. Затем мы вычисляем сумму произведений записей на по каждой из трех диагоналей (от верхнего левого угла к нижнему правому) и вычитаем произведение записей на по каждой из трех диагоналей (нижний левый верхний правый). Это легче понять с помощью наглядного пособия и примера.

    Найдите определитель матрицы 3 × 3.

    \ (A = \ begin {bmatrix} a_1 & b_1 & c_1 \\ a_2 & b_2 & c_2 \\ a_3 & b_3 & c_3 \ end {bmatrix} \)

    1. Дополнение \ (A \) первыми двумя столбцами.

      \ (\ det (A) = \ left | \ begin {array} {ccc | cc} a_1 & b_1 & c_1 & a_1 & b_1 \\ a_2 & b_2 & c_2 & a_2 & b_2 \\ a_3 & b_3 & c_3 & a_3 & b_3 \ end {array} \ right | \)

    2. С верхнего левого угла в нижний правый: умножение значений по первой диагонали. Добавьте результат к произведению записей по второй диагонали. Добавьте этот результат к произведению записей по третьей диагонали.
    3. От левого нижнего угла до правого верхнего: вычтите произведение входов вверх по первой диагонали.Из этого результата вычтите произведение входов вверх по второй диагонали. Из этого результата вычтите произведение входов до третьей диагонали.

    Алгебра выглядит следующим образом:

    \ (| A | = a_1b_2c_3 + b_1c_2a_3 + c_1a_2b_3 − a_3b_2c_1 − b_3c_2a_1 − c_3a_2b_1 \)

    Пример \ (\ PageIndex {3} \): поиск определителя матрицы 3 × 3

    Найдите определитель матрицы \ (3 × 3 \) при

    \ (A = \ begin {bmatrix} 0 & 2 & 1 \\ 3 & −1 & 1 \\ 4 & 0 & 1 \ end {bmatrix} \)

    Решение

    Дополните матрицу первыми двумя столбцами, а затем следуйте формуле.Таким образом,

    \ [\ begin {align *} | А | & = \ left | \ begin {array} {ccc | cc} 0 & 2 & 1 & 0 & 2 \\ 3 & -1 & 1 & 3 & -1 \\ 4 & 0 & 1 & 4 & 0 \ end {array} \ right | \\ & = 0 (−1) (1) +2 (1) (4) +1 (3) (0) −4 (−1) (1) −0 (1) (0) −1 (3) (2) \\ & = 0 + 8 + 0 + 4−0−6 \\ & = 6 \ end {align *} \]

    Упражнение \ (\ PageIndex {2} \)

    Найдите определитель матрицы 3 × 3.

    \ (\ det (A) = \ begin {vmatrix} 1 & −3 & 7 \\ 1 & 1 & 1 \\ 1 & −2 & 3 \ end {vmatrix} \)

    Ответ

    \ (- 10 \)

    Q&A: Можем ли мы использовать тот же метод, чтобы найти определитель большей матрицы?

    Нет, этот метод работает только для матриц 2 × 2 и 3 × 3.Для больших матриц лучше всего использовать графическую утилиту или компьютерное программное обеспечение.

    Использование правила Крамера для решения системы трех уравнений с тремя переменными

    Теперь, когда мы можем найти определитель матрицы \ (3 × 3 \), мы можем применить правило Крамера для решения системы трех уравнений с тремя переменными. Правило Крамера простое и соответствует шаблону, соответствующему правилу Крамера для матриц \ (2 × 2 \). Однако по мере увеличения порядка матрицы до \ (3 × 3 \) требуется гораздо больше вычислений.

    Когда мы вычисляем детерминант, равный нулю, правило Крамера не дает никаких указаний на то, что у системы нет решения или бесконечное количество решений. Чтобы выяснить это, мы должны выполнить устранение в системе.

    Рассмотрим систему уравнений \ (3 × 3 \).

    \ [\ begin {align} a_1x + b_1y + c_1z & = \ color {blue} d_1 \\ a_2x + b_2y + c_2z & = \ color {blue} d_2 \\ a_3x + b_3y + c_3z & = \ color {blue} d_3 \\ \ end {align} \]

    \ (x = \ dfrac {D_x} {D} \), \ (y = \ dfrac {D_y} {D} \), \ (z = \ dfrac {D_z} {D} \), \ (D ≠ 0 \)

    где

    \ [D = \ begin {vmatrix} a_1 & b_1 & c_1 \\ a_2 & b_2 & c_2 \\ a_3 & b_3 & c_3 \ end {vmatrix} \; , \; D_x = \ begin {vmatrix} \ color {blue} d_1 & b_1 & c_1 \\ \ color {blue} d_2 & b_2 & c_2 \\ \ color {blue} d_3 & b_3 & c_3 \ end {vmatrix} \; , \; D_y = \ begin {vmatrix} a_1 & \ color {blue} d_1 & c_1 \\ a_2 & \ color {blue} d_2 & c_2 \\ a_3 & \ color {blue} d_3 & c_3 \ end {vmatrix} \; , \; D_z = \ begin {vmatrix} a_1 & b_1 & \ color {blue} d_1 \\ a_2 & b_2 & \ color {blue} d_2 \\ a_3 & b_3 & \ color {blue} d_3 \ end {vmatrix} \]

    Если мы пишем определитель \ (D_x \), мы заменяем столбец \ (x \) постоянным столбцом.Если мы пишем определитель \ (D_y \), мы заменяем столбец y на столбец констант. Если мы пишем определитель \ (D_z \), мы заменяем столбец \ (z \) постоянным столбцом. Всегда проверяйте ответ.

    Пример \ (\ PageIndex {4} \): решение системы \ (3 × 3 \) с использованием правила Крамера

    Найдите решение данной системы \ (3 × 3 \), используя правило Крамера.

    \ [\ begin {align *} x + y-z & = 6 \\ 3x-2y + z & = -5 \\ x + 3y-2z & = 14 \ end {align *} \]

    Решение

    Используйте правило Крамера.

    \ (D = \ begin {vmatrix} 1 & 1 & −1 \\ 3 & −2 & 1 \\ 1 & 3 & −2 \ end {vmatrix} \), \ (D_x = \ begin {vmatrix} 6 & 1 & −1 \\ — 5 & −2 & 1 \ \ 14 & 3 & −2 \ end {vmatrix} \), \ (D_y = \ begin {vmatrix} 1 & 6 & −1 \\ 3 & −5 & 1 \\ 1 & 14 & −2 \ end {vmatrix} \), \ (D_z = \ begin {vmatrix } 1 & 1 & 6 \\ 3 & −2 & −5 \\ ​​1 & 3 & 14 \ end {vmatrix} \)

    Затем,

    \ [\ begin {align *} x & = \ dfrac {D_x} {D} & = \ dfrac {-3} {- 3} & = 1 \\ y & = \ dfrac {D_y} {D} & = \ dfrac {-9} {- 3} & = 3 \\ z & = \ dfrac {D_z} {D} & = \ dfrac {6} {- 3} & = -2 \\ \ end {align *} \]

    Решение: \ ((1,3, −2) \).

    Упражнение \ (\ PageIndex {3} \)

    Используйте правило Крамера, чтобы решить матрицу \ (3 × 3 \).

    \ [\ begin {align *} x-3y + 7z & = 13 \\ x + y + z & = 1 \\ x-2y + 3z & = 4 \ end {align *} \]

    Ответ

    \ (\ left (−2, \ dfrac {3} {5}, \ dfrac {12} {5} \ right) \)

    Пример \ (\ PageIndex {5A} \): использование правила Крамера для решения несовместимой системы

    Решите систему уравнений, используя правило Крамера.

    \ [\ begin {align} 3x-2y & = 4 \ label {eq3} \\ 6x-4y & = 0 \ label {eq4} \ end {align} \]

    Решение

    Начнем с нахождения определителей \ (D \), \ (D_x \) и \ (D_y \).

    \ (D = \ begin {vmatrix} 3 & −2 \\ 6 & −4 \ end {vmatrix} = 3 (−4) −6 (−2) = 0 \)

    Мы знаем, что нулевой определитель означает, что либо система не имеет решения, либо имеет бесконечное количество решений. Чтобы узнать, какой из них, мы используем процесс исключения. Наша цель — исключить одну из переменных.

    1. Умножьте уравнение \ ref {eq3} на \ (- 2 \).
    2. Добавьте результат в уравнение \ ref {eq4}.

    \ [\ begin {align *} & −6x + 4y = −8 \\ & \; \; \; \ underline {6x − 4y = 0} \\ & \; \; \; \; \; \ ; \; \; \; \; 0 = −8 \ end {align *} \]

    Получаем уравнение \ (0 = −8 \), которое неверно. Следовательно, у системы нет решения. График системы показывает две параллельные линии. См. Рисунок \ (\ PageIndex {1} \).

    Рисунок \ (\ PageIndex {1} \)

    Пример \ (\ PageIndex {5B} \): использование правила Крамера для решения зависимой системы

    Решите систему с бесконечным количеством решений.

    \ [\ begin {align} x-2y + 3z & = 0 \ label {eq5} \\ 3x + y-2z & = 0 \ label {eq6} \\ 2x-4y + 6z & = 0 \ label {eq7} \ end {align} \]

    Решение

    Давайте сначала найдем определитель. Создайте матрицу, дополненную первыми двумя столбцами.

    \ (\ left | \ begin {array} {ccc | cc} 1 & −2 & 3 & 1 & -2 \\ 3 & 1 & −2 & 3 & 1 \\ 2 & −4 & 6 & 2 & -4 \ end {array} \ right | \)

    Затем,

    \ (1 (1) (6) + (- 2) (- 2) (2) +3 (3) (- 4) −2 (1) (3) — (- 4) (- 2) (1 ) −6 (3) (- 2) = 0 \)

    Поскольку определитель равен нулю, решения либо нет, либо существует бесконечное количество решений.Чтобы выяснить это, нам нужно провести отбор.

    1. Умножьте уравнение \ ref {eq5} на \ (- 2 \) и добавьте результат к уравнению \ ref {eq7}:

    \ [\ begin {align *} & −2x + 4y − 6x = 0 \\ & \; \; \ underline {2x − 4y + 6z = 0} \\ & \; \; \; \; \; \ ; \; \; \; \; \; \; \; \; \; \; 0 = 0 \ end {align *} \]

    2. Получение ответа \ (0 = 0 \), утверждение, которое всегда верно, означает, что система имеет бесконечное количество решений. Изобразив систему, мы можем увидеть, что две плоскости одинаковы, и обе они пересекают третью плоскость по прямой.См. Рисунок \ (\ PageIndex {2} \).

    Рисунок \ (\ PageIndex {2} \)

    Понимание свойств детерминантов

    Есть много свойств определителей. Здесь перечислены некоторые свойства, которые могут быть полезны при вычислении определителя матрицы.

    СВОЙСТВА ДЕТЕРМИНАНТОВ

    1. Если матрица имеет верхнюю треугольную форму, определитель равен произведению элементов по главной диагонали.
    2. Когда две строки меняются местами, определитель меняет знак.{−1} \) — величина, обратная определителю матрицы \ (A \).
    3. Если какая-либо строка или столбец умножается на константу, определитель умножается на тот же коэффициент.

    Пример \ (\ PageIndex {6} \): иллюстрация свойств детерминантов

    Проиллюстрируйте каждое из свойств определителей.

    Решение

    Свойство 1 утверждает, что если матрица имеет верхнюю треугольную форму, определитель является произведением элементов по главной диагонали.

    \ (A = \ begin {bmatrix} 1 & 2 & 3 \\ 0 & 2 & 1 \\ 0 & 0 & −1 \ end {bmatrix} \)

    Дополните \ (A \) первыми двумя столбцами.

    \ (A = \ left [\ begin {array} {ccc | cc} 1 & 2 & 3 & 1 & 2 \\ 0 & 2 & 1 & 0 & 2 \\ 0 & 0 & −1 & 0 & 0 \ end {array} \ right] \)

    Затем

    \ [\ begin {align *} \ det (A) & = 1 (2) (- 1) +2 (1) (0) +3 (0) (0) -0 (2) (3) -0 (1) (1) +1 (0) (2) \\ & = -2 \ end {align *} \]

    Свойство 2 утверждает, что перестановка строк меняет знак.Учитывая

    \ [\ begin {align *} B & = \ begin {bmatrix} 4 & -3 \\ — 1 & 5 \ end {bmatrix} \\ \ det (B) & = (4) (5) — (- 1) (- 3) \\ & = 20-3 \\ & = 17 \ end {align *} \]

    Свойство 3 утверждает, что если две строки или два столбца идентичны, определитель равен нулю.

    \ [\ begin {align *} A & = \ left [\ begin {array} {ccc | cc} 1 & 2 & 2 & 1 & 2 \\ 2 & 2 & 2 & 2 & 2 \\ — 1 & 2 & 2 & -1 & 2 \ end {array} \ right] \\ \ det (A) & = 1 (2) (2) +2 (2) (- 1) +2 (2) (2) +1 (2) (2) -2 (2) (1) -2 (2) (2) \ \ & = 4-4 + 8 + 4-4-8 \\ & = 0 \ end {align *} \]

    Свойство 4 утверждает, что если строка или столбец равны нулю, определитель равен нулю.{-1}) & = — 2 \ left (- \ dfrac {1} {2} \ right) — \ dfrac {3} {2} (1) \\ & = — \ dfrac {1} {2} \ конец {выравнивание *} \]

    Свойство 6 утверждает, что если любая строка или столбец матрицы умножается на константу, определитель умножается на тот же коэффициент. Таким образом,

    Пример \ (\ PageIndex {7} \): использование правила Крамера и определяющих свойств для решения системы

    Найдите решение данной системы \ (3 × 3 \).

    Решение

    Используя правило Крамера, имеем

    \ (D = \ begin {bmatrix} 2 & 4 & 4 \\ 3 & 7 & 7 \\ 1 & 2 & 2 \ end {bmatrix} \)

    Обратите внимание, что второй и третий столбцы идентичны.Согласно свойству 3 определитель будет равен нулю, поэтому решения либо нет, либо существует бесконечное число решений. Чтобы выяснить это, нам нужно провести отбор.

    1. Умножьте уравнение \ ref {eq10} на \ (- 2 \) и добавьте результат в уравнение \ ref {eq8}.

    Получение противоречивого утверждения означает, что система не имеет решения.

    Медиа

    Получите доступ к этим онлайн-ресурсам для получения дополнительных инструкций и практики с правилом Крамера.

    Решающие системы с правилом Крамера

    Цели обучения

    В этом разделе вы будете:

    • Оцените детерминанты 2 × 2.
    • Используйте правило Крамера для решения системы уравнений с двумя переменными.
    • Оцените детерминанты 3 × 3.
    • Используйте правило Крамера, чтобы решить систему из трех уравнений с тремя переменными.
    • Знать свойства определителей.

    Мы научились решать системы уравнений с двумя переменными и тремя переменными, используя несколько методов: подстановку, сложение, исключение Гаусса, использование обратной матрицы и построение графиков. Некоторые из этих методов применять проще, чем другие, и они более подходят в определенных ситуациях.В этом разделе мы изучим еще две стратегии решения систем уравнений.

    Вычисление определителя матрицы 2 × 2

    Определитель — это действительное число, которое может быть очень полезно в математике, потому что у него есть несколько приложений, таких как вычисление площади, объема и других величин. Здесь мы будем использовать определители, чтобы определить, является ли матрица обратимой, используя элементы квадратной матрицы, чтобы определить, существует ли решение системы уравнений.Однако, возможно, одним из наиболее интересных приложений является их использование в криптографии. Защищенные сигналы или сообщения иногда отправляются в виде матрицы. Расшифровать данные можно только с помощью обратимой матрицы и определителя. В наших целях мы ориентируемся на определитель как на показатель обратимости матрицы. Вычисление определителя матрицы требует следования определенным шаблонам, описанным в этом разделе.

    Найдите определитель матрицы 2 × 2

    Определитель матрицы [latex] \, 2 \ text {} × \ text {} 2 \, [/ latex], учитывая

    [латекс] A = \ left [\ begin {array} {cc} a & b \\ c & d \ end {array} \ right] [/ latex]

    определяется как

    Обратите внимание на изменение обозначений.Есть несколько способов указать определитель, включая [latex] \, \ mathrm {det} \ left (A \ right) \, [/ latex] и замену скобок в матрице прямыми линиями, [latex] \, | A |. [/ Латекс]

    Нахождение определителя матрицы 2 × 2

    Найдите определитель заданной матрицы.

    [латекс] A = \ left [\ begin {array} {cc} 5 & 2 \\ -6 & 3 \ end {array} \ right] [/ latex]

    Показать решение

    [латекс] \ begin {array} {l} \ mathrm {det} \ left (A \ right) = | \ begin {array} {cc} 5 & 2 \\ -6 & 3 \ end {array} | \ hfill \ \ \, \, \, \, \, \, \, \, \, \, \, \, \, \, \, \, = 5 \ left (3 \ right) — \ left (-6 \ right ) \ left (2 \ right) \ hfill \\ \, \, \, \, \, \, \, \, \, \, \, \, \, \, \, \, = 27 \ hfill \ end {array} [/ latex]

    Использование правила Крамера для решения системы двух уравнений с двумя переменными

    Теперь мы представим последний метод решения систем уравнений, использующий определители.Этот метод, известный как правило Крамера, восходит к середине 18 века и назван в честь своего новатора, швейцарского математика Габриэля Крамера (1704-1752), который представил его в 1750 году во введении к анализу алгоритмов Курбских Альгебриков. Правило Крамера — это жизнеспособный и эффективный метод поиска решений систем с произвольным числом неизвестных при условии, что у нас есть такое же количество уравнений, что и неизвестных.

    Правило Крамера даст нам единственное решение системы уравнений, если оно существует.Однако, если система не имеет решения или бесконечное количество решений, это будет обозначено нулевым определителем. Чтобы выяснить, является ли система непоследовательной или зависимой, необходимо использовать другой метод, например исключение.

    Чтобы понять правило Крамера, давайте внимательно рассмотрим, как мы решаем системы линейных уравнений с использованием основных операций со строками. Рассмотрим систему двух уравнений с двумя переменными.

    [латекс] \ begin {array} {c} {a} _ {1} x + {b} _ {1} y = {c} _ {1} \, \, \, \, \ left (1 \ right ) \\ {a} _ {2} x + {b} _ {2} y = {c} _ {2} \, \, \, \, \ left (2 \ right) \ end {array} [/ latex ]

    Мы исключаем одну переменную, используя операции со строками, и решаем для другой.Скажем, мы хотим найти [латекс] \, x. \, [/ Latex] Если уравнение (2) умножить на коэффициент, противоположный [латексу] \, y \, [/ latex] в уравнении (1 ), уравнение (1) умножается на коэффициент [латекс] \, y \, [/ latex] в уравнении (2), и мы складываем два уравнения, переменную [latex] \, y \, [/ latex ] будет исключен.

    [латекс] \ begin {array} {l} \ underset {\ _ \ _ \ _ \ _ \ _ \ _ \ _ \ _ \ _ \ _ \ _ \ _ \ _ \ _ \ _ \ _ \ _ \ _ \ _ \ _ \ _ \ _ \ _ \ _ \ _ \ _ \ _ \ _ \ _ \ _ \ _ \ _ \ _ \ _ \ _ \ _ \ _ \ _ \ _ \ _ \ _ \ _ \ _ \ _ \ _ \ _ \ _ \ _ \ _ \ _ \ _ \ _ \ _ \ _ \ _ \ _} {\ begin {array} {llll} \ hfill & \ hfill & \ hfill & \ hfill \\ \, \, \, \, {b} _ {2} {a} _ {1} x + {b} _ {2} {b} _ {1} y = {b} _ {2} {c} _ {1} \ hfill & \ hfill & \ hfill & \ text {Умножение} {R} _ {1} \ text {by} {b} _ {2} \ hfill \\ — {b} _ {1} {a } _ {2} x- {b} _ {1} {b} _ {2} y = — {b} _ {1} {c} _ {2} \ hfill & \ hfill & \ hfill & \ text { Умножить} {R} _ {2} \ text {by} — {b} _ {1} \ hfill \ end {array}} \ hfill \\ \, \, \, \ begin {array} {ll} {b } _ {2} {a} _ {1} x- {b} _ {1} {a} _ {2} x = {b} _ {2} {c} _ {1} — {b} _ { 1} {c} _ {2} \ hfill & \ hfill \ end {array} \ hfill \ end {array} [/ latex]

    Теперь решите [латекс] \, x.[/ латекс]

    [латекс] \ begin {array} {l} \, \, \, {b} _ {2} {a} _ {1} x- {b} _ {1} {a} _ {2} x = {b} _ {2} {c} _ {1} — {b} _ {1} {c} _ {2} \ hfill \\ \, \, \, x \ left ({b} _ {2} {a} _ {1} — {b} _ {1} {a} _ {2} \ right) = {b} _ {2} {c} _ {1} — {b} _ {1} {c } _ {2} \ hfill \\ \ text {} x = \ frac {{b} _ {2} {c} _ {1} — {b} _ {1} {c} _ {2}} {{ b} _ {2} {a} _ {1} — {b} _ {1} {a} _ {2}} = \ frac {\ left [\ begin {array} {cc} {c} _ {1 } & {b} _ {1} \\ {c} _ {2} & {b} _ {2} \ end {array} \ right]} {\ left [\ begin {array} {cc} {a} _ {1} & {b} _ {1} \\ {a} _ {2} & {b} _ {2} \ end {array} \ right]} \ hfill \ end {array} [/ latex]

    Аналогичным образом, чтобы решить для [latex] \, y, [/ latex], мы удалим [latex] \, x.[/ латекс]

    [латекс] \ begin {array} {l} \ underset {\ _ \ _ \ _ \ _ \ _ \ _ \ _ \ _ \ _ \ _ \ _ \ _ \ _ \ _ \ _ \ _ \ _ \ _ \ _ \ _ \ _ \ _ \ _ \ _ \ _ \ _ \ _ \ _ \ _ \ _ \ _ \ _ \ _ \ _ \ _ \ _ \ _ \ _ \ _ \ _ \ _ \ _ \ _ \ _ \ _ \ _ \ _ \ _ \ _ \ _ \ _ \ _ \ _ \ _ \ _ \ _} {\ begin {array} {llll} \ hfill & \ hfill & \ hfill & \ hfill \\ \, \, \, \, {a} _ {2} {a} _ {1} x + {a} _ {2} {b} _ {1} y = {a} _ {2} {c} _ {1} \ hfill & \ hfill & \ hfill & \ text {Умножение} {R} _ {1} \ text {by} {a} _ {2} \ hfill \\ — {a} _ {1} {a } _ {2} x- {a} _ {1} {b} _ {2} y = — {a} _ {1} {c} _ {2} \ hfill & \ hfill & \ hfill & \ text { Умножить} {R} _ {2} \ text {by} — {a} _ {1} \ hfill \ end {array}} \ hfill \\ \, \, \, \, \, \, \ begin {array } {ll} {a} _ {2} {b} _ {1} y- {a} _ {1} {b} _ {2} y = {a} _ {2} {c} _ {1} — {a} _ {1} {c} _ {2} \ hfill & \ hfill \ end {array} \ hfill \ end {array} [/ latex]

    Решение для [latex] \, y \, [/ latex] дает

    [латекс] \ begin {array} {l} {a} _ {2} {b} _ {1} y- {a} _ {1} {b} _ {2} y = {a} _ {2 } {c} _ {1} — {a} _ {1} {c} _ {2} \ hfill \\ y \ left ({a} _ {2} {b} _ {1} — {a} _ {1} {b} _ {2} \ right) = {a} _ {2} {c} _ {1} — {a} _ {1} {c} _ {2} \ hfill \\ \ text { } y = \ frac {{a} _ {2} {c} _ {1} — {a} _ {1} {c} _ {2}} {{a} _ {2} {b} _ {1 } — {a} _ {1} {b} _ {2}} = \ frac {{a} _ {1} {c} _ {2} — {a} _ {2} {c} _ {1} } {{a} _ {1} {b} _ {2} — {a} _ {2} {b} _ {1}} = \ frac {| \ begin {array} {cc} {a} _ { 1} & {c} _ {1} \\ {a} _ {2} & {c} _ {2} \ end {array} |} {| \ begin {array} {cc} {a} _ {1 } & {b} _ {1} \\ {a} _ {2} & {b} _ {2} \ end {array} |} \ hfill \ end {array} [/ latex]

    Обратите внимание, что знаменатель для [latex] \, x \, [/ latex] и [latex] \, y \, [/ latex] является определителем матрицы коэффициентов.

    Мы можем использовать эти формулы для решения для [latex] \, x \, [/ latex] и [latex] \, y, \, [/ latex], но Правило Крамера также вводит новые обозначения:

    Ключ к правилу Крамера заключается в замене интересующего столбца переменных столбцом констант и вычислении детерминантов. Затем мы можем выразить [латекс] \, x \, [/ latex] и [latex] \, y \, [/ latex] как частное двух определителей.

    Правило Крамера для систем 2 × 2

    Правило Крамера — это метод, который использует детерминанты для решения систем уравнений, которые имеют то же количество уравнений, что и переменные.

    Рассмотрим систему двух линейных уравнений с двумя переменными.

    [латекс] \ begin {array} {c} {a} _ {1} x + {b} _ {1} y = {c} _ {1} \\ {a} _ {2} x + {b} _ {2} y = {c} _ {2} \ end {array} [/ latex]

    Решение, использующее правило Крамера, дается как

    [латекс] x = \ frac {{D} _ {x}} {D} = \ frac {| \ begin {array} {cc} {c} _ {1} & {b} _ {1} \\ {c} _ {2} & {b} _ {2} \ end {array} |} {| \ begin {array} {cc} {a} _ {1} & {b} _ {1} \\ { a} _ {2} & {b} _ {2} \ end {array} |}, \, \, D \ ne 0; \, \, \ text {} \ text {} \, y = \ гидроразрыв {{D} _ {y}} {D} = \ frac {| \ begin {array} {cc} {a} _ {1} & {c} _ {1} \\ {a} _ {2} & {c} _ {2} \ end {array} |} {| \ begin {array} {cc} {a} _ {1} & {b} _ {1} \\ {a} _ {2} & {b} _ {2} \ end {array} |}, \, \, D \ ne 0.[/ латекс]

    Если мы решаем для [latex] \, x, \, [/ latex], столбец [latex] \, x \, [/ latex] заменяется столбцом констант. Если мы решаем для [latex] \, y, \, [/ latex], столбец [latex] \, y \, [/ latex] заменяется постоянным столбцом.

    Использование правила Крамера для решения системы 2 × 2

    Решите следующую систему [latex] \, 2 \ text {} × \ text {} 2 \, [/ latex], используя правило Крамера.

    [латекс] \ begin {array} {c} 12x + 3y = 15 \\ \ text {} 2x-3y = 13 \ end {array} [/ latex]

    Показать решение

    Решите относительно [латекс] \, x.[/ латекс]

    [латекс] x = \ frac {{D} _ {x}} {D} = \ frac {| \ begin {array} {rr} \ hfill 15 & \ hfill 3 \\ \ hfill 13 & \ hfill -3 \ end {array} |} {| \ begin {array} {rr} \ hfill 12 & \ hfill 3 \\ \ hfill 2 & \ hfill -3 \ end {array} |} = \ frac {-45-39} {- 36- 6} = \ frac {-84} {- 42} = 2 [/ latex]

    Решите для [latex] \, y. [/ Latex]

    [латекс] y = \ frac {{D} _ {y}} {D} = \ frac {| \ begin {array} {rr} \ hfill 12 & \ hfill 15 \\ \ hfill 2 & \ hfill 13 \ end { array} |} {| \ begin {array} {rr} \ hfill 12 & \ hfill 3 \\ \ hfill 2 & \ hfill -3 \ end {array} |} = \ frac {156-30} {- 36-6} = — \ frac {126} {42} = — 3 [/ латекс]

    Решение: [латекс] \, \ left (2, -3 \ right).[/ латекс]

    Попробуй

    Используйте правило Крамера для решения системы уравнений 2 × 2.

    [латекс] \ begin {массив} {l} \ text {} x + 2y = -11 \ hfill \\ -2x + y = -13 \ hfill \ end {array} [/ latex]

    Показать решение

    [латекс] \ влево (3, -7 \ вправо) [/ латекс]

    Вычисление определителя матрицы 3 × 3

    Найти определитель матрицы 2 × 2 несложно, но найти определитель матрицы 3 × 3 сложнее. Один из способов — увеличить матрицу 3 × 3 повторением первых двух столбцов, получив матрицу 3 × 5.Затем мы вычисляем сумму произведений записей на по каждой из трех диагоналей (от верхнего левого угла к нижнему правому) и вычитаем произведение записей на по каждой из трех диагоналей (нижний левый верхний правый). Это легче понять с помощью наглядного пособия и примера.

    Найдите определитель матрицы 3 × 3.

    [латекс] A = \ left [\ begin {array} {ccc} {a} _ {1} & {b} _ {1} & {c} _ {1} \\ {a} _ {2} & {b} _ {2} & {c} _ {2} \\ {a} _ {3} & {b} _ {3} & {c} _ {3} \ end {array} \ right] [/ латекс]

    1. Дополните [латекс] \, A \, [/ latex] первыми двумя столбцами.

      [латекс] \ mathrm {det} \ left (A \ right) = | \ begin {array} {ccc} {a} _ {1} & {b} _ {1} & {c} _ {1} \ \ {a} _ {2} & {b} _ {2} & {c} _ {2} \\ {a} _ {3} & {b} _ {3} & {c} _ {3} \ конец {массив} \, \, \, | \, \, \, \ begin {array} {c} {a} _ {1} \\ {a} _ {2} \\ {a} _ {3} \ end {array} \, \, \, \, \ begin {array} {c} {b} _ {1} \\ {b} _ {2} \\ {b} _ {3} \ end {массив } | [/ латекс]

    2. С верхнего левого угла в нижний правый: умножение значений по первой диагонали. Добавьте результат к произведению записей по второй диагонали. Добавьте этот результат к произведению записей по третьей диагонали.
    3. От левого нижнего угла до правого верхнего: вычтите произведение входов вверх по первой диагонали. Из этого результата вычтите произведение входов вверх по второй диагонали. Из этого результата вычтите произведение входов до третьей диагонали.

    Алгебра выглядит следующим образом:

    [латекс] | A | = {a} _ {1} {b} _ {2} {c} _ {3} + {b} _ {1} {c} _ {2} {a} _ {3 } + {c} _ {1} {a} _ {2} {b} _ {3} — {a} _ {3} {b} _ {2} {c} _ {1} — {b} _ {3} {c} _ {2} {a} _ {1} — {c} _ {3} {a} _ {2} {b} _ {1} [/ latex]

    Нахождение определителя матрицы 3 × 3

    Найдите определитель матрицы 3 × 3 по заданному

    [латекс] A = \ left [\ begin {array} {ccc} 0 & 2 & 1 \\ 3 & -1 & 1 \\ 4 & 0 & 1 \ end {array} \ right] [/ latex]

    Показать решение

    Дополните матрицу первыми двумя столбцами, а затем следуйте формуле.Таким образом,

    [латекс] \ begin {array} {l} | A | = | \ begin {array} {ccc} 0 & 2 & 1 \\ 3 & -1 & 1 \\ 4 & 0 & 1 \ end {array} \, \, | \ begin {array} {c} 0 \\ 3 \\ \, \, 4 \ end {array} \, \, \, \, \ begin {array} {c} 2 \\ -1 \\ 0 \ end { массив} | \ hfill \\ \, \, \, \, \, \, \, = 0 \ left (-1 \ right) \ left (1 \ right) +2 \ left (1 \ right) \ left ( 4 \ вправо) +1 \ влево (3 \ вправо) \ влево (0 \ вправо) -4 \ влево (-1 \ вправо) \ влево (1 \ вправо) -0 \ влево (1 \ вправо) \ влево (0 \ вправо) -1 \ влево (3 \ вправо) \ влево (2 \ вправо) \ hfill \\ \, \, \, \, \, \, \, = 0 + 8 + 0 + 4-0-6 \ hfill \\ \, \, \, \, \, \, \, = 6 \ hfill \ end {array} [/ latex]

    Попробуй

    Найдите определитель матрицы 3 × 3.

    [латекс] \ mathrm {det} \ left (A \ right) = | \ begin {array} {ccc} 1 & -3 & 7 \\ 1 & 1 & 1 \\ 1 & -2 & 3 \ end {array} | [/ latex ]

    Можем ли мы использовать тот же метод, чтобы найти определитель большей матрицы?

    Нет, этот метод работает только для [latex] \, 2 \ text {} × \ text {} 2 \, [/ latex] и [latex] \, \ text {3} \ text {} × \ text { } 3 \, [/ латексные] матрицы. Для больших матриц лучше всего использовать графическую утилиту или компьютерное программное обеспечение.

    Использование правила Крамера для решения системы трех уравнений с тремя переменными

    Теперь, когда мы можем найти определитель матрицы 3 × 3, мы можем применить правило Крамера для решения системы трех уравнений с тремя переменными.Правило Крамера простое и следует шаблону, соответствующему правилу Крамера для матриц 2 × 2. Однако по мере увеличения порядка матрицы до 3 × 3 требуется гораздо больше вычислений.

    Когда мы вычисляем детерминант, равный нулю, правило Крамера не дает никаких указаний на то, что у системы нет решения или бесконечное количество решений. Чтобы выяснить это, мы должны выполнить устранение в системе.

    Рассмотрим систему уравнений 3 × 3.

    [латекс] x = \ frac {{D} _ {x}} {D}, y = \ frac {{D} _ {y}} {D}, z = \ frac {{D} _ {z} } {D}, D \ ne 0 [/ латекс]

    где

    Если мы записываем определитель [latex] \, {D} _ {x}, [/ latex], мы заменяем столбец [latex] \, x \, [/ latex] на столбец констант.Если мы пишем определитель [latex] {D} _ {y}, [/ latex], мы заменяем столбец [latex] \, y \, [/ latex] на постоянный столбец. Если мы пишем определитель [latex] \, {D} _ {z}, [/ latex], мы заменяем столбец [latex] \, z \, [/ latex] постоянным столбцом. Всегда проверяйте ответ.

    Решение системы 3 × 3 с использованием правила Крамера

    Найдите решение данной системы 3 × 3, используя правило Крамера.

    [латекс] \ begin {array} {c} x + y-z = 6 \\ 3x-2y + z = -5 \\ x + 3y-2z = 14 \ end {array} [/ latex]

    Показать решение

    Используйте правило Крамера.

    [латекс] D = | \ begin {array} {ccc} 1 & \, \, 1 & -1 \\ 3 & -2 & \, \, \, 1 \\ 1 & \, \, 3 & -2 \ end {array} |, {D} _ {x} = | \ begin {array} {ccc} 6 & 1 & -1 \\ -5 & -2 & \, \, \, 1 \\ 14 & \, \, 3 & -2 \ end {массив } |, {D} _ {y} = | \ begin {array} {ccc} 1 & \, 6 & -1 \\ 3 & -5 & \, \, 1 \\ 1 & 14 & -2 \ end {array} |, { D} _ {z} = | \ begin {array} {ccc} 1 & \, 1 & 6 \\ 3 & -2 & -5 \\ 1 & \, \, 3 & 14 \ end {array} | [/ latex]

    Затем,

    [латекс] \ begin {array} {l} x = \ frac {{D} _ {x}} {D} = \ frac {-3} {- 3} = 1 \ hfill \\ y = \ frac { {D} _ {y}} {D} = \ frac {-9} {- 3} = 3 \ hfill \\ z = \ frac {{D} _ {z}} {D} = \ frac {6} {-3} = — 2 \ hfill \ end {array} [/ latex]

    Решение [латекс] \ left (1,3, -2 \ right).[/ латекс]

    Попробуй

    Используйте правило Крамера, чтобы решить матрицу 3 × 3.

    [латекс] \ begin {array} {r} \ hfill x-3y + 7z = 13 \\ \ hfill x + y + z = 1 \, \, \, \\ \ hfill x-2y + 3z = 4 \ , \, \, \ end {array} [/ latex]

    Показать решение

    [латекс] \ left (-2, \ frac {3} {5}, \ frac {12} {5} \ right) [/ latex]

    Использование правила Крамера для решения несовместимой системы

    Решите систему уравнений, используя правило Крамера.

    [латекс] \ begin {array} {l} 3x-2y = 4 \ text {} \ left (1 \ right) \\ 6x-4y = 0 \ text {} \ left (2 \ right) \ end {массив } [/ latex]

    Показать решение

    Начнем с нахождения определителей [латекс] \, D, {D} _ {x}, \ text {и} {D} _ {y}.[/ латекс]

    [латекс] D = | \ begin {array} {cc} 3 & -2 \\ 6 & -4 \ end {array} | = 3 \ left (-4 \ right) -6 \ left (-2 \ right) = 0 [/ латекс]

    Мы знаем, что нулевой определитель означает, что либо система не имеет решения, либо имеет бесконечное количество решений. Чтобы узнать, какой из них, мы используем процесс исключения. Наша цель — исключить одну из переменных.

    1. Умножить уравнение (1) на [латекс] \, — 2. [/ Latex]
    2. Добавьте результат к уравнению [латекс] \, \ left (2 \ right). [/ Latex]

    [латекс] \ begin {array} {l} \ underset {\ _ \ _ \ _ \ _ \ _ \ _ \ _ \ _ \ _ \ _ \ _ \ _ \ _ \ _ \ _} {\ begin {массив} {l} \ begin {array} {l} \ hfill \\ -6x + 4y \, \, \, \, = — 8 \ hfill \ end {array} \ hfill \\ \, \, \, 6x-4y \, \, \, \, \, \, = \, \, \, \, 0 \ hfill \ end {array}} \ hfill \\ \, \, \, \, \, \, \ , \, \, \, \, \, \, \, \, \, \, \, \, 0 \, \, \, \, \, \, = \, — 8 \ hfill \ end {array} [/ латекс]

    Мы получаем уравнение [латекс] \, 0 = -8, \, [/ латекс], которое неверно.Следовательно, у системы нет решения. График системы показывает две параллельные линии. См. (Рисунок).

    Рисунок 1.

    Используйте правило Крамера для решения зависимой системы

    Решите систему с бесконечным количеством решений.

    [латекс] \ begin {array} {rr} \ hfill x-2y + 3z = 0 & \ hfill \ left (1 \ right) \\ \ hfill 3x + y-2z = 0 & \ hfill \ left (2 \ right) \\ \ hfill 2x-4y + 6z = 0 & \ hfill \ left (3 \ right) \ end {array} [/ latex]

    Показать решение

    Давайте сначала найдем определитель.Создайте матрицу, дополненную первыми двумя столбцами.

    [латекс] | \ begin {array} {rrr} \ hfill 1 & \ hfill -2 & \ hfill 3 \\ \ hfill 3 & \ hfill 1 & \ hfill -2 \\ \ hfill 2 & \ hfill -4 & \ hfill 6 \ end { array} \ text {} | \ text {} \ begin {array} {rr} \ hfill 1 & \ hfill -2 \\ \ hfill 3 & \ hfill 1 \\ \ hfill 2 & \ hfill -4 \ end {array} | [ / латекс]

    Затем,

    [латекс] 1 \ влево (1 \ вправо) \ влево (6 \ вправо) + \ влево (-2 \ вправо) \ влево (-2 \ вправо) \ влево (2 \ вправо) +3 \ влево (3 \ вправо) \ влево (-4 \ вправо) -2 \ влево (1 \ вправо) \ влево (3 \ вправо) — \ влево (-4 \ вправо) \ влево (-2 \ вправо) \ влево (1 \ вправо) -6 \ влево (3 \ вправо) \ влево (-2 \ вправо) = 0 [/ латекс]

    Поскольку определитель равен нулю, решения либо нет, либо существует бесконечное количество решений.Чтобы выяснить это, нам нужно провести отбор.

    1. Умножьте уравнение (1) на [latex] \, — 2 \, [/ latex] и добавьте результат к уравнению (3):

      [латекс] \ frac {\ begin {array} {r} \ hfill -2x + 4y-6x = 0 \\ \ hfill 2x-4y + 6z = 0 \ end {array}} {\, \, \, \ , \, \, \, \, \, \, \, \, \, \, \, \, \, \, \, \, \, \, \, \, \, \, \, \, \ , \, \, 0 = 0} [/ латекс]

    2. Получение ответа [latex] \, 0 = 0, \, [/ latex] утверждение, которое всегда верно, означает, что система имеет бесконечное количество решений. Изобразив систему, мы можем увидеть, что две плоскости одинаковы, и обе они пересекают третью плоскость по прямой.См. (Рисунок).

      Рисунок 2.

    Понимание свойств детерминантов

    Есть много свойств определителей. Здесь перечислены некоторые свойства, которые могут быть полезны при вычислении определителя матрицы.

    Свойства детерминантов

    1. Если матрица имеет верхнюю треугольную форму, определитель равен произведению элементов по главной диагонали.
    2. Когда две строки меняются местами, определитель меняет знак.{-1} \, [/ latex] — обратная величина определителю матрицы [latex] \, A. [/ Latex]
    3. Если какая-либо строка или столбец умножается на константу, определитель умножается на тот же коэффициент.

    Иллюстрация свойств детерминантов

    Проиллюстрируйте каждое из свойств определителей.

    Показать решение

    Свойство 1 утверждает, что если матрица имеет верхнюю треугольную форму, определитель является произведением элементов по главной диагонали.

    [латекс] A = \ left [\ begin {array} {rrr} \ hfill 1 & \ hfill \, \, 2 & \ hfill 3 \\ \ hfill 0 & \ hfill \, \, 2 & \ hfill 1 \\ \ hfill 0 & \ hfill \, \, 0 & \ hfill -1 \ end {array} \ right] [/ latex]

    Дополните [латекс] \, A \, [/ latex] первыми двумя столбцами.

    [латекс] A = \ left [\ begin {array} {ccc} 1 & 2 & 3 \\ 0 & 2 & 1 \\ 0 & 0 & -1 \ end {array} | \, \, \, \ begin {array} {c } 1 \\ 0 \\ 0 \ end {массив} \, \, \, \, \ begin {array} {c} 2 \\ 2 \\ 0 \ end {array} \ right] [/ latex]

    Затем

    [латекс] \ begin {array} {l} \ mathrm {det} \ left (A \ right) = 1 \ left (2 \ right) \ left (-1 \ right) +2 \ left (1 \ right) \ влево (0 \ вправо) +3 \ влево (0 \ вправо) \ влево (0 \ вправо) -0 \ влево (2 \ вправо) \ влево (3 \ вправо) -0 \ влево (1 \ вправо) \ влево (1 \ вправо) +1 \ влево (0 \ вправо) \ влево (2 \ вправо) \ hfill \\ \, \, \, \, \, \, \, \, \, \, \, \, \ , \, \, \, = — 2 \ hfill \ end {array} [/ latex]

    Свойство 2 утверждает, что перестановка строк меняет знак.Учитывая

    [латекс] \ begin {массив} {l} \ begin {array} {l} \\ A = \ left [\ begin {array} {cc} -1 & 5 \\ 4 & -3 \ end {array} \ right ], \, \, \ mathrm {det} \ left (A \ right) = \ left (-1 \ right) \ left (-3 \ right) — \ left (4 \ right) \ left (5 \ right) = 3-20 = -17 \ end {массив} \ hfill \\ \ hfill \\ B = \ left [\ begin {array} {cc} 4 & -3 \\ -1 & 5 \ end {array} \ right], \, \, \ mathrm {det} \ left (B \ right) = \ left (4 \ right) \ left (5 \ right) — \ left (-1 \ right) \ left (-3 \ right) = 20 -3 = 17 \ hfill \ end {array} [/ latex]

    Свойство 3 утверждает, что если две строки или два столбца идентичны, определитель равен нулю.

    [латекс] \ begin {массив} {l} \, \, \, \, \, \, \, \, \, \, \, A = \ left [\ begin {array} {ccc} 1 & 2 & 2 \\ 2 & 2 & 2 \\ -1 & 2 & 2 \ end {array} \ text {} | \ text {} \ begin {array} {c} 1 \\ 2 \\ -1 \ end {array} \ begin {array } {c} 2 \\ 2 \\ 2 \ end {array} \ right] \ hfill \\ \ hfill \\ \ mathrm {det} \ left (A \ right) = 1 \ left (2 \ right) \ left (2 \ вправо) +2 \ влево (2 \ вправо) \ влево (-1 \ вправо) +2 \ влево (2 \ вправо) \ влево (2 \ вправо) +1 \ влево (2 \ вправо) \ влево ( 2 \ вправо) -2 \ влево (2 \ вправо) \ влево (1 \ вправо) -2 \ влево (2 \ вправо) \ влево (2 \ вправо) \ hfill \\ \, \, \, \, \, \, \, \, \, \, \, \, \, \, \, = 4-4 + 8 + 4-4-8 = 0 \ hfill \ end {array} [/ latex]

    Свойство 4 утверждает, что если строка или столбец равны нулю, определитель равен нулю.{-1} \ right) = — 2 \ left (- \ frac {1} {2} \ right) — \ left (\ frac {3} {2} \ right) \ left (1 \ right) = — \ гидроразрыв {1} {2} \ hfill \ end {array} [/ latex]

    Свойство 6 утверждает, что если любая строка или столбец матрицы умножается на константу, определитель умножается на тот же коэффициент. Таким образом,

    [латекс] \ begin {array} {l} A = \ left [\ begin {array} {cc} 1 & 2 \\ 3 & 4 \ end {array} \ right], \ mathrm {det} \ left (A \ right) = 1 \ left (4 \ right) -2 \ left (3 \ right) = — 2 \ hfill \\ \ hfill \\ B = \ left [\ begin {array} {cc} 2 \ left (1 \ right) & 2 \ left (2 \ right) \\ 3 & 4 \ end {array} \ right], \ mathrm {det} \ left (B \ right) = 2 \ left (4 \ right) -3 \ left ( 4 \ right) = — 4 \ hfill \ end {array} [/ latex]

    Использование правила Крамера и определяющих свойств для решения системы

    Найдите решение данной системы 3 × 3.

    [латекс] \ begin {array} {ll} 2x + 4y + 4z = 2 \ hfill & \ left (1 \ right) \ hfill \\ 3x + 7y + 7z = -5 \ hfill & \ left (2 \ right ) \ hfill \\ \ text {} x + 2y + 2z = 4 \ hfill & \ left (3 \ right) \ hfill \ end {array} [/ latex]

    Показать решение

    Используя правило Крамера, имеем

    [латекс] D = | \ begin {array} {ccc} 2 & 4 & 4 \\ 3 & 7 & 7 \\ 1 & 2 & 2 \ end {array} | [/ latex]

    Обратите внимание, что второй и третий столбцы идентичны. Согласно свойству 3 определитель будет равен нулю, поэтому решения либо нет, либо существует бесконечное число решений.Чтобы выяснить это, нам нужно провести отбор.

    1. Умножьте уравнение (3) на –2 и прибавьте результат к уравнению (1).

      [латекс] \ frac {\ begin {array} {l} -2x-4y-4x = -8 \ hfill \\ \ text {} 2x + 4y + 4z = 2 \, \, \, \, \, \ hfill \ end {array}} {\, \, \, \, \, \, \, \, \, \, \, \, \, \, \, \, \, \, \, \, \, \, \, \, \, \, \, \, \, \, \, 0 = -6} [/ латекс]

    Получение противоречивого утверждения означает, что система не имеет решения.

    Ключевые понятия

    • Определитель для [latex] \, \ left [\ begin {array} {cc} a & b \\ c & d \ end {array} \ right] \, [/ latex] — [latex] \, ad-bc .\, [/ latex] См. (рисунок).
    • Правило Крамера заменяет переменный столбец постоянным столбцом. Решениями являются [latex] \, x = \ frac {{D} _ {x}} {D}, y = \ frac {{D} _ {y}} {D}. \, [/ Latex] См. (Рисунок ).
    • Чтобы найти определитель матрицы 3 × 3, дополните ее двумя первыми столбцами. Сложите три диагональных входа (верхний левый нижний правый) и вычтите три диагональных входа (нижний левый верхний правый). См. (Рисунок).
    • Чтобы решить систему трех уравнений с тремя переменными с использованием правила Крамера, замените столбец переменных столбцом констант для каждого желаемого решения: [latex] \, x = \ frac {{D} _ {x}} {D}, y = \ frac {{D} _ {y}} {D}, z = \ frac {{D} _ {z}} {D}.\, [/ latex] См. (рисунок).
    • Правило Крамера также полезно для поиска решения системы уравнений без решения или с бесконечными решениями. См. (Рисунок) и (Рисунок).
    • Некоторые свойства определителей полезны для решения задач. Например:
      • Если матрица имеет верхнюю треугольную форму, определитель равен произведению элементов по главной диагонали.
      • Когда две строки меняются местами, определитель меняет знак.
      • Если две строки или два столбца идентичны, определитель равен нулю.{-1} \, [/ latex] — обратная величина определителю матрицы [latex] \, A. [/ Latex]
      • Если какая-либо строка или столбец умножается на константу, определитель умножается на тот же коэффициент. См. (Рисунок) и (Рисунок).

    Упражнения по разделам

    Устный

    Объясните, почему мы всегда можем вычислить определитель квадратной матрицы.

    Показать решение

    Определитель — это сумма и произведения элементов матрицы, поэтому вы всегда можете оценить этот продукт, даже если в конечном итоге он окажется равным нулю.

    Исследуя правило Крамера, объясните, почему не существует единственного решения системы, когда определитель вашей матрицы равен 0. Для простоты используйте матрицу [latex] \, 2 \, × \, 2 \, [/ latex].

    Объясните, что в терминах обратного значения для матрицы означает наличие определителя 0.

    Показать решение

    Обратного не существует.

    Определитель [latex] \, 2 \, × \, 2 \, [/ latex] matrix [latex] \, A \, [/ latex] равен 3. Если вы поменяете строки и умножите первую строку на 6 а во второй строке — 2, объясните, как найти определитель и дать ответ.

    Алгебраические

    Найдите определитель для следующих упражнений.

    [латекс] | \ begin {array} {cc} 1 & 2 \\ 3 & 4 \ end {array} | [/ latex]

    [латекс] | \ begin {массив} {rr} \ hfill -1 & \ hfill 2 \\ \ hfill 3 & \ hfill -4 \ end {array} | [/ latex]

    [латекс] | \ begin {массив} {rr} \ hfill 2 & \ hfill -5 \\ \ hfill -1 & \ hfill 6 \ end {array} | [/ latex]

    [латекс] | \ begin {array} {cc} -8 & 4 \\ -1 & 5 \ end {array} | [/ latex]

    [латекс] | \ begin {массив} {rr} \ hfill 1 & \ hfill 0 \\ \ hfill 3 & \ hfill -4 \ end {array} | [/ latex]

    [латекс] | \ begin {массив} {rr} \ hfill 10 & \ hfill 20 \\ \ hfill 0 & \ hfill -10 \ end {array} | [/ latex]

    [латекс] | \ begin {array} {cc} 10 & 0.2 \\ 5 & 0.1 \ end {array} | [/ latex]

    [латекс] | \ begin {массив} {rr} \ hfill 6 & \ hfill -3 \\ \ hfill 8 & \ hfill 4 \ end {array} | [/ latex]

    [латекс] | \ begin {array} {rr} \ hfill -2 & \ hfill -3 \\ \ hfill 3.1 & \ hfill 4,000 \ end {array} | [/ latex]

    [латекс] | \ begin {array} {rr} \ hfill -1.1 & \ hfill 0.6 \\ \ hfill 7.2 & \ hfill -0.5 \ end {array} | [/ latex]

    [латекс] | \ begin {array} {rrr} \ hfill -1 & \ hfill 0 & \ hfill 0 \\ \ hfill 0 & \ hfill 1 & \ hfill 0 \\ \ hfill 0 & \ hfill 0 & \ hfill -3 \ end {массив } | [/ латекс]

    [латекс] | \ begin {array} {rrr} \ hfill -1 & \ hfill 4 & \ hfill 0 \\ \ hfill 0 & \ hfill 2 & \ hfill 3 \\ \ hfill 0 & \ hfill 0 & \ hfill -3 \ end {массив } | [/ латекс]

    [латекс] | \ begin {array} {ccc} 1 & 0 & 1 \\ 0 & 1 & 0 \\ 1 & 0 & 0 \ end {array} | [/ latex]

    [латекс] | \ begin {array} {rrr} \ hfill 2 & \ hfill -3 & \ hfill 1 \\ \ hfill 3 & \ hfill -4 & \ hfill 1 \\ \ hfill -5 & \ hfill 6 & \ hfill 1 \ end { массив} | [/ латекс]

    [латекс] | \ begin {array} {rrr} \ hfill -2 & \ hfill 1 & \ hfill 4 \\ \ hfill -4 & \ hfill 2 & \ hfill -8 \\ \ hfill 2 & \ hfill -8 & \ hfill -3 \ конец {массив} | [/ латекс]

    [латекс] | \ begin {array} {rrr} \ hfill 6 & \ hfill -1 & \ hfill 2 \\ \ hfill -4 & \ hfill -3 & \ hfill 5 \\ \ hfill 1 & \ hfill 9 & \ hfill -1 \ end {array} | [/ латекс]

    [латекс] | \ begin {array} {rrr} \ hfill 5 & \ hfill 1 & \ hfill -1 \\ \ hfill 2 & \ hfill 3 & \ hfill 1 \\ \ hfill 3 & \ hfill -6 & \ hfill -3 \ end { массив} | [/ латекс]

    [латекс] | \ begin {array} {rrr} \ hfill 1.1 & \ hfill 2 & \ hfill -1 \\ \ hfill -4 & \ hfill 0 & \ hfill 0 \\ \ hfill 4.1 & \ hfill -0.4 & \ hfill 2.5 \ end {array} | [/ latex]

    [латекс] | \ begin {array} {rrr} \ hfill 2 & \ hfill -1.6 & \ hfill 3.1 \\ \ hfill 1.1 & \ hfill 3 & \ hfill -8 \\ \ hfill -9.3 & \ hfill 0 & \ hfill 2 \ end {array} | [/ latex]

    [латекс] | \ begin {array} {ccc} — \ frac {1} {2} & \ frac {1} {3} & \ frac {1} {4} \\ \ frac {1} {5} & — \ frac {1} {6} & \ frac {1} {7} \\ 0 & 0 & \ frac {1} {8} \ end {array} | [/ latex]

    Для следующих упражнений решите систему линейных уравнений, используя правило Крамера.

    [латекс] \ begin {array} {l} 2x-3y = -1 \\ 4x + 5y = 9 \ end {array} [/ latex]

    Показать решение

    [латекс] \ левый (1,1 \ правый) [/ латекс]

    [латекс] \ begin {array} {r} 5x-4y = 2 \\ -4x + 7y = 6 \ end {array} [/ latex]

    [латекс] \ begin {массив} {l} \ text {} 6x-3y = 2 \, \, \, \, \, \ hfill \\ -8x + 9y = -1 \ hfill \ end {array} [ / латекс]

    Показать решение

    [латекс] \ left (\ frac {1} {2}, \ frac {1} {3} \ right) [/ latex]

    [латекс] \ begin {array} {l} 2x + 6y = 12 \\ 5x-2y = 13 \ end {array} [/ latex]

    [латекс] \ begin {array} {l} 4x + 3y = 23 \, \, \ hfill \\ \ text {} 2x-y = -1 \ hfill \ end {array} [/ latex]

    Показать решение

    [латекс] \ левый (2,5 \ правый) [/ латекс]

    [латекс] \ begin {array} {l} 10x-6y = 2 \, \, \, \, \ hfill \\ -5x + 8y = -1 \ hfill \ end {array} [/ latex]

    [латекс] \ begin {array} {l} 4x-3y = -3 \\ 2x + 6y = -4 \ end {array} [/ latex]

    Показать решение

    [латекс] \ left (-1, — \ frac {1} {3} \ right) [/ latex]

    [латекс] \ begin {array} {r} 4x-5y = 7 \\ -3x + 9y = 0 \ end {array} [/ latex]

    [латекс] \ begin {array} {l} 4x + 10y = 180 \, \, \, \, \ hfill \\ -3x-5y = -105 \ hfill \ end {array} [/ latex]

    Показать решение

    [латекс] \ левый (15,12 \ правый) [/ латекс]

    [латекс] \ begin {массив} {l} \ text {} 8x-2y = -3 \ hfill \\ -4x + 6y = 4 \, \, \, \, \ hfill \ end {array} [/ latex ]

    Для следующих упражнений решите систему линейных уравнений, используя правило Крамера.

    [латекс] \ begin {массив} {l} \ text {} x + 2y-4z = -1 \ hfill \\ \ text {} 7x + 3y + 5z = 26 \, \, \ hfill \\ -2x- 6y + 7z = -6 \ hfill \ end {array} [/ latex]

    Показать решение

    [латекс] \ левый (1,3,2 \ правый) [/ латекс]

    [латекс] \ begin {array} {l} -5x + 2y-4z = -47 \ hfill \\ \ text {} 4x-3y-z = -94 \ hfill \\ \ text {} 3x-3y + 2z = 94 \, \, \, \, \ hfill \ end {array} [/ latex]

    [латекс] \ begin {массив} {l} \ text {} 4x + 5y-z = -7 \ hfill \\ -2x-9y + 2z = 8 \, \, \, \, \ hfill \\ \ text {} 5y + 7z = 21 \, \ hfill \ end {array} [/ latex]

    Показать решение

    [латекс] \ влево (-1,0,3 \ вправо) [/ латекс]

    [латекс] \ begin {array} {r} 4x-3y + 4z = 10 \\ 5x-2z = -2 \\ 3x + 2y-5z = -9 \ end {array} [/ latex]

    [латекс] \ begin {array} {l} 4x-2y + 3z = 6 \, \, \, \ hfill \\ \ text {} -6x + y = -2 \ hfill \\ 2x + 7y + 8z = 24 \ hfill \ end {array} [/ latex]

    Показать решение

    [латекс] \ влево (\ frac {1} {2}, 1,2 \ вправо) [/ латекс]

    [латекс] \ begin {array} {r} \ hfill 5x + 2y-z = 1 \, \, \, \, \, \\ \ hfill -7x-8y + 3z = 1.5 \\ \ hfill 6x-12y + z = 7 \, \, \, \, \ end {array} [/ latex]

    [латекс] \ begin {array} {l} \ text {} 13x-17y + 16z = 73 \, \, \, \, \ hfill \\ -11x + 15y + 17z = 61 \, \, \, \ , \ hfill \\ \ text {} 46x + 10y-30z = -18 \ hfill \ end {array} [/ latex]

    Показать решение

    [латекс] \ влево (2,1,4 \ вправо) [/ латекс]

    [латекс] \ begin {массив} {l} \ begin {array} {l} \ hfill \\ -4x-3y-8z = -7 \ hfill \ end {array} \ hfill \\ \ text {} 2x- 9y + 5z = 0,5 \ hfill \\ \ text {} 5x-6y-5z = -2 \ hfill \ end {array} [/ latex]

    [латекс] \ begin {array} {l} \ text {} 4x-6y + 8z = 10 \, \, \ hfill \\ -2x + 3y-4z = -5 \ hfill \\ \ text {} x + y + z = 1 \, \, \, \, \, \ hfill \ end {array} [/ latex]

    [латекс] \ begin {array} {r} \ hfill 4x-6y + 8z = 10 \, \, \, \, \, \\ \ hfill -2x + 3y-4z = -5 \, \, \, \\ \ hfill 12x + 18y-24z = -30 \ end {array} [/ latex]

    Технологии

    Для следующих упражнений используйте детерминантную функцию в графической утилите.

    [латекс] | \ begin {array} {rrrr} \ hfill 1 & \ hfill 0 & \ hfill 8 & \ hfill 9 \\ \ hfill 0 & \ hfill 2 & \ hfill 1 & \ hfill 0 \\ \ hfill 1 & \ hfill 0 & \ hfill 3 & \ hfill 0 \\ \ hfill 0 & \ hfill 2 & \ hfill 4 & \ hfill 3 \ end {array} | [/ latex]

    [латекс] | \ begin {array} {rrrr} \ hfill 1 & \ hfill 0 & \ hfill 2 & \ hfill 1 \\ \ hfill 0 & \ hfill -9 & \ hfill 1 & \ hfill 3 \\ \ hfill 3 & \ hfill 0 & \ hfill -2 & \ hfill -1 \\ \ hfill 0 & \ hfill 1 & \ hfill 1 & \ hfill -2 \ end {array} | [/ latex]

    [латекс] | \ begin {array} {rrrr} \ hfill \ frac {1} {2} & \ hfill 1 & \ hfill 7 & \ hfill 4 \\ \ hfill 0 & \ hfill \ frac {1} {2} & \ hfill 100 & \ hfill 5 \\ \ hfill 0 & \ hfill 0 & \ hfill 2 & \ hfill 2,000 \\ \ hfill 0 & \ hfill 0 & \ hfill 0 & \ hfill 2 \ end {array} | [/ latex]

    [латекс] | \ begin {array} {rrrr} \ hfill 1 & \ hfill 0 & \ hfill 0 & \ hfill 0 \\ \ hfill 2 & \ hfill 3 & \ hfill 0 & \ hfill 0 \\ \ hfill 4 & \ hfill 5 & \ hfill 6 & \ hfill 0 \\ \ hfill 7 & \ hfill 8 & \ hfill 9 & \ hfill 0 \ end {array} | [/ latex]

    Реальные приложения

    Для следующих упражнений создайте систему линейных уравнений для описания поведения.Затем вычислите определитель. Будет ли уникальное решение? Если да, найдите уникальное решение.

    Два числа в сумме дают 56. Одно число на 20 меньше другого.

    Два числа в сумме дают 104. Если вы сложите два раза первое число плюс два раза второе число, ваша сумма составит 208

    .

    Три числа в сумме дают 106. Первое число на 3 меньше второго. Третье число на 4 больше, чем первое.

    Три числа добавляют к 216.Сумма первых двух чисел равна 112. Третье число на 8 меньше, чем первые два числа вместе взятые.

    Для следующих упражнений создайте систему линейных уравнений для описания поведения. Затем решите систему для всех решений, используя правило Крамера.

    Вы вкладываете 10 000 долларов в два счета, которые получают 8% годовых и 5% годовых. В конце года на ваших комбинированных счетах было 10 710 долларов. Сколько было вложено в каждую учетную запись?

    Показать решение

    7000 долларов на первом счете, 3000 долларов на втором счете.

    Вы вкладываете 80 000 долларов в два счета, 22 000 долларов в один счет и 58 000 долларов в другой. В конце года, если исходить из простых процентов, вы заработали 2470 долларов в виде процентов. Второй счет получает на полпроцента меньше, чем удвоенный процент по первому счету. Какие процентные ставки по вашим счетам?

    Кинотеатру необходимо знать, сколько билетов для взрослых и детей было продано из 1200 билетов. Если детские билеты 5 долларов.95, билеты для взрослых стоят 11,15 долларов, а общая сумма выручки составила 12 756 долларов. Сколько билетов для детей и взрослых было продано?

    Показать решение

    120 детей, 1080 взрослых

    Концертная площадка продает одиночные билеты по 40 долларов каждый и билеты для пар по 65 долларов. Если общий доход составил 18 090 долларов и был продан 321 билет, сколько разовых билетов и сколько билетов для пар было продано?

    Вы решили покрасить свою кухню в зеленый цвет. Вы создаете цвет краски, смешивая желтую и синюю краски.Вы не можете вспомнить, сколько галлонов каждого цвета было добавлено в вашу смесь, но вы знаете, что всего было 10 галлонов. Кроме того, вы сохранили квитанцию ​​и знаете, что общая потраченная сумма составила 29,50 долларов США. Если каждый галлон желтого стоит 2,59 доллара, а каждый галлон синего стоит 3,19 доллара, сколько галлонов каждого цвета входит в вашу зеленую смесь?

    Вы продали на фермерском рынке шарфы двух типов и хотите знать, какой из них пользуется большей популярностью. Всего было продано 56 шарфов, желтый платок стоил 10 долларов, а фиолетовый — 11 долларов.Если ваш общий доход составил 583 доллара, сколько желтых и фиолетовых шарфов было продано?

    В вашем саду выращивали два вида помидоров: зеленый и красный. Красный весит 10 унций, а зеленый — 4 унции. У вас 30 помидоров, а общий вес составляет 13 фунтов 14 унций. Сколько у вас помидоров каждого вида?

    Показать решение

    13 зеленых помидоров, 17 красных помидоров

    На рынке три самых популярных овоща составляют 53% продаж овощей. Продажи кукурузы на 4% выше, чем у брокколи, у которой на 5% больше продаж, чем у лука.Какую долю занимает каждый овощ на рынке?

    На этом же рынке три самых популярных фрукта составляют 37% от общего количества проданных фруктов. Клубника продается вдвое больше, чем апельсины, а киви продаются на один процентный пункт больше, чем апельсины. Для каждого фрукта найдите процент от общего количества проданных фруктов.

    Показать решение

    Клубника 18%, апельсины 9%, киви 10%

    Три ансамбля выступили на концертной площадке. Первый диапазон взимал 15 долларов за билет, второй диапазон — 45 долларов за билет, а последний диапазон — 22 доллара за билет.Было продано 510 билетов на общую сумму 12 700 долларов. Если у первой группы было на 40 человек больше аудитории, чем у второй, сколько билетов было продано каждой группе?

    В кинотеатре продаются билеты на три фильма. Билеты на первый фильм стоили 5 долларов, билеты на второй фильм — 11 долларов, а третий фильм — 12 долларов. На первый фильм было продано 100 билетов. Всего было продано 642 билета, общий доход составил 6 774 доллара. Сколько билетов на каждый фильм было продано?

    Показать решение

    100 для фильма 1, 230 для фильма 2, 312 для фильма 3

    Мужчины в возрасте 20–29, 30–39 и 40–49 лет в прошлом году составляли 78% заключенных.В этом году эти же возрастные группы составили 82,08% населения. Возрастная группа 20–29 лет увеличилась на 20%, возрастная группа 30–39 лет увеличилась на 2%, а возрастная группа 40–49 лет уменьшилась до [latex] \, \ frac {3} {4} \, [/ latex] их предыдущего населения. Первоначально в возрастной группе 30–39 лет было на 2% больше заключенных, чем в возрастной группе 20–29 лет. Определите процентную долю заключенных для каждой возрастной группы в прошлом году.

    В женской тюрьме по дороге общее количество заключенных в возрасте от 20 до 49 лет составило 5 525 человек. В этом году возрастная группа 20–29 лет увеличилась на 10%, возрастная группа 30–39 лет уменьшилась на 20%, а возрастная группа 40–49 лет увеличилась вдвое.Сейчас в тюрьме 6040 заключенных. Первоначально в возрастной группе 30–39 лет их было на 500 человек больше, чем в возрастной группе 20–29 лет. Определите количество заключенных для каждой возрастной группы в прошлом году.

    Показать решение

    20–29: 2,100, 30–39: 2,600, 40–49: 825

    Для следующих упражнений используйте этот сценарий: Компания, заботящаяся о своем здоровье, решает сделать смесь из миндаля, сушеной клюквы и кешью в шоколаде. Информация о питательной ценности этих продуктов показана на (Рисунок).

    Жир (г) Белок (г) Углеводы (г)
    Миндаль (10) 6 2 3
    Клюква (10) 0.02 0 8
    Кешью (10) 7 3,5 5,5

    Для специальной «низкоуглеводной» смеси для трейлов имеется 1000 штук смеси. Общее количество углеводов — 425 г, а общее количество жиров — 570,2 г. Если кешью на 200 штук больше, чем клюквы, сколько каждого из них входит в состав смеси?

    Для «походной» смеси в смеси 1000 штук, содержащих 390 штук.8 г жира и 165 г белка. Если миндаля столько же, сколько и кешью, сколько каждого из них входит в состав смеси?

    Показать решение

    300 миндальных орехов, 400 клюквы, 300 кешью

    Для смеси «усилитель энергии» в смеси 1000 штук, содержащих 145 г белка и 625 г углеводов. Если сумма миндальных орехов и кешью эквивалентна количеству клюквы, сколько каждого из них входит в состав смеси?

    Обзор упражнений

    Системы линейных уравнений: две переменные

    В следующих упражнениях определите, является ли упорядоченная пара решением системы уравнений.

    [латекс] \ begin {array} {l} 3x-y = 4 \\ x + 4y = -3 \, \ end {array} [/ latex] and [latex] \, \ left (-1,1 \ справа) [/ латекс]

    [латекс] \ begin {array} {l} 6x-2y = 24 \\ -3x + 3y = 18 \, \ end {array} [/ latex] and [latex] \, \ left (9,15 \ right ) [/ латекс]

    В следующих упражнениях используйте подстановку для решения системы уравнений.

    [латекс] \ begin {array} {l} 10x + 5y = -5 \ hfill \\ \, \, \, 3x-2y = -12 \ hfill \ end {array} [/ latex]

    Показать решение

    [латекс] \ влево (-2,3 \ вправо) [/ латекс]

    [латекс] \ begin {array} {l} \ frac {4} {7} x + \ frac {1} {5} y = \ frac {43} {70} \\ \ frac {5} {6} x — \ frac {1} {3} y = — \ frac {2} {3} \ end {array} [/ latex]

    [латекс] \ begin {array} {l} 5x + 6y = 14 \\ 4x + 8y = 8 \ end {array} [/ latex]

    Показать решение

    [латекс] \ влево (4, -1 \ вправо) [/ латекс]

    В следующих упражнениях используйте сложение для решения системы уравнений.

    [латекс] \ begin {array} {l} 3x + 2y = -7 \\ 2x + 4y = 6 \ end {array} [/ latex]

    [латекс] \ begin {array} {r} 3x + 4y = 2 \\ 9x + 12y = 3 \ end {array} [/ latex]

    [латекс] \ begin {array} {l} 8x + 4y = 2 \\ 6x-5y = 0,7 \ end {array} [/ latex]

    Для следующих упражнений напишите систему уравнений для решения каждой задачи. Решите систему уравнений.

    Завод имеет стоимость производства [латекс] \, C \ left (x \ right) = 150x + 15 \ text {,} 000 \, [/ latex] и функцию дохода [латекс] \, R \ left ( х \ вправо) = 200x.\, [/ latex] Какая точка безубыточности?

    Показать решение

    [латекс] \ влево (300,60,000 \ вправо) [/ латекс]

    Исполнитель взимает [латекс] \, C \ left (x \ right) = 50x + 10 \ text {,} 000, \, [/ latex], где [latex] \, x \, [/ latex] — общая сумма количество посетителей на шоу. Стоимость билета составляет 75 долларов. После того, как сколько людей купит билеты, место проведения станет безубыточным, и какова общая стоимость билетов, проданных в этот момент?

    Показать решение

    [латекс] \ влево (400,30,000 \ вправо) [/ латекс]

    Системы линейных уравнений: три переменные

    Для следующих упражнений решите систему трех уравнений, используя замену или сложение.

    [латекс] \ begin {array} {l} \ text {} 0,5x-0,5y = 10 \ hfill \\ \ text {} -0,2y + 0,2x = 4 \ hfill \\ \ text {} 0,1x + 0.1z = 2 \ hfill \ end {array} [/ latex]

    Показать решение

    [латекс] \ влево (10, -10,10 \ вправо) [/ латекс]

    [латекс] \ begin {array} {r} \ hfill 5x + 3y-z = 5 \, \, \, \\ \ hfill 3x-2y + 4z = 13 \\ \ hfill 4x + 3y + 5z = 22 \ конец {array} [/ latex]

    [латекс] \ begin {array} {r} x + y + z = 1 \\ 2x + 2y + 2z = 1 \\ 3x + 3y = 2 \ end {array} [/ latex]

    [латекс] \ begin {массив} {l} \ text {} 2x-3y + z = -1 \ hfill \\ \ text {} x + y + z = -4 \ hfill \\ \ text {} 4x + 2y-3z = 33 \ hfill \ end {array} [/ latex]

    [латекс] \ begin {array} {l} \, \, 3x + 2y-z = -10 \ hfill \\ \, \, \, \, x-y + 2z = 7 \ hfill \\ -x + 3y + z = -2 \ hfill \ end {array} [/ latex]

    Показать решение

    [латекс] \ влево (-1, -2,3 \ вправо) [/ латекс]

    [латекс] \ begin {array} {r} \ hfill 3x + 4z = -11 \\ \ hfill x-2y = 5 \, \, \, \, \, \, \, \\ \ hfill 4y-z = -10 \ end {array} [/ latex]

    [латекс] \ begin {array} {r} 2x-3y + z = 0 \\ 2x + 4y-3z = 0 \\ 6x-2y-z = 0 \ end {array} [/ latex]

    Показать решение

    [латекс] \ left (x, \ frac {8x} {5}, \ frac {14x} {5} \ right) [/ latex]

    [латекс] \ begin {array} {r} 6x-4y-2z = 2 \\ 3x + 2y-5z = 4 \\ 6y-7z = 5 \ end {array} [/ latex]

    Для следующих упражнений напишите систему уравнений для решения каждой задачи.Решите систему уравнений.

    Три нечетных числа в сумме дают 61. Меньшее на треть больше, а среднее число на 16 меньше большего. Какие три числа?

    Местный театр распродает билеты на их спектакль. Они продают все 500 билетов на общую сумму 8 070 долларов. Билеты стоили 15 долларов для студентов, 12 долларов для детей и 18 долларов для взрослых. Если группа продала в три раза больше билетов для взрослых, чем детских, сколько билетов каждого типа было продано?

    Системы нелинейных уравнений и неравенств: две переменные

    Для следующих упражнений решите систему нелинейных уравнений.{2}} [/ латекс]

    Матрицы и матричные операции

    Для следующих упражнений выполните требуемые операции с заданными матрицами.

    [латекс] A = \ left [\ begin {array} {rr} \ hfill 4 & \ hfill -2 \\ \ hfill 1 & \ hfill 3 \ end {array} \ right], B = \ left [\ begin {array } {rrr} \ hfill 6 & \ hfill 7 & \ hfill -3 \\ \ hfill 11 & \ hfill -2 & \ hfill 4 \ end {array} \ right], C = \ left [\ begin {array} {r} \ hfill \ begin {array} {cc} 6 & 7 \\ 11 & -2 \ end {array} \\ \ hfill \ begin {array} {cc} 14 & 0 \ end {array} \ end {array} \ right], D = \ left [\ begin {array} {rrr} \ hfill 1 & \ hfill -4 & \ hfill 9 \\ \ hfill 10 & \ hfill 5 & \ hfill -7 \\ \ hfill 2 & \ hfill 8 & \ hfill 5 \ end {array} \ right], E = \ left [\ begin {array} {rrr} \ hfill 7 & \ hfill -14 & \ hfill 3 \\ \ hfill 2 & \ hfill -1 & \ hfill 3 \\ \ hfill 0 & \ hfill 1 & \ hfill 9 \ конец {массив} \ справа] [/ латекс]

    Показать решение

    [латекс] \ left [\ begin {array} {cc} -16 & 8 \\ -4 & -12 \ end {array} \ right] [/ latex]

    Показать решение

    undefined; размеры не совпадают

    Показать решение

    undefined; внутренние размеры не соответствуют

    Показать решение

    [латекс] \ left [\ begin {array} {ccc} 113 & 28 & 10 \\ 44 & 81 & -41 \\ 84 & 98 & -42 \ end {array} \ right] [/ latex]

    Показать решение

    [латекс] \ left [\ begin {array} {ccc} -127 & -74 & 176 \\ -2 & 11 & 40 \\ 28 & 77 & 38 \ end {array} \ right] [/ latex]

    Показать решение

    undefined; внутренние размеры не соответствуют

    Решение систем с исключением Гаусса

    Для следующих упражнений напишите систему линейных уравнений из расширенной матрицы.Укажите, будет ли уникальное решение.

    [латекс] \ left [\ begin {array} {rrr} \ hfill 1 & \ hfill 0 & \ hfill -3 \\ \ hfill 0 & \ hfill 1 & \ hfill 2 \\ \ hfill 0 & \ hfill 0 & \ hfill 0 \ end { массив} \ text {} | \ text {} \ begin {array} {r} \ hfill 7 \\ \ hfill -5 \\ \ hfill 0 \ end {array} \ right] [/ latex]

    Показать решение

    [латекс] \ begin {array} {l} x-3z = 7 \\ y + 2z = -5 \, \ end {array} [/ latex] с бесконечными решениями

    [латекс] \ left [\ begin {array} {rrr} \ hfill 1 & \ hfill 0 & \ hfill 5 \\ \ hfill 0 & \ hfill 1 & \ hfill -2 \\ \ hfill 0 & \ hfill 0 & \ hfill 0 \ end { массив} \ text {} | \ text {} \ begin {array} {r} \ hfill -9 \\ \ hfill 4 \\ \ hfill 3 \ end {array} \ right] [/ latex]

    Для следующих упражнений напишите расширенную матрицу из системы линейных уравнений.

    [латекс] \ begin {массив} {l} \\ \ begin {array} {r} \ hfill -2x + 2y + z = 7 \\ \ hfill 2x-8y + 5z = 0 \\ \ hfill 19x-10y + 22z = 3 \ end {массив} \ end {array} [/ latex]

    Показать решение

    [латекс] \ left [\ begin {array} {rrr} \ hfill -2 & \ hfill 2 & \ hfill 1 \\ \ hfill 2 & \ hfill -8 & \ hfill 5 \\ \ hfill 19 & \ hfill -10 & \ hfill 22 \ end {array} \ text {} | \ text {} \ begin {array} {r} \ hfill 7 \\ \ hfill 0 \\ \ hfill 3 \ end {array} \ right] [/ latex]

    [латекс] \ begin {array} {l} \, \, \, \, \, 4x + 2y-3z = 14 \ hfill \\ -12x + 3y + z = 100 \ hfill \\ \, \, \ , \, \, 9x-6y + 2z = 31 \ hfill \ end {array} [/ latex]

    [латекс] \ begin {array} {r} \ hfill x + 3z = 12 \, \\ \ hfill -x + 4y = 0 \, \, \, \, \\ \ hfill y + 2z = -7 \ конец {array} [/ latex]

    Показать решение

    [латекс] \ left [\ begin {array} {rrr} \ hfill 1 & \ hfill 0 & \ hfill 3 \\ \ hfill -1 & \ hfill 4 & \ hfill 0 \\ \ hfill 0 & \ hfill 1 & \ hfill 2 \ end { массив} \ text {} | \ text {} \ begin {array} {r} \ hfill 12 \\ \ hfill 0 \\ \ hfill -7 \ end {array} \ right] [/ latex]

    Для следующих упражнений решите систему линейных уравнений, используя метод исключения Гаусса.

    [латекс] \ begin {array} {r} 3x-4y = -7 \\ -6x + 8y = 14 \ end {array} [/ latex]

    [латекс] \ begin {array} {r} 3x-4y = 1 \\ -6x + 8y = 6 \ end {array} [/ latex]

    [латекс] \ begin {массив} {l} \ begin {array} {l} \\ -1.1x-2.3y = 6.2 \ end {array} \ hfill \\ -5.2x-4.1y = 4.3 \ hfill \ конец {array} [/ latex]

    [латекс] \ begin {array} {r} \ hfill 2x + 3y + 2z = 1 \, \, \, \, \, \\ \ hfill -4x-6y-4z = -2 \\ \ hfill 10x + 15y + 10z = 0 \, \, \, \, \, \ end {array} [/ latex]

    [латекс] \ begin {array} {r} \ hfill -x + 2y-4z = 8 \, \, \, \, \\ \ hfill 3y + 8z = -4 \\ \ hfill -7x + y + 2z = 1 \, \, \, \, \ end {array} [/ latex]

    Решающие системы с инверторами

    Для следующих упражнений найдите обратную матрицу.

    [латекс] \ left [\ begin {array} {rr} \ hfill -0.2 & \ hfill 1.4 \\ \ hfill 1.2 & \ hfill -0.4 \ end {array} \ right] [/ latex]

    Показать решение

    [латекс] \ frac {1} {8} \ left [\ begin {array} {cc} 2 & 7 \\ 6 & 1 \ end {array} \ right] [/ latex]

    [латекс] \ left [\ begin {array} {rr} \ hfill \ frac {1} {2} & \ hfill — \ frac {1} {2} \\ \ hfill — \ frac {1} {4} & \ hfill \ frac {3} {4} \ end {array} \ right] [/ latex]

    [латекс] \ left [\ begin {array} {ccc} 12 & 9 & -6 \\ -1 & 3 & 2 \\ -4 & -3 & 2 \ end {array} \ right] [/ latex]

    [латекс] \ left [\ begin {array} {ccc} 2 & 1 & 3 \\ 1 & 2 & 3 \\ 3 & 2 & 1 \ end {array} \ right] [/ latex]

    Для следующих упражнений найдите решения, вычислив обратную матрицу.

    [латекс] \ begin {массив} {l} \, \, \, \, 0,3x-0,1y = -10 \ hfill \\ -0,1x + 0,3y = 14 \ hfill \ end {array} [/ latex ]

    Показать решение

    [латекс] \ влево (-20,40 \ вправо) [/ латекс]

    [латекс] \ begin {array} {l} \, \, \, \, \, \, \, \, 0,4x-0,2y = -0,6 \ hfill \\ -0,1x + 0,05y = 0,3 \ hfill \ end {array} [/ latex]

    [латекс] \ begin {array} {r} 4x + 3y-3z = -4,3 \\ 5x-4y-z = -6,1 \\ x + z = -0,7 \ end {array} [/ latex]

    Показать решение

    [латекс] \ влево (-1,0,2,0,3 \ вправо) [/ латекс]

    [латекс] \ begin {массив} {r} \ hfill \ begin {array} {l} \\ -2x-3y + 2z = 3 \ end {array} \\ \ hfill -x + 2y + 4z = -5 \\ \ hfill -2y + 5z = -3 \ end {array} [/ latex]

    Для следующих упражнений напишите систему уравнений для решения каждой задачи.Решите систему уравнений.

    Студентов попросили принести в класс их любимые фрукты. 90% фруктов состояли из бананов, яблок и апельсинов. Если апельсины были наполовину популярнее бананов, а яблоки на 5% популярнее бананов, каков процент каждого отдельного фрукта?

    Показать решение

    17% апельсинов, 34% бананов, 39% яблок

    Женское общество провело распродажу выпечки, чтобы собрать деньги, и продавало пирожные и печенье с шоколадной крошкой. Они оценили пирожные в 2 доллара и печенье с шоколадной крошкой в ​​1 доллар.Они собрали 250 долларов и продали 175 вещей. Сколько было продано пирожных и печенья?

    Решение систем с правилом Крамера

    Найдите определитель для следующих упражнений.

    [латекс] | \ begin {array} {cc} 100 & 0 \\ 0 & 0 \ end {array} | [/ latex]

    [латекс] | \ begin {array} {cc} 0,2 & -0,6 \\ 0,7 & -1,1 \ end {array} | [/ latex]

    [латекс] | \ begin {array} {ccc} -1 & 4 & 3 \\ 0 & 2 & 3 \\ 0 & 0 & -3 \ end {array} | [/ latex]

    [латекс] | \ begin {array} {ccc} \ sqrt {2} & 0 & 0 \\ 0 & \ sqrt {2} & 0 \\ 0 & 0 & \ sqrt {2} \ end {array} | [/ latex]

    В следующих упражнениях используйте правило Крамера для решения линейных систем уравнений.

    [латекс] \ begin {массив} {r} \ hfill 4x-2y = 23 \, \, \, \, \\ \ hfill -5x-10y = -35 \ end {array} [/ latex]

    Показать решение

    [латекс] \ left (6, \ frac {1} {2} \ right) [/ latex]

    [латекс] \ begin {array} {l} 0,2x-0,1y = 0 \\ -0,3x + 0,3y = 2,5 \ end {array} [/ latex]

    [латекс] \ begin {массив} {r} \ hfill -0,5x + 0,1y = 0,3 \, \, \, \\ \ hfill -0,25x + 0,05y = 0,15 \ end {array} [/ latex]

    [латекс] \ begin {array} {l} x + 6y + 3z = 4 \\ 2x + y + 2z = 3 \\ 3x-2y + z = 0 \ end {array} [/ latex]

    [латекс] \ begin {array} {r} \ hfill 4x-3y + 5z = — \ frac {5} {2} \\ \ hfill 7x-9y-3z = \ frac {3} {2} \, \ , \, \, \\ \ hfill x-5y-5z = \ frac {5} {2} \, \, \, \, \ end {array} [/ latex]

    Показать решение

    [латекс] \ left (0,0, — \ frac {1} {2} \ right) [/ latex]

    [латекс] \ begin {array} {r} \ frac {3} {10} x- \ frac {1} {5} y- \ frac {3} {10} z = — \ frac {1} {50 } \\ \ frac {1} {10} x- \ frac {1} {10} y- \ frac {1} {2} z = — \ frac {9} {50} \\ \ frac {2} { 5} x- \ frac {1} {2} y- \ frac {3} {5} z = — \ frac {1} {5} \ end {array} [/ latex]

    Практический тест

    Является ли следующая упорядоченная пара решением системы уравнений?

    [латекс] \ begin {массив} {l} \\ \ begin {array} {l} -5x-y = 12 \, \ hfill \\ x + 4y = 9 \ hfill \ end {array} \ end {массив } [/ latex] с [латексом] \, \ left (-3,3 \ right) [/ latex]

    Для следующих упражнений решите системы линейных и нелинейных уравнений с помощью замены или исключения.Укажите, если решения не существует.

    [латекс] \ begin {array} {r} \ frac {1} {2} x- \ frac {1} {3} y = 4 \\ \ frac {3} {2} xy = 0 \ end {массив } [/ latex]

    [латекс] \ begin {массив} {r} \ hfill \ begin {array} {l} \\ — \ frac {1} {2} x-4y = 4 \ end {array} \\ \ hfill 2x + 16y = 2 \ end {array} [/ latex]

    [латекс] \ begin {массив} {r} \ hfill 5x-y = 1 \, \, \, \, \\ \ hfill -10x + 2y = -2 \ end {array} [/ latex]

    [латекс] \ begin {array} {l} 4x-6y-2z = \ frac {1} {10} \ hfill \\ \, \, \, x-7y + 5z = — \ frac {1} {4 } \ hfill \\ 3x + 6y-9z = \ frac {6} {5} \ hfill \ end {array} [/ latex]

    Показать решение

    [латекс] \ frac {1} {20} \ left (10,5,4 \ right) [/ latex]

    [латекс] \ begin {array} {r} x + z = 20 \\ x + y + z = 20 \\ x + 2y + z = 10 \ end {array} [/ latex]

    [латекс] \ begin {array} {r} 5x-4y-3z = 0 \\ 2x + y + 2z = 0 \\ x-6y-7z = 0 \ end {array} [/ latex]

    Показать решение

    [латекс] \ left (x, \ frac {16x} {5} — \ frac {13x} {5} \ right) [/ latex]

    [латекс] \ begin {array} {l} y = {x} ^ {2} + 2x-3 \\ y = x-1 \ end {array} [/ latex]

    [латекс] \ begin {array} {l} {y} ^ {2} + {x} ^ {2} = 25 \\ {y} ^ {2} -2 {x} ^ {2} = 1 \ конец {array} [/ latex]

    Показать решение

    [латекс] \ left (-2 \ sqrt {2}, — \ sqrt {17} \ right), \ left (-2 \ sqrt {2}, \ sqrt {17} \ right), \ left (2 \ sqrt {2}, — \ sqrt {17} \ right), \ left (2 \ sqrt {2}, \ sqrt {17} \ right) [/ latex]

    Для следующих упражнений нарисуйте следующие неравенства. {- 1} [/ latex]

    Показать решение

    [латекс] \ left [\ begin {array} {cc} 12 & -20 \\ -15 & 30 \ end {array} \ right] [/ latex]

    [латекс] \ mathrm {det} | \ begin {array} {cc} 0 & 0 \\ 400 & 4 \ text {,} 000 \ end {array} | [/ latex]

    [латекс] \ mathrm {det} | \ begin {array} {rrr} \ hfill \ frac {1} {2} & \ hfill — \ frac {1} {2} & \ hfill 0 \\ \ hfill — \ frac {1} {2} & \ hfill 0 & \ hfill \ frac {1} {2} \\ \ hfill 0 & \ hfill \ frac {1} {2} & \ hfill 0 \ end {array} | [/ latex]

    Показать решение

    [латекс] — \ frac {1} {8} [/ латекс]

    Если [latex] \, \ mathrm {det} \ left (A \ right) = — 6, \, [/ latex], что будет определяющим, если вы поменяете местами строки 1 и 3, умножите вторую строку на 12 и взяли обратное?

    Перепишите систему линейных уравнений в виде расширенной матрицы.

    [латекс] \ begin {array} {l} 14x-2y + 13z = 140 \ hfill \\ -2x + 3y-6z = -1 \ hfill \\ x-5y + 12z = 11 \ hfill \ end {array} [/ латекс]

    Показать решение

    [латекс] \ left [\ begin {array} {rrr} \ hfill 14 & \ hfill -2 & \ hfill 13 \\ \ hfill -2 & \ hfill 3 & \ hfill -6 \\ \ hfill 1 & \ hfill -5 & \ hfill 12 \ end {array} \ text {} | \ text {} \ begin {array} {r} \ hfill 140 \\ \ hfill -1 \\ \ hfill 11 \ end {array} \ right] [/ latex]

    Перепишите расширенную матрицу как систему линейных уравнений.

    [латекс] \ left [\ begin {array} {rrr} \ hfill 1 & \ hfill 0 & \ hfill 3 \\ \ hfill -2 & \ hfill 4 & \ hfill 9 \\ \ hfill -6 & \ hfill 1 & \ hfill 2 \ end {массив} | \ begin {array} {r} \ hfill 12 \\ \ hfill -5 \\ \ hfill 8 \ end {array} \ right] [/ latex]

    В следующих упражнениях используйте метод исключения Гаусса для решения систем уравнений.

    [латекс] \ begin {array} {r} x-6y = 4 \\ 2x-12y = 0 \ end {array} [/ latex]

    [латекс] \ begin {array} {r} \ hfill 2x + y + z = -3 \\ \ hfill x-2y + 3z = 6 \, \, \, \, \\ \ hfill xyz = 6 \, \, \, \, \ end {array} [/ latex]

    В следующих упражнениях используйте обратную матрицу для решения систем уравнений.

    [латекс] \ begin {массив} {r} \ hfill 4x-5y = -50 \\ \ hfill -x + 2y = 80 \, \, \, \, \ end {array} [/ latex]

    Показать решение

    [латекс] \ влево (100,90 \ вправо) [/ латекс]

    [латекс] \ begin {array} {r} \ hfill \ frac {1} {100} x- \ frac {3} {100} y + \ frac {1} {20} z = -49 \\ \ hfill \ frac {3} {100} x- \ frac {7} {100} y- \ frac {1} {100} z = 13 \, \, \, \, \\ \ hfill \ frac {9} {100} x- \ frac {9} {100} y- \ frac {9} {100} z = 99 \, \, \, \, \ end {array} [/ latex]

    В следующих упражнениях используйте правило Крамера для решения систем уравнений.